McNeil's notes - stats/epi, coagulation cascade, trauma, cancer (gen, uro only), ICU, fluids, anesthesia, Neurology, SCI/nerve injuries Flashcards

1
Q
A new test is able to identify true positives in 350 patients, true negatives in 1200 patients and false negatives in 150 patients. What is the sensitivity of the test?
A. 60%
B. 70%
C. 80%
D. 90%
A

Answer: 70%

Sen = TP/TP + FN

How well did you know this?
1
Not at all
2
3
4
5
Perfectly
2
Q

When the test has 95% sensitivity, this means:

a. 95% of the patient will be positive
b. Patients who are tested positive, 95% of them will have the disease
c. Patients who are tested positive, 5% will have the disease
d. Of the patients who have the condition, the test will detect 95% of them

A

d. Of the patients who have the condition, the test will detect 95% of them.

sensitivity is the ability of the test to identify correctly those who have the disease (SNOUT-highly sensitive test, a negative result helps rule out disease); specificity is the ability of the test to identify correctly those who do not have the disease (SPIN-a highly specific test, a positive result helps rule in the disease).

How well did you know this?
1
Not at all
2
3
4
5
Perfectly
3
Q

A new blood test is available to diagnose pulmonary embolus. The data from a trial of 1000 post operative patients is shown in the 2x2 table below:
PE Present PE Absent
Test Positive 95 100
Test Negative 5 800

1) The sensitivity of the above test is calculated by the equation
a. 800/(100+800)
b. 800/(800+5)
c. 95/(95+5)
d. 95/(95+100)
e. none of the above

A

c. 95/(95+5)

Sensitivity = true positive (TP)/true positive + false negative

How well did you know this?
1
Not at all
2
3
4
5
Perfectly
4
Q
A new blood test is available to diagnose pulmonary embolus.  The data from a trial of 1000 post operative patients is shown in the 2x2 table below:
		PE Present	PE Absent
Test Positive	  95		  100
Test Negative	  5		  800
2) The specificity of the above test is calculated by the equation
a. 800/(100+800)
b. 800/(800+5)
c. 95/(95+5)
d. 95/(95+100)
e. none of the above
A

a . 800/(100+800)

Specificity = true neg / (true neg + false positive)

How well did you know this?
1
Not at all
2
3
4
5
Perfectly
5
Q
A new blood test is available to diagnose pulmonary embolus.  The data from a trial of 1000 post operative patients is shown in the 2x2 table below:
		PE Present	PE Absent
Test Positive	  95		  100
Test Negative	  5		  800
3) The positive predictive value of the above test is calculated by the equation:
a. 800/(100+800)
b. 800/(800+5)
c. 95/(95+5)
d. 95/(95+100)
e. none of the above
A

d. 95/(95+100)

PPV = true positive/(true positive + false positive)

How well did you know this?
1
Not at all
2
3
4
5
Perfectly
6
Q

A test that detects the number of people who actrally have the disease measures …

a) specificity
b) sensitivity
c) positive predictive value
d) negative predictive value

A

Answer given: sensitivity. Disagree however - this sounds more like PPV.

sensitivity – the proportion (%age) of truly diseased people identified as diseased by a screening test
specificity - the proportion (%age) of truly non-diseased prople identified as non-diseased by the screening test
PPV - the proportion of true positives in all positive tests (the number of cases that truly have disease among all those who test positive)
NPV – the proportion of true negatives in all the negative tests

How well did you know this?
1
Not at all
2
3
4
5
Perfectly
7
Q
Which of the following is the MOST helpful in establishing a causal relationship between exposure and disease?
A. Positive predictive value
B. Sensitivity
C. Odds ratio
D. T-test

Odds ratio = retrospective studies
Risk = prosp[ective studies

A

Answer: Odds ratio

How well did you know this?
1
Not at all
2
3
4
5
Perfectly
8
Q

A world health organization epidemiologist is studying esophageal cancer in females. In Canada, 5,000,000 females over the age of 25 years have been followed from January 1, 1980 to January 1, 2000. Within this population, a group of 1,000,000 women chronically exposed to sulphur dioxide fumes are found to have an increased incidence of developing esophageal cancer as compared to the 4,000,000 that were not chronically exposed. The data is shown below:
Group Esophageal Cancer Incidence (per 1,000,000)
Exposed 100
Not exposed 20

Calculate the relative risk of developing esophageal cancer in those women chronically exposed to sulphur dioxide fumes

a) 100
b) 20
c) 5
d) 0.2
e) none of the above

A

Answer: 5

Relative Risk = A/ A+B
C/ C+D

RR = 100/1,000,000
80/4,000,000

   = 	100/1,000,000
20/1,000,000	

   =	5
How well did you know this?
1
Not at all
2
3
4
5
Perfectly
9
Q

A new treatment changes the mortality of acute MI from 26 % in the placebo group to 16 % in the treatment group. The number needed to treat is:

a. 10
b. 100
c. 200
d. 1000

A

a. 10

ARR = |CER - EER| = |26% - 16%| =10%
NNT = 1/ARR = 1/.1 = 10
How well did you know this?
1
Not at all
2
3
4
5
Perfectly
10
Q

If β blockers decrease risk of MI by 25% and the mortality of MI is 1% in one year, what is the absolute risk reduction and the number of patients needed to be treated to decrease mortality by one patient:

a. 0.25% - 400 patients.
b. 2.5% - 40 patients.
c. 25% - 4 patients.
d. 1% - 10 patients

A

Answer: 0.25% - 400 patients

ARR = |CER - EER|
where
CER = control group event rate (1%)
EER = experimental group event rate (1% * [1- 0.25] = 0.75%)
ARR = 1% - 0.75% = 0.25%
How well did you know this?
1
Not at all
2
3
4
5
Perfectly
11
Q

Power is:

a. Probability to detect statistically significance if one exists
b. A calculation of sample size
c. A calculation of validity
d. Positive predictive value
e. 1 – sensitivity
f. is not related to specificity
g. a stastistic that is not dependent on the prevalence
h. Calculation of the sample size needed to determine if a difference exists

A

Answer: Probability to detect statistically significance if one exists

The power of a trial is the probability of detecting a treatment effect of a given size, if one truly exists.[65] [66] [67] [68] Studies are usually designed to have a power of 0.80 or greater. Because the power of the trial is the chance of finding a true treatment effect, the quantity (1 - power) is the chance of missing a true treatment effect (i.e., risk of committing a type II error).[65] [67] The value of (1?β), or the power, and the magnitude of the treatment effect the clinical trial is designed to detect (defined by the alternative hypothesis) determine the sample size required for the study.[68]

How well did you know this?
1
Not at all
2
3
4
5
Perfectly
12
Q

Regarding statistical errors all are true EXCEPT:

a. The probability that the null hypothesis is considered false when it is true is called the beta
b. In a fixed sample population α error is inversely proportional to the β error.
c. Increasing the sample size decreases α error but doesn’t change the β error.
d. Power equals (1-beta)

A

Power: The probability of detecting an effect in the treatment vs. control group if a difference actually exists. Must also specify the size of the difference. For example, a paper describing a clinical trial with a new hypertension medication may contain the following statement - “The study had a power of 80% to detect a difference of 5 mm Hg in diastolic blood pressure between the treatment and control groups.” Typical power probabilities are 80% or greater. Power = 1 - ß

Type I Error: Mistakenly rejecting the null hypothesis when it is actually true. The maximum probability of making a Type I error that the researcher is willing to accept is call alpha (a). Alpha is determined before the study begins. False positive conclusion. Studies commonly set alpha to 1 in 20 (=0.05).

Type II Error: Mistakenly accepting (not rejecting) the null hypothesis when it is false. The probability of making a Type II error is called beta (b). Power = 1 - b (see above). False negative conclusion. For trials the probability of a b error is usually set at 0.20 or 20% probability. A 20% chance of missing a true difference.

How well did you know this?
1
Not at all
2
3
4
5
Perfectly
13
Q

Which is the most appropriate test to compare the means of 2 normal distributions?

a. Chi square
b. t-test
c. ANOVA
d. Fischer exact test
e. Variance

A

Answer: t-test

Student’s t test for independent samples is used to determine whether two samples were drawn from populations with different means.

Needs to be normally distributed population

How well did you know this?
1
Not at all
2
3
4
5
Perfectly
14
Q

A study has been designed to assess the wear properties of two different metal interfaces for total hip arthroplasty. There are 11 patients in one group and 13 in the other. Which test would best determine if a difference exists between these two groups?

a) t-test
b) Fischer exact test
c) Chi-square test
d) Linear regression

A

? T-test if the outcome is the same and normal distribution. Need more info.

Chi-square test - Used with categoric variables (two or more discrete treatments with two or more discrete outcomes) to test the null hypothesis that there is no effect of treatment on outcome; assumes at least five expected observations of each combination of treatment and outcome under the null hypothesis
Fisher’s exact test - Used similar to chi-square test; may be used even when fewer than five observations are expected in one or more categories of treatment and outcome
- Marx: Rosen’s Emergency Medicine, 7th ed.

How well did you know this?
1
Not at all
2
3
4
5
Perfectly
15
Q

Which test used to measure proportion :

a. Chi square
b. t-test
b. Nova
c. None of the above
d. All of the above

A

Answer: Chi square

How well did you know this?
1
Not at all
2
3
4
5
Perfectly
16
Q

To complete a retrospective study with a dichotomous outcome i.e. yes or no, the following tests could be used except:

a) Chi Square
b) Fischer T
c) Students T
d) Odds ratio

A

T-test

a) Chi Square (yes)
b) Fischer T (used for categorical data)
c) Students T (measure differenceb/w two means)
d) Odds ratio (yes)

How well did you know this?
1
Not at all
2
3
4
5
Perfectly
17
Q
Doing a study of femoral head size (26- 28-32 and 36) in conjuntion with rate of dislocation.  What is the most appropriate statistical test?
 a - student t teat
 b – fisher exact test
 c – ANOVA
 d – chi squared
A

Chi squared?

Z-test/t-test tests differences between two sample means for continuous data.

Chi-square test - Used with categoric variables (two or more discrete treatments with two or more discrete outcomes) to test the null hypothesis that there is no effect of treatment on outcome; assumes at least five expected observations of each combination of treatment and outcome under the null hypothesis

Fisher’s exact test - Used similar to chi-square test; may be used even when fewer than five observations are expected in one or more categories of treatment and outcome

Analysis of variance compares mean values from three or more groups simultaneously

How well did you know this?
1
Not at all
2
3
4
5
Perfectly
18
Q
Define standard deviation. 
A. Difference between mean and median
B. Measure of variance and dispersion
C. The midpoint in a series of numbers
D. Measure of dispersion around the mode
A

Answer: Measure of variance and dispersion

The standard deviation is the average deviation of scores around the mean of the variable for the set of observations in the sample. It is often designated SD.
- Tasman: Psychiatry, 1st ed

How well did you know this?
1
Not at all
2
3
4
5
Perfectly
19
Q

Given a normal distribution 1 standard deviation is:

a. 2/3
b. 99/100
c. 3/4
d. 1/3
e. None of the above

A
  • 1 SD = 68%, 2 SD = 95%, 3 SD = 99%

The standard deviation is the square root of the variance. For a normally distributed population, 68% of values fall within 1 standard deviation of the mean and 95% of values within 1.96 standard deviations.
- Long: Principles and Practice of Pediatric Infectious Diseases, 3rd ed.

How well did you know this?
1
Not at all
2
3
4
5
Perfectly
20
Q

Patients are administered a survey with pain scale to check the effectiveness of a new non-steroidal antiinflammatory drug. The mean is 12, the mode is 8, median is 10, and SD is 11. Which of the following is true

a) most common value is 10
b) the average is 8
c) 68% of values will fall between 1 and 23

A

c.

a) most common value is 10 (8)
b) the average is 8 (12)
c) 68% of values will fall between 1 and 23 (12 +/-11) one SD

How well did you know this?
1
Not at all
2
3
4
5
Perfectly
21
Q

What is the variance of 3,6,9,11:

a. 3.00
b. 5.00
c. 7.00
d. 11.00
e. 29.00

A

Answer is none of the above. (12.25)

Summary of the calculation procedures:

  1. subtract the mean from each score
  2. square each result
  3. sum all the square
  4. divide the sum of square by N. Now you get variance
How well did you know this?
1
Not at all
2
3
4
5
Perfectly
22
Q
All of the following represent quantitative continuous data EXCEPT: 
A. Age 
B. BP
C. Number of asthma attacks per month
D. Height
A

Answer: Number of asthma attacks per month

How well did you know this?
1
Not at all
2
3
4
5
Perfectly
23
Q
Which of the following is an example of ordinal data? (Ordinal = order) 
 A – stages of breast cancer
 B – ABO blood type
 C – sex
 D – Death / Life
A

Answer: stages of breast cancer

Ordinal data are categorical data where there is a logical ordering to the categories. A good example is the Likert scale that you see on many surveys: 1=Strongly disagree; 2=Disagree; 3=Neutral; 4=Agree; 5=Strongly agree.
- http://www.cmh.edu/stats/definitions/ordinal.htm

How well did you know this?
1
Not at all
2
3
4
5
Perfectly
24
Q

What does the Kaplan-Meier Curver measure?

A

Answer: survival

How well did you know this?
1
Not at all
2
3
4
5
Perfectly
25
Q

When using median as a measure of central tendency, when is it best?

a. slightly skewed
b. normal distribution

A

Answer: Normal distribution

The median is primarily used for skewed distributions, which it summarizes differently than the arithmetic mean. Consider the multiset { 1, 2, 2, 2, 3, 9 }. The median is 2 in this case, as is the mode, and it might be seen as a better indication of central tendency than the arithmetic mean of 3.166.
- for a normal distribution, mean = median = mode.

How well did you know this?
1
Not at all
2
3
4
5
Perfectly
26
Q

You have a mean of 12, median of 10, and average (?) (probably mean mode) of 8. What is the distribution? (where is the tail?)

a) Negatively skewed
b) Positively skewed
c) Normal distribution

A

Answer: Positively skewed

This is a positively skewed curve, because the mean is greater than the mode.

In probability theory and statistics, skewness is a measure of the asymmetry of the probability distribution of a real-valued random variable. The skewness value can be positive or negative, or even undefined. Qualitatively, a negative skew indicates that the tail on the left side of the probability density function is longer than the right side and the bulk of the values (possibly including the median) lie to the right of the mean. A positive skew indicates that the tail on the right side is longer than the left side and the bulk of the values lie to the left of the mean.

Negative skew: The left tail is longer; the mass of the distribution is concentrated on the right of the figure. The mean is smaller than the median.
Positive skew: The right tail is longer; the mass of the distribution is concentrated on the left of the figure. The mean is larger than the median.

How well did you know this?
1
Not at all
2
3
4
5
Perfectly
27
Q

Compare three groups of patients blood pressures. Which is appropriate test?

  • Analysis of variance
  • t test
  • chi squared
A

Answer: Analysis of variance

Analysis of variance compares mean values from three or more groups simultaneously.

Chi-square test tests difference between proportions or tests for association between categories. Used when variable is categorical.

How well did you know this?
1
Not at all
2
3
4
5
Perfectly
28
Q

If the study has CI of 95%. What is the chance to have population out side this interval:

a. 1:5 (20%)
b. 1:10 (10%)
c. 1:20 (5%)
d. 1:95 (1-2%)

A

Answer: 1:20 (ie 5%)

How well did you know this?
1
Not at all
2
3
4
5
Perfectly
29
Q

In which situation would a surgeon not be liable?

a) Failure to diagnose a breast lump
b) Undue delay in diagnosing a malignant breast lump
c) Performing a mastectomy without explaining the other options
d) Failing to refer for adjuvant chemotherapy
e) Recurrence of breast malignancy, when patient has entered a clinical trial of adjuvant therapy

A

e) Recurrence of breast malignancy, when patient has entered a clinical trial of adjuvant therapy

They consent to this

How well did you know this?
1
Not at all
2
3
4
5
Perfectly
30
Q

In the OR, the surgeon is responsible for:

a) Anesthesiologist
b) Scrub nurse
c) Secretary in office
d) Med student observer

A
  • Med student observer
How well did you know this?
1
Not at all
2
3
4
5
Perfectly
31
Q

The surgeon is responsible for:

a. The anesthesiologists
b. The OR nurses
c. The student observers
d. The secretaries he employees in his office

A
  • The secretaries he employees in his office
How well did you know this?
1
Not at all
2
3
4
5
Perfectly
32
Q

A patient undergoes surgery and a complication arises. Surrounding the issue of disclosure, which of the following is correct:

a) the patient doesn’t need to know of any medical errors which took place if there are no immediate consequences
b) the patient should be informed of all medical errors as is their right for autonomy
c) disclosure of medical errors often leads to lawsuits
d) every attempt should be made to prevent the patient from learning about the medical error

A

b?

How well did you know this?
1
Not at all
2
3
4
5
Perfectly
33
Q

Disclosure of medical errors.
A – more ligation if error is disclosed
B – no legal obligation to disclose error
C – disclosure is in keeping with principles of automony
D – only disclose if there is harm to patient and they will find out anyways

A
  • disclosure is in keeping with principles of automony?
How well did you know this?
1
Not at all
2
3
4
5
Perfectly
34
Q

Which is most associated with the authorship of a publication:

a) participated in the writing of the paper
b) provided lab space
c) developed the background behind the hypothesis question
d) responsible for the content of the publication
e) assisted in the technical aspect of the experiment
f) provision of grants

A

Answer: responsibility for content of the publication

How well did you know this?
1
Not at all
2
3
4
5
Perfectly
35
Q

6 yr old kid is in car crash. Dad is killed. Mom is alive and OK. Mom is Jehovah’s witness. Kid urgently needs transfusion and laparotomy. Mom insists that kid be given no transfusion. What to you do?

a. disregard Mom, take kid to OR, give blood as needed to save her
b. call children’s aid society
c. obey Mom’s wishes no matter what the expected outcome
d. arrange for a substitute decision maker to give consent for OR and transfusion

A

Disregard Mom, take kid to OR, give blood as needed to save her. Kid is too young/does not have the capacity to reject treatment/be Jehovah? No time to call CAS - deal with it later.

Rule: Must contact CAS but if threat is imminent, then transfuse right away

How well did you know this?
1
Not at all
2
3
4
5
Perfectly
36
Q

Post-op a patient is found to have unresectable pancreatic ca. The patient has a history of depression and the wife asks you to not tell him for he would not be able to cope mentally. Your next course of action should be:

a. tell the patient
b. give the patient a forum to ask about operative findings and their interest in knowing all details
c. Lie to the patient

A
  • give the patient a forum to ask about operative findings and their interest in knowing all details
How well did you know this?
1
Not at all
2
3
4
5
Perfectly
37
Q

A 67 year old male undergoes an emergent laparotomy for peritonitis. He is noted to have a perforated cecal carcinoma with intraperitoneal carcinomatosis and multipe metastasis to his liver. After telling the family the bad news and explaining to them his poor prognosis, they ask you not to tell him of his dismal prognosis. You the surgeon should:

a. Consult the hospital lawyer
b. Consult the hospital ethics review board
c. Explain to the family your obligation to your patient and tell the patient
d. Explain to the family your obligation to your patient and tell the patient only if he asks
e. Do as the family wishes

A
  • Explain to the family your obligation to your patient and tell the patient
How well did you know this?
1
Not at all
2
3
4
5
Perfectly
38
Q

Patient plans on committing a violent crime, what should you do:

a. Nothing
b. Document it in the chart and tell no body
c. Tell the police
d. Discuss it with patient

A

Answer: Tell the police

Duty to Warn (look it up)
If there is a threat, then you should tell police, person in question, or holding person against will

How well did you know this?
1
Not at all
2
3
4
5
Perfectly
39
Q

Patient is planning to sue a surgical team whom you treated few weeks age. He came to ER with intestinal obstruction and you are the oncall for surgery. what should you do:

a. Refuse to see and treat the patient
b. Transfer him to another hospital
c. See the patient and ask a doctor who was not involved in his treatment before to look after him
d. ask the patient not to suit the team if he wants to be treated

A

Answer: See the patient and ask a doctor who was not involved in his treatment before to look after him

How well did you know this?
1
Not at all
2
3
4
5
Perfectly
40
Q

Patient with advanced metastatic cancer. When wants every thing to done for him. You think he should be palliative and DNR. What kind of discussion will not be appropriate:

a. Negotiation
b. Arbitration
c. Rationalism

A

Answer: Arbitration

Arbitration – legal technique for the resolution of disputes outside of courts, wherein the parties to a dispute refer it to one or more persons, by whose decision they agree to be bound
Negotiation – a process of resolving disputes and conflicts via talks and discussions without using force
Rationalism – any view appealing to reason as a source of knowledge or justification
Autonomy – capacity of a rational indivdual to make a informed, uncoerced decision

How well did you know this?
1
Not at all
2
3
4
5
Perfectly
41
Q

After opening of the abdomen of a patient with you staff in the OR, You as a junior resident found out that your staff is drunk, What should you do:

a. Continue the procedure
b. Take over and let scrub out
c. As him to close the abdomen and reschedule the procedure
d. Tell the chief.

A

Answer: Tell the chief.

How well did you know this?
1
Not at all
2
3
4
5
Perfectly
42
Q

You are newly licensed, and are preparing to do your first total hip replacement as staff surgeon. Your obligation is:

a) Tell the patient this is your first one
b) Tell the patient this is your first one, and refer to someone more experienced
c) Don’t disclose to patient, because you did several as a resident
d) Don’t disclose to patient, because having your specialty certification implies a level of competence

A

A?

The legal standard of risk disclosure expected of a physician hinges on the interpretation of the entity called “material risk.” Any impairment of the physician related to drug usage, disease, or alcohol which compounds the risk of a procedure is very likely to be considered material by a patient. This paper argues that physician inexperience is a factor that a reasonable patient would attach significance to and that it should therefore be viewed as a “material risk” requiring disclosure.

How well did you know this?
1
Not at all
2
3
4
5
Perfectly
43
Q

In attempting to practice medicine that pays attention to minimizing economic burden/appropriate resource use, you should do all of the following, EXCEPT:

a) Avoid inappropriate tests
b) If choice exists, use less costly materials
c) Give your patients advantages
d) Treat patients on first-come first-serve basis

A

c) Give your patients advantages
d) Treat patients on first-come first-serve basis (also a silly thing since a prescription refill should not be seen before a trauma)

How well did you know this?
1
Not at all
2
3
4
5
Perfectly
44
Q

A patient tells you that he intends to harm another person. What is your legal responsibility?

a) Chart the incident and tell no one
b) Inform the person in question
c) Call psych
d) Inform the police

A

d) Inform the police (this is legally what you should be doing)

Duty to warn

How well did you know this?
1
Not at all
2
3
4
5
Perfectly
45
Q

You are doing a laminectomy on a patient, and mistakenly entered the dural space (i.e., dural tear). You should:

a) Repair and no further management required
b) Don’t tell patient about complication, but monitor for CSF leak post-operatively
c) Tell patient about intra-op complication, possible sequelae, and what to look for (i.e., symptoms of CSF leak)
d) Call the hospital lawyer, and risk management

A

c) Tell patient about intra-op complication, possible sequelae, and what to look for (i.e., symptoms of CSF leak)

How well did you know this?
1
Not at all
2
3
4
5
Perfectly
46
Q

Negligence is breach of legal duty. Definition is all except:

1) Physican has duty to patient
2) Duty was breached
3) Patient is harmed
4) Harm is direct cause of break of duty
5) Physician provides good care

A

5) Physician provides good care

Negligence = Not doing the standard of care, which is what another reasonable physician would have done in the same situation. NB: cannot be blamed unless this would have affected the patient → poor outcome (doesn’t count if it doesn’t influence management).

How well did you know this?
1
Not at all
2
3
4
5
Perfectly
47
Q

A 75 yo male with newly diagnosed adenocarcinoma in his right lower lobe presents with ischemic bowel. Preoperatively he says that he doesn’t want any heroic measures taken. Postoperatively he remains intubated, and develops renal failure. If you withhold hemodialysis, which of the following is true?

a. Liable for negligence.
b. Liable in a civil lawsuit by family.
c. Would be deemed unethical by his colleagues.
d. Would suffer no consequences.

A

Answer given: Liable for negligence

One could argue hemodialysis is not a heroic measure.

How well did you know this?
1
Not at all
2
3
4
5
Perfectly
48
Q

After finishing your research under supervision of your staff you are leaving to another lab. You are obliged / entitled to :

a. Take all research material with you.
b. Ask permission to take a copy to continue with your own research.
c. Leave a copy in case data are required for research authentication.
d. Not take anything

A

Answer: Not take anything

How well did you know this?
1
Not at all
2
3
4
5
Perfectly
49
Q

Patient in ER unconscious, driver other car killed, you smell Etoh. Police ask for blood sample for etoh level
A – give it to them
B – draw blood for you and give result only if court order
C – draw blood for your use and save sample for police
D – draw blood for police if patients family consents

A

Answer: draw blood for you and give result only if court order

The confidentiality of patient information is prescribed in law. For example, physicians in Ontario are prohibited from providing information to third parties regarding a patient’s condition or any professional service performed for a patient without the consent of the patient or his or her authorized agent unless such disclosure is required by law.[2] A breach of confidentiality that is not required by law may prompt disciplinary action by the College of Physicians and Surgeons of Ontario. Similar provisions concerning confidentiality exist in other provinces. Moreover, a breach of confidentiality may result in a civil suit.
CMAJ ethics online
http://collection.nlc-bnc.ca/100/201/300/cdn_medical_association/cmaj/vol-156/issue-4/0521.htm

How well did you know this?
1
Not at all
2
3
4
5
Perfectly
50
Q
A patient is not capable of making their own decision. which is not an appropriate way to seek guidance about medical decisions?
 a- personal directive
 b – family members
 c – appointed patient advocate
 d – hospital staff
A

d (consent)

How well did you know this?
1
Not at all
2
3
4
5
Perfectly
51
Q

Informed consent must includes all of the following except:

a. general risk
b. specific risk
c. alternative therapy
d. percentage of failure and success
e. Special and unique risks

A

e. special and unique risks

The law presumes that an adult is mentally competent to make medical decisions and that the competent adult is entitled to sufficient information to make an informed decision concerning the physician’s proposed course of examination and treatment.[74] Under the doctrine of informed consent, physicians have the duty to disclose the following information to patients[74-76]:

  1. The patient’s condition and/or diagnosis
  2. The nature and purpose of the proposed treatment, including the likelihood of success in the physician’s practice
  3. Reasonable alternative measures related to the diagnosis and treatment, including the probable outcome of those alternatives
  4. The particular known inherent risks that are material to make an informed decision about whether to accept or reject the proposed treatment, including the consequences of refusing that treatment
    - Marx: Rosen’s Emergency Medicine, 7th ed.
How well did you know this?
1
Not at all
2
3
4
5
Perfectly
52
Q

All are true regarding obtaining consent EXCEPT:

a. Disclosure.
b. Voluntary
c. Capacity.
d. Has to be written.

A

Answer: Has to be written (implied consent ex: lifting up your sleeve for someone to give you a vaccine)

How well did you know this?
1
Not at all
2
3
4
5
Perfectly
53
Q

A pregnant woman is brought to the trauma suite with substantial intra-abdominal trauma. She expresses that she is a Jehovah’s witness and cannot accept blood or blood product transfusion. Her baby is stable at present. Patient consents to undergo surgery and intra-operatively, the patient becomes hypotensive, placing the unborn child at risk. Of the following people, who has the right to act on the patient’s behalf as POA in aiding in the decision making in this situation:

a) the baby’s biological father
b) the patient’s common-law partner
c) the patient’s sibling
d) the physician

A

b) the patient’s common-law partner

order of SDM when patient is not capable (ex: under GA?)

How well did you know this?
1
Not at all
2
3
4
5
Perfectly
54
Q

In disclosure, one must discuss with the patient:
a-what a reasonable physician would say
b-what a reasonable patient wants to know
c-what a reasonable patient in those circumstances needs to know

A

For consent to be valid, it must be informed. The patient must be provided all relevant information. To be valid, it must also be voluntary, that is, as free from coercion as possible while recognizing that in extremis the patient’s condition itself may be inherently coercive. The surgeon’s ethical objective is to judiciously provide the patient sufficient information with which to decide what course to follow. This entails selectively presenting all information pertinent to the patient’s condition regarding benefits, risks, and alternatives while avoiding overwhelming the patient with extraneous data. To walk the line between what is pertinent and what is extraneous requires prudent judgment.
- Townsend: Sabiston Textbook of Surgery, 18th ed.

How well did you know this?
1
Not at all
2
3
4
5
Perfectly
55
Q

While you are explaining the risks of surgery to a patient with breast cancer she appears not to be listening and says that she doesn’t want to hear these bad things and wants you to do the operation. You should:

a. Insist that she listens or you will cancel the surgery.
b. Consult another surgeon to try to explain the procedure.
c. Ask a next of kin or a patient advocate to assess the risks and benefits.
d. Document that you explained the risks and proceed with the surgery.

A

Answer given: Ask a next of kin or a patient advocate to assess the risks and benefits

How well did you know this?
1
Not at all
2
3
4
5
Perfectly
56
Q

A 45 y old man is rushed to the OR after identification of a leaking aortic aneurysm on CT scan in the trauma evaluation following a high speed MVC. The patient declares that he is a Jehovah’s witness and therefore consented to the surgery but not to the administration of any blood or blood products. During the surgery the patient’s blood pressure falls and it becomes clear the patient will likely not survive without blood transfusions. The nurse then comes into the OR stating that the wife is outside and threatens to sue you unless you do everything you can to save her husband, emphasizing that you should administer blood if that’s “what he needs”. The most appropriate action at this time is:

a) listen to the wife because she is the POA while the patient is incapable of making this decision on his own under the general anesthetic
b) administer blood and disguard the evidence
c) do not administer any blood products and continue to rescusitate the patient to the best of your ability

A

Answer: Support with crystalloid but don’t give blood

How well did you know this?
1
Not at all
2
3
4
5
Perfectly
57
Q

Which of the following applies to consent EXCEPT?

a. not a discreet process but continuous
b. finished once you sign on the dotted line

A

b

How well did you know this?
1
Not at all
2
3
4
5
Perfectly
58
Q

A 57 year old male undergoes a resection of his left parotid gland for adenocarcinoma. Upon waking he is unable to smile on his left side. The intern who did the pre-operative history and physical and obtained the consent did not list or explain the complications of this surgery to the patient or their family.
As the surgeon of record, you should
a) call the hospital legal counsel immediately
b) avoid discussing this finding with the patient
c) fail the intern
d) explain to the patient what has happened
e) consult plastic surgery for a sural nerve graft

A

Answer: explain to the patient what has happened

How well did you know this?
1
Not at all
2
3
4
5
Perfectly
59
Q

PSA screening and its increase in detection and decrease in mortality is invalidated by all EXCEPT one of the following:

a) lead time bias
b) length time bias
c) overdetection
d) selection bias
e) stage migration

A

Answer: stage migration (???)

Bias is a systematic error in making inference (assumptions). Any bias would invalidate results, therefore a), b) and d) are not correct answers.

Stage migration describes change in the distribution of stage in a particular cancer population induced by either a change in the staging system itself or else a change in technology which allows more sensitive detection of tumor spread and therefore more sensitivity in detecting spread of disease (e.g., the use of MRI scan). Stage migration can lead to curious statistical phenomena.
- http://en.wikipedia.org/wiki/Cancer_staging

How well did you know this?
1
Not at all
2
3
4
5
Perfectly
60
Q
Which carries the GREATEST risk of life-threatening post-op cardiac complication for non-cardiac surgery?
A. CHF
B. Greater than 5 PVC/min
C. MI within last 6 months
D. Arterial pO2
A

Answer: CHF

Based on goldman classification CHF has the highest score

How well did you know this?
1
Not at all
2
3
4
5
Perfectly
61
Q

Which one of the following is not associated with an increased cardiac mortality
a. surgery

A
Answer: intraoperative hypotension or class II angina
??
How well did you know this?
1
Not at all
2
3
4
5
Perfectly
62
Q

Exposure in operating room personal leads to increased risk of:

Answer: Spontaneous abortion (old data, likely not to be on our exam cause looks like it’s no longer the case)

A

Answer: Spontaneous abortion

[…] Nevertheless, as late as 1997 a meta-analysis of over 19 studies completed between 1984 and 1992 reported an RR of abortion in females exposed to anesthetic gases to be 1.48 (CI 95%) 1.4-1.58.
[…] This preliminary report showed the incidence of infertility, spontaneous abortion and children with congenital abnormalities in female anesthesiologists to be the same as that in other physicians […]
No definitive evidence has shown that trace concentrations of anesthetics in the ambient air of the operating room present a health hazard.
- Miller: Miller’s Anesthesia, 7th ed.101 – Environmental Safety Including Chemical Dependency

There used to be “strong” evidence but now mostly debunked.

How well did you know this?
1
Not at all
2
3
4
5
Perfectly
63
Q

Diabetic on insulin and had MI 18 months going to inguinal hernia:

a. ASA II
b. ASA III
c. ASA IV
d. ASA V

A

Answer: ASA III

I. A normal healthy patient.
II. A patient with mild systemic disease.
III. A patient with severe systemic disease.
IV. A patient with severe systemic disease that is a constant threat to life.
V. A moribund patient who is not expected to survive without the operation.
VI. A declared brain-dead patient whose organs are being removed for donor purposes.

How well did you know this?
1
Not at all
2
3
4
5
Perfectly
64
Q

1) ALL of the following carry high mortality rate except:
a. Gallop 3 (CHF)
b. MI 18 months
c. Valve disease
d. CAGB 3 years ago

A

CABG 3 years ago

How well did you know this?
1
Not at all
2
3
4
5
Perfectly
65
Q

2) Highest risk of major cardiac event following noncardiac surgery
a. presence of heart failure
b. subendocardial infarct in last 2 months

A

Presence of heart failure

How well did you know this?
1
Not at all
2
3
4
5
Perfectly
66
Q

3) Which of the following does not increase the risk of a peri-operative MI
a. mi within the previous 6 months
b. angina functional class II
c. s3 gallop
d. aortic stenosis
e. age > 70 years

A

3) Angina functional class II

Perioperative MI probably is the leading cause of death in elderly after noncardiac surgery;

  • presence of CAD increases the incidence of peri-op MI from 0.1 to 0.7-1.1%; >40 with or without CAD: infarction rate is 1.8%; previous MI, reinfarction rate is 5-8%; MI within 3 months: reinfarction rate is 27%; 3-6 months: 11%; >6months: 5%;
  • patients with EF 35%); thus, CHF is a higher risk.

a. mi within the previous 6 months - 10 points
b. angina functional class II - not on the cardiac risk index (class III and IV are)
c. s3 gallop (CHF) - see above
d. aortic stenosis - 20 points for critical aortic stenosis
e. age > 70 years- 5 points

How well did you know this?
1
Not at all
2
3
4
5
Perfectly
67
Q

Healthy smoker patient going for surgery. All are correct except

a. At least ASA II
b. High postoperative complication
c. High risk of wound infection

A

Answer: High postoperative complication

How well did you know this?
1
Not at all
2
3
4
5
Perfectly
68
Q

A 62 year old female patient presents as a same day surgery patient. A preoperative EKG reveals a LBBB. The patient denies any symptoms and has no cardiac history. The most appropriate course of action is to:

a. Proceed with surgery
b. Proceed with the surgery, but use regional anesthesia rather than general anesthesia
c. Reschedule surgery and arrange a pre-operative anesthesia consult
d. Cancel the surgery and have the patient admitted to cardiology
e. none of the above

A

c. Reschedule surgery and arrange a pre-operative anesthesia consult

A patient with a new LBBB could have had an infarct. Considering this is elective surgery, the best course of action is to delay the operation and has the patient assessed by anesthesia.

Unlike RBBB, which is occasionally seen without evident cardiac disease, LBBB is usually a sign of organic heart disease. LBBB may develop in patients with long-standing hypertensive heart disease, a valvular lesion (e.g., calcification of the mitral annulus, aortic stenosis, or aortic regurgitation), or different types of cardiomyopathy (see Chapter 11 ). It is also seen in patients with coronary artery disease and often correlates with impaired left ventricular function. Most patients with LBBB have underlying left ventricular hypertrophy (see Chapter 6 ). Rarely, otherwise normal individuals have an LBBB pattern without evidence of organic heart disease by examination, echocardiogram, or even invasive studies.
Important clinical consideration: LBBB may be the first clue to four previously undiagnosed but clinically important abnormalities. These are advanced coronary artery disease, valvular heart disease, hypertensive heart disease, and cardiomyopathy.
- Goldberger: Clinical Electrocardiography: A Simplified Approach, 7th ed.

How well did you know this?
1
Not at all
2
3
4
5
Perfectly
69
Q

Which factor would not be decreased in hepatic failure?

a. 2
b. 5
c. 7
d. 8
e. 10

A

The liver is the major site of synthesis of all the coagulation factors except factor VIII
- Townsend: Sabiston Textbook of Surgery, 18th ed.

How well did you know this?
1
Not at all
2
3
4
5
Perfectly
70
Q

What factor converts prothrombin to thrombin

a. Factor IX
b. Factor X
c. Factor XI

A

10 (actually Xa and Va)

How well did you know this?
1
Not at all
2
3
4
5
Perfectly
71
Q

A 22 y old male is to undergo elective surgery on the knee. In order to get the goal factor VIII of 60-80% normal, what can you do regarding perioperative planning:

a) give replacement factor before the surgery and for 10 days following
b) administer factor VIII immediately after the surgery
c) administer Factor VIII for 2 doses pre-operatively, and one dose post-op
d) give Factor VIII for 5 days following surgery

A

Answer: give replacement factor before the surgery and for 10 days following

In elective surgery, levels of the deficient coagulation factor should be assayed 48 hours before surgery and the level restored to 40% of normal before surgery. Because the half-life is 6 to 10 hours for factor VIII (8h for factor 8) and 8 to 16 hours for factor IX, approximately 1.5 U/hr/kg of factor VIII or 1.5 U/2 hr/kg of factor IX should be given. Additional administration of factors VIII and IX should be guided by the activity of the clotting factors for about 6 to 10 days postoperatively.
- Miller: Miller’s Anesthesia, 7th ed.

How well did you know this?
1
Not at all
2
3
4
5
Perfectly
72
Q

A 36 year old otherwise healthy male with Hemophilia A is about to undergo surgery. His factor VIII level is 80% normal. Which scenario appropriately predicts his post-operative course?

a) No increased risk of bleeding
b) A minor increase in bleeding risk
c) A moderate increase in bleeding risk
d) A major increase in bleeding risk
e) Is not safe to undergo surgery

A

D. nothing. unlikely spontaneous bleed if >5-10%

Hemophilia A is inherited as a sex-linked recessive deficiency of factor VIII, with fewer cases secondary to spontaneous mutation. The incidence of this abnormality is about 1 in 10,000 births. Clinical findings range from bleeding into joints and muscles, epistaxis, hematuria, and bleeding after minor trauma to prolonged postoperative bleeding, retroperitoneal bleeding, and intramural bowel hemorrhage. Laboratory screening tests usually reveal a prolongation of aPTT, with other tests being normal. The minimum level of factor VIII required for hemostasis is 30%, and spontaneous bleeding is uncommon with factor VIII levels greater than 5% to 10% of normal. Levels less than 2% constitute severe, 2% to 5% moderate, and greater than 5% mild deficiency. Severe deficiency with levels less than 1% are at risk for spontaneous bleeding episodes. Although the half-life of factor VIII is 2.9 days in normal subjects, the half-life of factor VIII concentrates is only 9 to 18 hours. Levels between 80% to 100% of normal should be attained for surgical bleeding or life-threatening hemorrhage. Acquired deficiency has been reported to occur with the development of antibodies to factor VIII after therapy.
- Greenfield CD, ch 4

How well did you know this?
1
Not at all
2
3
4
5
Perfectly
73
Q
What levels are required for a hemophiliac to undergo elective surgery?
100%
60-80%
20-40%
greater than 10%
A

In elective surgery, levels of the deficient coagulation factor should be assayed 48 hours before surgery and the level restored to 40% of normal before surgery. Additional administration of factors VIII and IX should be guided by the activity of the clotting factors for about 6 to 10 days postoperatively
- Miller: Miller’s Anesthesia, 7th ed.

To achieve hemostasis for major surgical procedures, an initial level of 60 to 100 percent is achieved. This level is followed by a prophylactic level of 30 to 50 percent until the wound is healed, typically 10 to 14 days. After some orthopedic procedures, factor level may need to be maintained for a longer period of time.

How well did you know this?
1
Not at all
2
3
4
5
Perfectly
74
Q

Pt with Haemophilia, the lowest acceptable level for elective surgery

a. 10%
b. 20%
c. 40%
d. 60%

A

(depends on the site of operation/risk of bleeding):

a. 10%
b. 20%
c. 40% = elective surgery
d. 60% = major surgery and high risk of bleeding

To achieve hemostasis for major surgical procedures, an initial level of 60 to 100 percent is achieved. This level is followed by a prophylactic level of 30 to 50 percent until the wound is healed, typically 10 to 14 days. After some orthopedic procedures, factor level may need to be maintained for a longer period of time.

How well did you know this?
1
Not at all
2
3
4
5
Perfectly
75
Q
1) A woman in hospital is bleeding. The INR is noted to be normal but the activated Partial Thromboplastin time (aPTT) is elevated. Which of the following MOST likely explains her problem?
A. Medication induced
B. Warfarin
 C. Deficiency of factor VII
D. Unrecognized bleeding dyscrasia
A

Answers:

1) Medication induced (most likely heparin for inpatients)

How well did you know this?
1
Not at all
2
3
4
5
Perfectly
76
Q

2) A patient has a prolonged PTT but normal INR. Which of the following could BEST explain these findings?
a. deficiency of factor VII
b. deficiency of factor VIII
c. dysfibrogenemia
d. X inhibitor

A

b. deficiency of factor VIII

a. deficiency of factor VII (INR)
b. deficiency of factor VIII (yes, intrinsic pathway)
c. dysfibrogenemia (both)
d. X inhibitor (both)

How well did you know this?
1
Not at all
2
3
4
5
Perfectly
77
Q

3) Bleeding patient has normal INR but abnormal APTT , most likely cause is :
a. Platelet dysfunction.
b. Drug induced coagulation defect.
c. Undiagnosed bleeding dyscriasis.
d. Cirrhosis.

A

Undiagnosed bleeding dyscriasis.

a. Platelet dysfunction. (depends, may be both)
b. Drug induced coagulation defect. (depends on factors inovolved)
c. Undiagnosed bleeding dyscriasis.
d. Cirrhosis. (Both)

How well did you know this?
1
Not at all
2
3
4
5
Perfectly
78
Q

4) young, male patient, pre-op for elective surgery, found to have isolated prolonged PTT, with no history of bleeding problems, most likey diagnsosis
a. lack of factor XI
b. lack of factor XIII
c. acquired antibody to factor VIII

A

4) lack of factor XIII

When aPTT is performed in conjunction with PT (INR), a further clarification of coagulation defects is possible. For example, a normal INR with an abnormal aPTT means that the defect lies within the first stage of the (intrinsic) clotting cascade (factors VIII, IX, X, XI, and/or XII). The pattern of a normal aPTT with an abnormal INR suggests a possible factor VII deficiency (extrinsic). If both INR and aPTT are prolonged, a deficiency of factor I, II, V, or X is suggested (common pathway). Used together, APTT and INR will detect approximately 95% of coagulation defects.

How well did you know this?
1
Not at all
2
3
4
5
Perfectly
79
Q

Patient bleeds after dental extraction. Bleeding time and aPTT increased but INR is normal. Normal capillary frag. and clot retraction.

a. VW
b. Factor IX
c. Glandsmans disease
d. Factor VII

A

Answer: a. VW

vWF has 2 functions

1) platelet aggregation (bleeding time)
2) carrier for factor 8 (intrinsic pathway – PTT)

  • aPTT prolonged in (Factor 12, 8, 9, 11 – deficiencies), Lupus Anticoagulant,
  1. Activated partial thromboplastin time (aPTT): Measures intrinsic system; requires factors V, VIII, IX, X, XI, XII, fibrinogen, and prothrombin. May be prolonged in heparin administration, in hemophilia, in von Willebrand disease (vWD), in DIC, and in the presence of circulating inhibitors (e.g., lupus anticoagulants or other antiphospholipid antibodies).
  2. Prothrombin time (PT): Measures extrinsic pathway; requires factors V, VII, X, fibrinogen, and prothrombin. May be prolonged in warfarin administration, in deficiencies of vitamin K–associated factors, in malabsorption, in liver disease, in DIC, and in the presence of circulating inhibitors.
  3. Bleeding time (BT): Evaluates clot formation, including platelet number and function, and von Willebrand factor (vWF). Performed at patient’s bedside. Always assess the platelet number and history of ingestion of platelet inhibitors, such as nonsteroidal anti-inflammatory drugs, before a BT test. The Platelet Function Analyzer-100 (PFA-100) system is another in vitro method for measuring platelet and vWF function.
    - Johns Hopkins: The Harriet Lane Handbook, 18th ed.
How well did you know this?
1
Not at all
2
3
4
5
Perfectly
80
Q

Which of the following mechanisms BEST explains the coagulopathy associated with severe hemorrhagic shock?
A. Decreased circulating coagulation factors
B. Decreased circulating platelets
C. Acidosis and hypothermia
D. Increased fibrinolysis

A

Answer: Acidosis and hypothermia

The wording of this question is a bit misleading. The acidosis and hypothermia are usually associated with transfusion, not the shock itself.

How well did you know this?
1
Not at all
2
3
4
5
Perfectly
81
Q
What is the half-life of factor VIII?
A. 3-6 hours
B. 8-12 hours
C. 1-2 days
D. 4-7 days
A

Easy way to remember: factor 8 = 8 hours.

vWF stabilizes factor VIII, which has a half-life of about 2.4 hours when free and 12 hours when bound to vWF in the circulation.
- Kumar: Robbins and Cotran Pathologic Basis of Disease, Professional Edition , 8th ed.

In its circulating form, factor VIII is inactive or minimally active as a cofactor in blood coagulation. It circulates in the blood at very low concentration (100 ng/mL) bound to vWF. Its plasma half-life is approximately 8 to 12 hours.
- Hoffman: Hematology: Basic Principles and Practice, 5th ed.

Because the half-life is 6 to 10 hours for factor VIII ….
- Miller: Miller’s Anesthesia, 7th ed.

How well did you know this?
1
Not at all
2
3
4
5
Perfectly
82
Q
1) Which factor deficiency is MOST likely to cause a high INR but normal PTT?
A. VII
B. VIII 
C. IX 
D. XI
A

Answers given:
1) VII

INR : 1972 : X, IX, VII, II (rules out B and D in 1st stem)
Hemophilia B is Factor IX deficiency : elevated PTT (rules out C)
Also, VIII, IX, and XI are in intrinsic (PTT) pathway, while VII is extrinsic pathway (INR)

Profiles of Hemostasis Screening Tests in Patients with Bleeding Disorders

How well did you know this?
1
Not at all
2
3
4
5
Perfectly
83
Q

2) Patient with normal PTT and prolong PT with INR 1.5 which factor is
a. Factor VIII.
b. Factor VII.
c. Factor II.
d. DIC

A

b. Factor VII

Extrinsic pathway
- The ONLY Factor Deficiency that DOES NOT prolong aPTT = 7

Disorders of the Extrinsic Pathway of Coagulation
A prolonged prothrombin time and a normal aPTT suggest an isolated deficiency of factor VII, which is rare and may be the result of an inherited or an acquired abnormality. Less commonly, inhibitors to factor VII have been reported. Additionally, certain cases of DIC or dysfibrinogenemia may present with isolated prolonged PT values. Because factor VII is essential only in the tissue factor-activated extrinsic pathway of coagulation, the coagulation time and the Stypven time are normal in patients with this disorder.
- Lee: Wintrobe’s Clinical Hematology, 10th ed., Copyright © 1999 Lippincott Williams ; Wilkins

How well did you know this?
1
Not at all
2
3
4
5
Perfectly
84
Q

3) All of the following would increase the INR EXCEPT:
a) Factor II deficiency (affects both)
b) Factor V deficiency (affects both)
c) Factor VII deficiency
d) Factor X deficiency (PTT)
e) Factor XI deficiency (PTT)

A

Factor XI deficiency

a) Factor II deficiency (affects both - common pathway)
b) Factor V deficiency (affects both - common pathway)
c) Factor VII deficiency (only INR - extrinsic pathway)
d) Factor X deficiency (affects both - common pathway)
e) Factor XI deficiency (intrinsic pathway only)

How well did you know this?
1
Not at all
2
3
4
5
Perfectly
85
Q

1) All of the flowing are causes of hypercoaguability except:
a) Decreased activated protein C
b) Venous stasis
c) Immobility
d) Neoplasm
e) Increase functional protein S

A

1) Increase functional protein S

protein C + S are “anticoagulants”
Protein C and S deficiencies lead to a hypercoagulable state

How well did you know this?
1
Not at all
2
3
4
5
Perfectly
86
Q

2) Which of the following is responsible for a hypercoagulable state?
a. High protein S
b. Low protein C
c. Low factor VIII
d. Low fibrinogen

A

2) Low protein C

protein C + S are “anticoagulants”
Protein C and S deficiencies lead to a hypercoagulable state

How well did you know this?
1
Not at all
2
3
4
5
Perfectly
87
Q

Hypercoagulability that occurs with birth control pills is due to:

a. Increased antithrombin III
b. Decreased Protein S
c. Increase protein S
d. Factor II deficiency
e. Platelet dysfunction
f. Increased factor VII

A

B+F: Increased factor VII, decrease in protein S

a. Increased antithrombin III (bleeding)
b. Decreased Protein S (clot)
c. Increase protein S (bleeding)
d. Factor II deficiency (bleeding)
e. Platelet dysfunction (bleeding)
f. Increased factor VII (clot)

The mechanism by which oral contraceptives are prothrombotic is complex and uncertain. Prothrombotic effects include modest increases in the levels of procoagulant factors (factor VII, factor VIII, factor X, prothrombin, fibrinogen) and decreases in the levels of anticoagulant proteins (antithrombin III, protein S). The development of acquired APC resistance has been demonstrated, by means of a thrombin generation assay, in women taking oral contraceptives. Although the molecular basis for this phenomenon is unknown, it provides a plausible explanation for the greatly increased thrombotic risk among oral contraceptive users who are carriers of the factor V Leiden mutation.[4]
- Hoffman: Hematology: Basic Principles and Practice, 5th ed.

Oral contraceptives cause alterations of the vessel wall with intimal hyperplasia. They also increase blood viscosity. There are decreased levels of protein S, antithrombin activity, and plasminogen activator content in women taking oral contraceptives. There also may be an increase in the levels of fibrinogen, factor VII, and factor X.
- Bradley: Neurology in Clinical Practice, 5th ed.

The estrogen component of combination OCs increases hepatic production of serum clotting factors, particularly factors I (fibrinogen), VII, and X
- Noble: Textbook of Primary Care Medicine, 3rd ed., Mosby, Inc. p.332

How well did you know this?
1
Not at all
2
3
4
5
Perfectly
88
Q

Which factor deficit is seen with vitamin K deficiency?

a. II
b. V
c. IX
d. XI
e. XII

A

Not sure..

Factor VII has the shortest half-life of all the factors carboxylated by vitamin K; therefore, when deficient, it is the PT that rises first, since the activated Factor VII is the first to “disappear.” In later stages of deficiency, the other factors (which have longer half lives) are able to “catch up,” and the PTT becomes elevated as well.

old answer:
This isn’t mentioned in common texts. The following might suggest factor II is most sensitive marker ….

When there is mild vitamin K deficiency, the PT is normal, but there are elevated levels of the undercarboxylated forms of the proteins that are normally carboxylated in the presence of vitamin K. These undercarboxylated proteins are called proteins induced by vitamin K absence (PIVKA). Measurement of undercarboxylated factor II (PIVKA-II) can be used to detect mild vitamin K deficiency
- Kliegman: Nelson Textbook of Pediatrics, 18th ed.

How well did you know this?
1
Not at all
2
3
4
5
Perfectly
89
Q

FDP (Fibrin degradation product ~ D-dimer) is a result of:

a. Conversion of plasminogen to plasmin
b. Conversion of fibrinogen to fibrin
c. By product of RBCs degradation
d. All of the above
e. None of the above

A

Answer: Conversion of plasminogen to plasmin (well, almost)

Plasminogen activators convert plasminogen to plasmin. Plasmin then degrades fibrin into soluble fibrin degradation products.
- Hoffman: Hematology: Basic Principles and Practice, 5th ed.

How well did you know this?
1
Not at all
2
3
4
5
Perfectly
90
Q

In a DIC picture all of the following are true except:

a. Low D dimer
b. High D dimer
c. High platelets
d. Fibrinogen High
e. None of the above

A

Answer: A and D.
I think A makes most sense based on below explanation and the fact that A and B are contradictory (so one must be the right one)

Laboratory abnormalities in DIC are variable and related to the many diseases associated with this condition. Common abnormalities include an abnormal PT and aPTT with depressed fibrinogen levels and abnormal platelet counts. Levels of fibrin degradation products and d-dimer are elevated. The peripheral smear reveals fragmented RBCs, but this finding is not specific. Because of the continued activation of coagulation, thrombin/antithrombin complexes will be formed. Levels of thrombin/antithrombin and AT-III are depressed. Fragments of coagulation factor degradation are elevated, including F1.2 and FpA. Because of activation of the fibrinolytic system, plasminogen and α2-antiplasmin inhibitor levels are decreased.
- Townsend: Sabiston Textbook of Surgery, 18th ed.

Another feature of disseminated intravascular coagulation, hypofibrinogenemia, is unlikely to contribute to a defect in platelet aggregation, except in extreme cases, as fibrinogen concentration in plasma exceeds by at least 10-fold the quantity needed to saturate fibrinogen receptors on platelets.
- Hoffman: Hematology: Basic Principles and Practice, 5th ed.

Hmmm … based on the stem both A and D are possible.

How well did you know this?
1
Not at all
2
3
4
5
Perfectly
91
Q

What is the mechanism of action of Epsilon-aminocaproic acid (Amicar) in the treatment of DIC?

a) decreased fibrinolysis by inhibiting plasminogen activators
b) activation of complement cascade
c) activation of intrinsic pathway
d) induction of prostacyclin production
e) inhibitor of thrombin-anthrombin complex

A

Answer: decreased fibrinolysis by inhibiting plasminogen activators (similar to tranexamic acid)

Amicar is an antifibrinolytic that inhibits plasminogen to plasmin conversion thereby inhibiting fibrinolysis by plasmin

An antifibrinolytic for treatment of excessive bleeding from fibrinolysis. Competitively inhibits activation of plasminogen to plasmin, also, a lesser antiplasmin effect

How well did you know this?
1
Not at all
2
3
4
5
Perfectly
92
Q

What is the physiologic mechanism of thrombolytic therapy?

  • transformation of plasminogen into plasmin
  • transformation of fibrinogen into fibrin
  • inhibition of factor V
  • inhibition of factor VIII
  • inhibition of factor XIII
A

Logical answer: inhibiting factors would prevent further thrombosis without lysing current clots. fibrin causes clots. therefore plasminogen/plasmin is the answer

(Urokinase and Streptokinase increase Plasminogen → Plasmin formation) ⇒ Increase Plasmin activity ⇒ Increase Fibrinolysis

How well did you know this?
1
Not at all
2
3
4
5
Perfectly
93
Q

What is the best measure of the extrinsic pathway of hemostasis?

PT/INR
aPTT
Thrombin time
Bleeding time

A

PT/INR

How well did you know this?
1
Not at all
2
3
4
5
Perfectly
94
Q

which of following is reduced in obstructive jaundice:

a. Prothrombin
b. Fibrinogen
c. Factor XIII
d. Factor VIII

A

Answers:
- a. Prothrombin (factor II)

The uptake of the vitamin is intimately linked to the liver, as biliary salts are required for intestinal absorption.[10] Vitamin K deficiency can therefore be caused by anything that impairs the metabolism of bile acids. This includes intra- or extrahepatic cholestasis, biliary system fistulae or obstruction, primary biliary cirrhosis, or treatment with bile acid binders (i.e., cholestyramine).
Vitamin K serves as a coenzyme in the post-translational carboxylation of factors II, VII, IX, and X.[18] This modification creates sites on these proteins for calcium ion coordination and thereby renders them functional

How well did you know this?
1
Not at all
2
3
4
5
Perfectly
95
Q

Cause of increased bleeding in Obstructive jaundice is :

a. Decreased fibrinogen
b. Decreased absorbtion of Vit K
c. Decreased platelet function

A

b. Decreased absorbtion of Vit K

The uptake of the vitamin is intimately linked to the liver, as biliary salts are required for intestinal absorption.[10] Vitamin K deficiency can therefore be caused by anything that impairs the metabolism of bile acids. This includes intra- or extrahepatic cholestasis, biliary system fistulae or obstruction, primary biliary cirrhosis, or treatment with bile acid binders (i.e., cholestyramine).
Vitamin K serves as a coenzyme in the post-translational carboxylation of factors II, VII, IX, and X.[18] This modification creates sites on these proteins for calcium ion coordination and thereby renders them functional

How well did you know this?
1
Not at all
2
3
4
5
Perfectly
96
Q

What will activate the coagulation cascade during acute inflammation:

a. Factor XIII
b. VII
c. X
d. Prekallikrein

A

Answer: b. VII (extrinsic pathway, trauma)

Activator of extrinsic Pathway:
-Tissue factor comes into contact with factor 7

Activator of Intrinsic Pathway:
-Factor 12 binding to negatively charged surfaces (ex proteins)

Tissue kallikrein can also form bradykinin. Plasma kallikrein and bradykinin are activated during the acute and chronic phases of several experimental rat models of small intestinal ulceration and granulomatous enterocolitis, with a pathogenic role indicated by suppression of inflammation by a specific kallikrein inhibitor. [243] [244] Of considerable interest, genetic susceptibility to intestinal and systemic inflammation in inbred rat strains correlated with activation of the contact system[245] and is mediated by a single nucleotide polymorphism in kininogen that encodes a defect in glycosylation that accelerates cleavage by kininogen.
- Feldman: Sleisenger amp; Fordtran’s Gastrointestinal and Liver Disease, 8th ed.

Inflammation and blood clotting are often intertwined, with each promoting the other.[74] The clotting system is divided into two pathways that converge, culminating in the activation of thrombin and the formation of fibrin ( Fig. 2-15 ) ( Chapter 4). The intrinsic clotting pathway is a series of plasma proteins that can be activated by Hageman factor (factor XII), a protein synthesized by the liver that circulates in an inactive form. ***Factor XII is activated upon contact with negatively charged surfaces, for instance when vascular permeability increases and plasma proteins leak into the extravascular space and come into contact with collagen, or when it comes into contact with basement membranes exposed as a result of endothelial damage. Factor XII then undergoes a conformational change (becoming factor XIIa), exposing an active serine center that can subsequently cleave protein substrates and activate a variety of mediator systems (see later). Inflammation increases the production of several coagulation factors, makes the endothelial surface pro-thrombogenic, and inhibits anticoagulation mechanisms, thus promoting clotting.
- Kumar: Robbins and Cotran Pathologic Basis of Disease, Professional Edition , 8th ed.

How well did you know this?
1
Not at all
2
3
4
5
Perfectly
97
Q

What is the most common inherited factor deficiency:

a. Factor V deficiency
b. Factor VII
c. Factor VIII
d. Factor IX

A

Answer: c. Factor VIII

vWF is actually more common > VIII > IX > XI

a. Factor V deficiency (1 in million)
b. Factor VII (1/500 000)
c. Factor VIII (1/5000, X linked)
d. Factor IX (1/30 000)

vWF deficiency 1/100 (asymptomatic) - 1/10000 (symptomatic)

How well did you know this?
1
Not at all
2
3
4
5
Perfectly
98
Q

Which does not increase INR

a) Coumadin
b) heparin
c) decreased factor VII

A

Answer: b) heparin

In addition to the administration of warfarin, there are other causes of prothrombin time prolongation. These include (show table 1):

  • Vitamin K deficiency due, for example, to poor nutrition, or prolonged use of broad spectrum antibiotics.
  • Severe liver disease, which decreases the synthesis of both vitamin K-dependent and -independent clotting factors.
  • Deficiency or inhibition of factors VII, X, II (prothrombin), V, or fibrinogen
  • The infrequent antiphospholipid antibodies (lupus anticoagulant phenomenon) with antiprothrombin activity. In such patients, the acquired prothrombin deficiency may be severe enough to cause clinical bleeding. (
  • While treatment with heparin does not normally prolong the PT (due to the addition of heparin-neutralizing materials to the PT reagent), the PT may be transiently elevated after heparin bolus administration.

Increase INR (Extrinsic pathway)

  • Warfarin, Vitamin K deficiency
  • Factors 10, 9, 7, 2 Deficient (7 MOST IMPORTANT)
  • Liver Disease, DIC
How well did you know this?
1
Not at all
2
3
4
5
Perfectly
99
Q

A girl intentionally slashes her wrist. Despite applying pressure, the bleeding continues. Why is this?

a) retained foreign body
b) complete laceration of radial artery
c) partial transection of a major vessel

A

Likely B

How well did you know this?
1
Not at all
2
3
4
5
Perfectly
100
Q

Most common protein spilled in trauma?

a) glutamine
b) valine
c) alanine

A

a. Glutamine

The pattern of changes in amino acid concentration in muscle during catabolism shows an increase in branched-chain amino acids, aromatic amino acids, and methionine, and a decrease in glutamine and basic amino acids (lysine and arginine). A uniform reduction of approximately 50% of free muscular glutamine associated with negative N balance seems to be one of the most typical features of the response to trauma and infection

Glutamine is the most abundant amino acid in the human body, comprising nearly two thirds of the free intracellular amino acid pool. Of this, 75% is found within the skeletal muscles. During stress states such as sepsis, or
in tumor-bearing hosts, peripheral glutamine stores are rapidly depleted, and the amino acid is preferentially shunted as a fuel source toward the visceral organs and tumors, respectively. (Schwartz)

How well did you know this?
1
Not at all
2
3
4
5
Perfectly
101
Q

Options of treatment of rhabdomyolysis:

a. fluid resuscitation until u/o > 0.5 cc/kg/hr
b. mannitol
c. bicarbonate

A

Answer: fluid resuscitation until u/o > 0.5 cc/kg/hr

Rhabdomyolysis is the breakdown of muscle fibers with leakage of potentially toxic cellular contents into the systemic circulation. The final common pathway of rhabdomyolysis may be a disturbance in myocyte calcium homeostasis.[

Vigorous hydration with isotonic crystalloid is the cornerstone of therapy for rhabdomyolysis. Administer isotonic crystalloid 500 mL/h and titrate to maintain a urine output of 200-300 mL/h
Urinary alkalinization is recommended for patients with rhabdomyolysis and CK levels in excess of 6000 IU/L. Alkalinization should be considered earlier in patients with acidemia, dehydration, or underlying renal disease. A suggested regimen is 0.5 isotonic sodium chloride solution with one ampule of sodium bicarbonate administered at 100 mL/h and titrated to a urine pH higher than 7.
After establishing an adequate intravascular volume, mannitol may be administered to enhance renal perfusion.
Loop diuretics may be used to enhance urinary output in oliguric patients, despite adequate intravascular volume.
- http://emedicine.medscape.com/article/827738-treatment

Clinical sequelae of rhabdomyolysis include the following:
• Hypovolemia (sequestration of plasma water within injured myocytes)
• Hyperkalemia (release of cellular potassium into the systemic circulation)
• Metabolic acidosis (release of cellular phosphate and sulfate)
• Acute renal failure (nephrotoxic effects of liberated myocyte components)
• Disseminated intravascular coagulation (DIC)
• Hypocalemia (from phospate binding to calcium)

How well did you know this?
1
Not at all
2
3
4
5
Perfectly
102
Q

In a high pressure solvent injection, prognosis is best indicated by?

A

Answer: delay to OR

Synonyms: “high-pressure injection injury,” “grease gun injury,” “paint gun injury,” “pressure gun injury,” and “high-pressure injection.”

Worsens with:
• Greater time from injury to decompression/debridement
o>10 hours – very high rate of amputation
• Chemical/toxic nature of injectant
o Amputation rate 60-80% paint/solvents; 7000 -100% rate of amputation
• Velocity of injectant
• Secondary infection
oMay develop oleogranulomas (grease and oil based compounds) – drain through skin (fistulas/sinus)
• Portals of entry
o more distal worse
o digits: into tendon sheath poor
o palm: better because not governed by fascial planes
• MUST warn patients even if digit survives – will be left with painful, stiff, atrophic digit. Prepare for poor outcome and possible later ankylosis/amputation

How well did you know this?
1
Not at all
2
3
4
5
Perfectly
103
Q

A young man presents to the emergency department with a nearly complete circumferential laceration to the right arm. The wound itself is extremely dirty. After ample irrigation you should:

a) perform a primary closure in the ER
b) perform a primary closure in the OR
c) Perform a mechanical debridement
d) Harvest a skin graft to cover the defect

A

Answer: Perform a mechanical debridement

How well did you know this?
1
Not at all
2
3
4
5
Perfectly
104
Q

All are life threatening condition except:

a. Open femur fracture – embolis and infection
b. B/L femur fracture – embolis
c. Traumatic arm amputation – bleed out
d. Thigh compartment syndrome

A

Answer: Thigh compartment syndrome

a. Open femur fracture – embolis and infection
b. B/L femur fracture – embolis
c. Traumatic arm amputation – bleed out
d. Thigh compartment syndrome

How well did you know this?
1
Not at all
2
3
4
5
Perfectly
105
Q

What is the most common cause of death in children:

a. Congenital heart disease
b. Leukemia
c. Trauma
d. Infection

A

Answer: Trauma

#1 Unintentional Injury Age 1-4 11.1/100000, 5-14 6.8/100000 (of this MVA>drowning (1-4) and firearms (5-14))
#2 Congential Malformations 3.6/100,000(1-4) amp; Malignant Neoplams 2.4/100,000(5-14)
(source National Center for Health Statistics 2001)
How well did you know this?
1
Not at all
2
3
4
5
Perfectly
106
Q

5 year old boy involved in MVA. They showed you a normal CXR and AXR. His SaO2 is 84% difficult bagging. What is you next step:

a. Lt chest tube
b. Rt chest tube
c. Intubate and ventilate
d. Pericardiocenthesis

A

Answer: Intubate and ventilate

How well did you know this?
1
Not at all
2
3
4
5
Perfectly
107
Q

35yo male trauma to lower extremity 2h ago. Which most likely determines limb salvage vs amputation?

a. Post tib. Nerve injury
b. Vascular injury
c. Segmental bone loss
d. Contamination

A

a. Post tib. Nerve injury

b. Vascular injury - fix the vascular injury
c. Segmental bone loss - bone graft
d. Contamination - debride

Posterior tibial nerve doesn’t exist. I think they mean tibial nerve. If you don’t have tibial nerve you can’t feel the sole of your foot, and it’s a contraindication to salvage

“two absolute contraindications for lower limb salvage: anatomical complete disruption of the posterior tibial nerve in adults and crush injuries with warm ischemia time of more than 6 hours” - again, posterior tibial nerve isn’t technically correct but it’s cited in this article http://www.hindawi.com/journals/tswj/2014/640430/

How well did you know this?
1
Not at all
2
3
4
5
Perfectly
108
Q

Patient involved in MVC. “Damage control” surgery appropriate in all except the following:

a) Uncontrolled arterial bleeding
b) Unstable, hypotensive patient
c) Patient temp 34 degrees (a core body temp less than 32 during laporotomy is considered unitversally fatal)
d) Patient lactate >5 (indication of anerobic metabolism and possible loss of blood)

A

c) Patient temp 34 degrees (a core body temp less than 32 during laporotomy is considered universally fatal)

There is convincing evidence from multiple clinical series that a damage control strategy can be lifesaving in patients with severe injury. With such a strategy the surgeon limits interventions during the first operation to those that stop the bleeding and enable the patient to be resuscitated from hemorrhagic shock. In damage control, the surgeon selects a point in the first operation when the basic life-threatening problems have been managed and then decides to stop operating and return the patient to the ICU for resuscitation and correction of body temperature, acidemia, and coagulation disorders.
- Townsend: Sabiston Textbook of Surgery, 18th ed.

Indications for damage control surgery
Thoracic trauma
• Penetrating injury and SBP 5 (indication of anerobic metabolism and possible loss of blood)

How well did you know this?
1
Not at all
2
3
4
5
Perfectly
109
Q

Trauma patient scenario. When is the most appropriate time to order X Rays?

a. as soon as the patient arrives in the emergency department
b. before the primary survey
c. after the primary survey
d. after C spine is controlled and there is intravenous access
e. after the secondary survey is finished

A

c. After the primary survey.

Based on ATLS, the phases of emergency department care are divided into the primary survey with interventions, initial diagnostics and imaging, the secondary survey, and disposition.
- Marx: Rosen’s Emergency Medicine, 7th ed.

Radiographs of the chest, pelvis, C-spine, and FAST are adjuncts to the primary survey.
Imaging is considered helpful but should be used judiciously and should not interrupt or delay the resuscitation process. When appropriate, radiography may be postponed until the secondary survey.
- Kool, Blickman. Advanced Trauma Life Support®. ABCDE from a radiological point of view. Emerg Radiol. 2007 July; 14(3): 135–141.

Order from ATLS :
Primary survery
Resuscitation 
Adjuncts to primary surgery and resuscitation 
Consider need for patient transfer 
Secondary survery 
Continued post-resuscitation monitoring and re-evaluation 
Definitive care
How well did you know this?
1
Not at all
2
3
4
5
Perfectly
110
Q

Guy suffers anterior shoulder dislocation and complains of numbness over upper lateral shoulder, muscle most likely to be affected

  • biceps
  • triceps
  • deltoid
A

Answer: deltoid

Anterior Shoulder Dislocations
A careful assessment of the neurovascular status of the affected extremity is essential. Injury to the axillary artery is rare, usually occurring in the elderly,[13] and can be quickly assessed by palpation of the radial pulse or the presence of an expanding hematoma. It is important to assess the status of the axillary nerve, because this is the most common nerve lesion resulting from anterior dislocations.[14] The sensory component of the axillary nerve is assessed by testing for sensation over the lateral aspect of the upper arm ( Fig. 49–5 ). The motor component of the axillary nerve would be tested by assessing the strength of the deltoid muscle, a difficult undertaking in the patient with a dislocated shoulder.
- Roberts: Clinical Procedures in Emergency Medicine, 5th ed.

How well did you know this?
1
Not at all
2
3
4
5
Perfectly
111
Q

Girl falls off horse has a seizure –stable in emerg, negative ct scan. What is management?

a) Observe
b) load with dilantin,
c) MRI

A

Answer dilantin.

How well did you know this?
1
Not at all
2
3
4
5
Perfectly
112
Q

What is the best indicator of likelihood of focal seizure post-head trauma?
a) GCS

A

ATLS book
15% of patients with severe brain injury will have seizure
Severe definted as GCS 3-8

Approximately 5-10% of individuals with traumatic brain injury (TBI) experience new- onset seizures. The risk of seizure increases with increasing injury severity, depressed skull fracture, intracranial hematoma, and penetrating trauma. The risk is greatest in the first two years after injury and gradually declines thereafter. All types of seizures may occur as a result of trauma, but the most frequent are focal or partial complex seizures. Generalized complex seizures (what are commonly called “grand mal’ seizures) occur in approximately 33% of cases.
Immediate onset seizures, those that occur immediately or in the first few hours after a brain injury, do not suggest a chronic seizure disorder. Early onset seizures and those which develop within the first 7-8 days after trauma require prophylaxis for up to one year. Spontaneous resolution of seizure activity has been noted in this group. The highest risk group for persisting seizures are those individuals who experience seizure 7-8 days or longer following injury. Current recommendations for seizure prophylaxis suggest that patients who have not suffered a seizure within the first 7-8 days following a closed head injury, probably do not require prophylaxis

Posttraumatic seizures (PTS) frequently occur after moderate or severe TBI. Seizures are usually general or partial, and absence seizures are uncommon. Seizures are classified according to the time elapsed after the initial injury: Immediate seizures occur in the first 24 hours. Early seizures occur in the first 2-7 days, and late seizures occur after 7 days.
The incidence of late PTS is in the range of 5-18.9%. Risk factors include chronic alcoholism, older age at the time of injury, and a history of seizure disorder. Approximately one half to two thirds of patients with these risk factors develop late PTS within the first year after injury.16 If a patient with TBI has 1 PTS, his or her likelihood of having another is approximately 50%.
 - http://emedicine.medscape.com/article/326643-overview
How well did you know this?
1
Not at all
2
3
4
5
Perfectly
113
Q
A 6 week old baby in a trauma to stabilize head on a spine board, the head must be:
 a-parallel to board
 b-20 degrees flexed
 c-20 degrees extended
 d-some other position?
A

Answer: 20 degrees extension

Examination of a child with a suspected spinal injury is carried out with the patient supine, in a neutral position, after the head and neck have been stabilized. In small children, it is appropriate to place a pad beneath the trunk to avoid hyperflexion of the neck because of the disproportion in head size.
- Green amp; Swiontkowski: Skeletal Trauma in Children, 4th ed.

How well did you know this?
1
Not at all
2
3
4
5
Perfectly
114
Q
A young male is involved in an MVA. He sustains a closed head injury and on presentation has a decreased GCS, BP 90/50, HR = 105, RR = 20 and an increased ICP at 22mmHg. All of the following are acceptable courses of action EXCEPT (2 answers):
A. Propofol and intubation
B. Lasix, 40mg IV
C. Mannitol, 20mg/kg IV
D. IV morphine for pain control
E. Raise bed to 30 degree incline, head up
F. Hyperventilate to PCO2 of 34-36
G. Dexamethasone
A

B. Lasix, 40mg IV and G. Dexamethasone

Glucocorticoids — Glucocorticoids were associated with a worse outcome in a large randomized clinical trial (CRASH) of their use in moderate to severe head injury. They should not be used in this setting.

Other agents — Furosemide, 0.5 to 1.0 mg/kg intravenously, may be given with mannitol to potentiate its effect. However, this effect can also exacerbate dehydration and hypokalemia (this patient is hypotensive)

Management of Elevated Intracranial Pressure
- Prevention of venous engorgement
- CO2 control (mild hyperventilation) NORMOCAPNEA and NORMOXIA
- Sedation and pain control
- Cerebrospinal fluid drainage
- Mannitol
- Lasix
- Hypertonic saline
- Decompressive craniectomy
- Pentobarbital coma - used as last-ditch effort
Bope: Conn’s Current Therapy 2010, 1st ed

ICP is normally 0–10 mm Hg; at 20–25 mm Hg, the upper limit of normal, treatment to reduce ICP is needed.

Propofol decreases intracranial pressure (ICP) in patients with either normal or increased ICP.

How well did you know this?
1
Not at all
2
3
4
5
Perfectly
115
Q

Patient post MVC. MRI showed diffuse brain edema. Patient is agitated in the ICU. What would be the best kind of medication for sedation for 1-2days:

a. Midazolam
b. Lorazepam
c. Propofol
d. Morphine

A

Propofol does not increase ICP and can be run as an infusion.

Sedation — Keeping patients appropriately sedated can decrease ICP by reducing metabolic demand, ventilator asynchrony, venous congestion, and the sympathetic responses of hypertension and tachycardia [72]. Establishing a secure airway and close attention to blood pressure allow the clinician to identify and treat apnea and hypotension quickly.

Propofol has been utilized to good effect in this setting, as it is easily titrated and has a short half-life, thus permitting frequent neurologic reassessment.

How well did you know this?
1
Not at all
2
3
4
5
Perfectly
116
Q

When are flexion/extension views contraindicated?
A. Midline cervical tenderness
B. Diagnosed Brown-Sequard syndrome
C. Patient can flex and extend neck without assistance
D. When a ligamentous injury is suspected
E. Spinal shock
F. Anterior cord syndrome

A

Answer: Brown-Sequard

  • the flexed view is usually most helpful in detecting ligamentous injury that is not apparent on the neutral view
  • typically, this view is ordered at 7 to 10 days post injury when C-spine is less tender

Contraindications :

  • known cervical spine fracture or dislocation
  • Brown sequard, central cord
  • not with anterior cord syndrome

In alert patients with normal plain radiographs but persistent symptoms, flexion-extension views, especially of the cervical spine, can be obtained to look for ligamentous instability.
- Townsend: Sabiston Textbook of Surgery, 18th ed.

Central cord syndrome is a contraindication to flex-ex.

Flex-ext stress views should obtained for only those who are alert, cooperative, and neurologically intact and patients who can describe pain or early onset of any subjective neurologic symptoms

Brown-sequard: hemisection of cord. Asymmetric paresis and jt position/vibration loss (ipsilateral) with contralateral pain and temperature loss.
Ant. Cord syndrome: disproportionately greater motor impairment in upper compared to lower extremities, bladder dysfunction, and variable degree of sensory loss below the level of injury. Typically after hyperextension injury. Flex-Ext views indicated

How well did you know this?
1
Not at all
2
3
4
5
Perfectly
117
Q

Neuro injury in multisystem trauma due to

a. vasopressors
b. toxins
c. perfusion

A

Answer: perfusion

In addition, polytrauma patients can present with hypotension caused by ongoing hemorrhage, resulting in a critically low perfusion to a potentially swollen, injured spinal cord. As with brain injury, prolonged hypoperfusion and hypoxia may be harbingers of poor outcome.
- Bradley: Neurology in Clinical Practice, 5th ed.

How well did you know this?
1
Not at all
2
3
4
5
Perfectly
118
Q

A young healthy male involved in a MVA suffers a closed head injury. He remains hypotensive despite 2 L of crystalloid. The likely cause is:

a. SIADH
b. intracranial bleed
c. unrecognized intra-abdominal injury

A

Answer: unrecognized intra-abdominal injury

SIADH would make you hypertensive

How well did you know this?
1
Not at all
2
3
4
5
Perfectly
119
Q

Which of the following is the worst prognostic indicator in head injury?

a. cerebral hypoxia
b. systemic hypotension
c. age
d. co-morbid illness

Which is the best prognostic factor after TBI
 a – initial GCS
 b – Lack of a sub dural
 c – Age
 d – Moving all four limbs
A

Not sure what the answers are… probably A for the second question

PROGNOSIS — Outcome from severe head injury is dependent on a range of factors including baseline patient characteristics, severity of TBI, and the occurrence of medical complications and secondary brain insults. Specific negative outcome predictors that have been identified from these factors include :
●GCS at presentation (especially the GCS motor score)
●Presence and severity of CT abnormalities (subarachnoid hemorrhage, cisternal effacement, midline shift)
●Pupillary function
●Age
●Associated injuries and complications
●Hypotension
●Hypoxemia
●Pyrexia
●Elevated ICP
●Reduced CPP
●Bleeding diathesis (low platelet count, abnormal coagulation parameters)

How well did you know this?
1
Not at all
2
3
4
5
Perfectly
120
Q

Snowmobile accident, severe head injury and hypotensive. Why?

a. Hemmorhage
b. Spinal cord injury
c. Medulla injury

A

Hypotension is always hemhorrage until proven otherwise ?

How well did you know this?
1
Not at all
2
3
4
5
Perfectly
121
Q

The most common cranial nerve injured in trauma:

a. I
b. II
c. III
d. IV
e. V

A

CN 1

Posttraumatic single nerve palsy was observed in 38 patients (77.6%), and multiple nerve injuries were observed in 11 (22.4%). Cranial nerves were affected in 62 cases. The most affected CN was the olfactory nerve (CN I), followed by the facial nerve (CN VII) and the oculomotor nerves (CNs III, IV, and VI).

Other cranial nerve injuries — The risk of injury to other cranial nerves increases with the severity of brain injury, but these can also occur in mild TBI with an incidence of 0.3 percent according to one case series [143]. The distribution is similar to that for moderate or severe head injuries:

●Anosmia and hyposmia, which are often reported by the patient as impaired taste as well as smell, occur following injury to olfactory filaments as they enter the brain through the cribiform plate. While recovery occurs in about one-third of patients, loss of smell is likely permanent if present one year after the injury [144]. (See “Anatomy and etiology of taste and smell disorders”.)

●Diplopia may result from injury to cranial nerves III, IV, and VI. In the setting of mild TBI, injury to cranial nerve IV is most common, followed by VI. Cranial nerve III is less commonly affected by mild TBI. (See “Overview of diplopia”.)

●Facial pain and occipital neuralgia may occur in association with mild TBI. The former usually occurs in the setting of blunt force injury to the trigeminal nerve in the face, while occipital nerve injury may be indirect from a contiguous musculoskeletal injury in the neck. (See “Overview of craniofacial pain”.)

How well did you know this?
1
Not at all
2
3
4
5
Perfectly
122
Q

A patient is brought to the emergency department following an injury. On arrival, his eyes are closed, he is mumbling incoherently, and he localizes to pain. What is his GCS?

a. 6
b. 7
c. 8
d. 9
e. 10

A

d. 9

E1V3M5 = 9

his eyes are closed (do not open to command) = 1, he is mumbling incoherently = 3, and he localizes to pain = 5

Defintion of incoherent: without logical or meaningful connection; disjointed; rambling

How well did you know this?
1
Not at all
2
3
4
5
Perfectly
123
Q

Patient was involved in MVA. Presented with decreased LOC and ecchymosis around eyes and behind ears. What is the most common cause:

a. LeFort II
b. Basal skull fracture
c. Intracerebral bleeding
d. ????

A

Answer: Basal skull fracture

Raccoon eyes, battle’s sign

How well did you know this?
1
Not at all
2
3
4
5
Perfectly
124
Q

Blurb of basal skull #, and otorrhea. What is the best option?

a. Treat with broadspectrum Abx and observe
b. Swab ear and treat Abx specific
c. Consult neurosurg for OR
d. Consult ENT for OR
e. Observe and consult if drainage continues

A

e- Observe and consult if drainage continues

Realistically should get neurosurg assessment, but the key to answering this question is to know that 1. usually the CSF leaks stop on their own (so don’t need urgent OR), and 2. you don’t need to give prophylactic antibiotics. See below for more details.

Basal skull fractures involve the floor of the skull. Bruising may occur about the eye (raccoon sign) or behind the ear (Battle’s sign), suggesting a fracture involving either the anterior or middle fossa, respectively. Isolated cranial nerve deficits can be seen in association with this fracture type because the nerves’ exit foramina lie at the skull base. The facial nerve is frequently affected, with injury due to laceration or swelling. Most facial nerve deficits resolve spontaneously, so no specific therapy is warranted. On the other hand, complete transections of the facial nerve are usually explored, although the timing of exploration remains a matter of debate.

Any associated CSF rhinorrhea or otorrhea should be treated expectantly. Traumatic CSF leaks typically stop within the first 7 to 10 days. If a leak persists, lumbar CSF drainage can be implemented to seal the leak by lowering CSF volume and ICP. If drainage therapy fails, surgical exploration is indicated and accomplished by oversewing the defect with an intradural fascial patch graft. Fewer than 5 percent of patients require surgical repair. Prophylactic antibiotics are not used routinely because recent prospective studies have failed to demonstrate any benefit from their use.
- Schwartz

Uptodate
Basilar fractures — Intracranial hemorrhage caused by a basilar skull fracture frequently represents a surgical emergency and requires immediate neurosurgical consultation. All patients with basilar skull fractures require admission for observation, regardless of the need for surgical intervention.

Basilar skull fractures can produce a dural tear, which can cause a cerebral spinal fluid (CSF) leak. Clinically, this is suggested by the presence of clear or blood-tinged rhinorrhea or otorrhea in a head-injured patient. If the fluid is blood tinged, the “halo” sign (also called the “ring” or “target” sign) may be useful to determine the presence of CSF.

To perform the test, a drop of the fluid is placed on a tissue or filter paper. A rapidly expanding ring of clear fluid around red blood defines a positive test. Of note, the halo test does NOT differentiate among CSF, saline, saliva, and other clear fluids and has not been formally studied in a clinical setting. (See “Facial trauma in adults”, section on ‘Examination of specific body parts’.)

The majority of CSF leaks resolve spontaneously within one week of injury and without CNS complications. The risk of meningitis has been estimated at 3 percent in the first week. Retrospective observational studies suggest that the administration of prophylactic antibiotics during the first week of a CSF leak is unnecessary [46-49]. The incidence of bacterial meningitis rises substantially if the leak persists past seven days and we suggest that prophylactic antibiotics be given in such cases [46]. Antibiotic selection is identical to that for penetrating head trauma. (See ‘Penetrating injuries’ below.)

Cranial nerve palsies are a delayed complication of basilar skull fractures. They generally present two to three days following injury and are due to nerve compression or contusion [50,51]. These palsies may respond to treatment with glucocorticoids, although no large clinical trials have been performed to support this approach. Common treatment regimens are similar to those used for nontraumatic palsies (eg, Bell’s palsy). We suggest starting such treatment in consultation with otolaryngology or neurosurgery. (See “Bell’s palsy: Pathogenesis, clinical features, and diagnosis in adults”.)

Facial nerve palsies that appear acutely in association with basilar skull fractures are due to nerve transections. These injuries do NOT respond to glucocorticoid therapy and carry a poor prognosis for recovery of nerve function [45].

How well did you know this?
1
Not at all
2
3
4
5
Perfectly
125
Q

70 year old male patient was involved in MVA. Vitally stable with no complaint but could not recall what happened. What is the most appropriate

action:
a. Discharge home
b. CT head
c. Skull x-ray
d. Intubation

A

Answer: CT Head because he is >65 years old

Canadian CT Head Rule
CT Head Rule is only required for patients with minor head injuries
with any one of the following:

High risk (for neurological intervention)
● GCS score 30 min
● Dangerous mechanism (pedestrian struck by motor vehicle, occupant ejected from motor vehicle, fall from height >3 feet or five stairs)

Minor head injury is defined as witnessed loss of consciousness, definite amnesia, or witnessed disorientation in a patients with a GCS score of 13–15.

How well did you know this?
1
Not at all
2
3
4
5
Perfectly
126
Q

Man brought into emerg after falling at shopping mall. HR, BP, RR all normal, neuro exam normal. Well-healed pacemaker scar. Evidence of malar # (Zygomaticomaxillary complex fracture). Can’t remember events of fall. In addition to arranging for repair of #, you should do:

a) EKG; pacemaker interrogation
b) ECHO
c) CT head
d) Stress test

Similar question:

Elderly man brought into the ER after a fall. Past history includes a pacemaker. Although he could not recall the incident he is now mentally clear with nothing to find on exam except for a malar fracture. What is the most appropriate investigation?

a. CT scan with contrast
b. MRI
c. Stress EKG
d. Resting EKG and pacemaker check.

A

? I think further details on stem would help. Unsure if this is a syncope question or CT head rule question

For the second, CT is likely the answer. Hard to argue EKG vs. stress EKG so likely those two are not the answer. MRI is almost always contraindicated if you have a pacemaker

How well did you know this?
1
Not at all
2
3
4
5
Perfectly
127
Q

Which is correct regarding linear vs. depressed skull fracture:

a. Linear fracture is critical if occurred over a vessel
b. Linear fracture indicate more sever impaction
c. Depressed fracture is not associated with brain contusion

A

Answer: Linear fracture is critical if occurred over a vessel

The great majority of linear skull fractures have minimal or no clinical significance. However, fractures that cross the middle meningeal groove in the temporal bone or major venous dural sinuses may disturb these vascular structures causing significant extraaxial bleeding (ie, bleeding beneath the skull but outside the brain parenchyma). Separation (ie, diastasis) of skull sutures can occur following trauma and most often involves the coronal or lambdoid sutures.

Uptodate
Linear skull fracture — A linear skull fracture is a single fracture that most often extends through the entire thickness of the calvarium. They occur most often in the temporoparietal, frontal, and occipital regions.

128
Q

What is the effects of other systems injury in patients with head trauma:

  • You might miss the head injury
  • Decrease cerebral oxygenation by the effect of other injuries.
A

Answer: Decrease cerebral oxygenation by the effect of other injuries

129
Q

Which of the following is the most common electrolyte abnormality after head injury?

a) hyponatremia
b) hypernatremia
c) hypokalemia
d) hypomagnesemia

A

Answer: hyponatremia

SIADH is the most common abnormality after CHI, thus hyponatremia is correct. SIADH causes a dilutional hyponatremia secondary to inappropriate renal water conservation. Relatively less common post-TBI endocrinopathies include anterior hypopituitarism (AH), DI, cerebral salt wasting (CSW), and primary adrenal insufficiency (PAI)

130
Q

A trauma patient presents with a wide, fixed pupil and contralateral hemiplegia. This is most likely due to:

a. Subfalcine herniation
b. Bilateral frontal contusions
c. Transtentorial herniation

A

Answer: Transtentorial herniation

An evaluation of the patient’s pupil size and responsiveness must be done early in the initial assessment of the head-injured patient. A large fixed pupil suggests herniation syndrome; it is usually on the side of the expanding lesion. Traumatic mydriasis, resulting from direct injury to the eye and periorbital structures, may confuse the assessment of the pupillary responsiveness.
The most common clinically significant traumatic herniation syndrome is uncal herniation, a form of transtentorial herniation. Uncal herniation is often associated with traumatic extraaxial hematomas in the lateral middle fossa or the temporal lobe. The classic signs and symptoms are caused by compression of the ipsilateral uncus of the temporal lobe on the U-shaped edge of the tentorium cerebelli as the brain is forced through the tentorial hiatus. As compression of the uncus begins, the third cranial nerve is compressed; anisocoria, ptosis, impaired extraocular movements, and a sluggish pupillary light reflex develop on the side ipsilateral to the expanding mass lesion. This phase may last for minutes to hours, depending on how rapidly the expanding lesion is changing. As the herniation progresses, compression of the ipsilateral oculomotor nerve eventually causes ipsilateral pupillary dilation and nonreactivity.
Initially in the uncal herniation process, the motor examination can be normal, but contralateral Babinski’s responses develop early. Contralateral hemiparesis develops as the ipsilateral peduncle is compressed against the tentorium. With continued progression of the herniation, bilateral decerebrate posturing eventually occurs; decorticate posturing is not always seen with the uncal herniation syndrome. In up to 25% of patients the contralateral cerebral peduncle is forced against the opposite edge of the tentorial hiatus. Hemiparesis is then detected ipsilateral to the dilated pupil and the mass lesion. This is termed Kernohan’s notch syndrome and causes false-localizing motor findings.
- Marx: Rosen’s Emergency Medicine: Concepts and Clinical Practice, 5th Edition, pg 290

131
Q

Question about head injury with edema, what anticoagulation would you use? (none vs LMWH)

A

The prevention of deep venous thrombosis (DVT) is a difficult management issue in TBI. Patients with TBI are at increased risk of DVT which can be reduced by the use of mechanical thromboprophylaxis using intermittent pneumatic compression stockings [50,51]. While DVT risk can be further reduced with antithrombotic therapy, this has to be weighed against the potential risk of hemorrhage expansion, which is greatest in the first 24 to 48 hours [52-54]. The use and timing of antithrombotic agents in patients with TBI must therefore be individualized according to the degree of intracranial bleeding and the perceived risk of DVT. Observational studies suggest that antithrombotic therapy may not be associated with the increased risk of intracranial hemorrhage expansion previously thought to occur in this setting, even when administered early on [55,56].

But based on newest evidence, it appears that enoxaparin may be used in these patients without increased risk of bleeding.

132
Q

A 30 y old lady is involved in a motor vehicle accident and sustains multiple facial fractures. 5 days later, she presents to the ER with bilateral anosmia, fever, and delirium. What is the most likely diagnosis:

a) bacterial meningitis
b) infected subdural hematoma
c) superior sagittal sinus thrombosis
d) cavernous sinus thrombosis

A

Cavernous sinus thrombosis usually presents with visual symptoms as CN 3,4,6 and sensory 5 pass through there.

Superior sagiall sinus thrombosis can get focal findings with weird hemorrhades in weird places, typically involving the abduces nerve (it is the longest nerve)

Meningitis , cerebral edema and saggitial sinus thrombosis are less likely to be focal seizures in nature
This patient has a bilateral loss of sense of smell – olfactory nerve is CN I and the receptors are located in the roof and walls of the nasal cavity. CN I and II are derived from the forebrain
- also, the fever and leukocytosis are more suggestive of infection, therefore making a) the more likely answer

133
Q

Zone I is between cricoid and which?

a) clavicle
b) cricothyroid membrane
c) Hyoid
d) Angle of Mandible

A

Answer: Zone I is clavicle to cricoid

Zone I, the base of the neck, is demarcated by the thoracic inlet inferiorly and the cricoid cartilage superiorly. Structures at greatest risk in this zone are the great vessels (subclavian vessels, brachiocephalic veins, common carotid arteries, aortic arch, and jugular veins, trachea, esophagus, lung apices, cervical spine, spinal cord, and cervical nerve roots. Signs of a significant injury in the zone I region may be hidden from inspection of the chest or the mediastinum

Zone II encompasses the midportion of the neck and the region from the cricoid cartilage to the angle of the mandible. Important structures in this region include the carotid and vertebral arteries, jugular veins, pharynx, larynx, trachea, esophagus, and cervical spine and spinal cord. Zone II injuries are likely to be the most apparent on inspection and tend not to be occult. Additionally, most carotid artery injuries are associated with zone II injuries.

Zone III characterizes the superior aspect of the neck and is bounded by the angle of the mandible and the base of the skull. Diverse structures, such as the salivary and parotid glands, esophagus, trachea, vertebral bodies, carotid arteries, jugular veins, and major nerves (including cranial nerves IX-XII), traverse this zone. Injuries in zone III can prove difficult to access surgically.

134
Q

1) Investigations and management of injuries to the 3 neck zones
2) Which neck stab do you not explore

A

Answer:

1) Likely question/answer is q) for asymptomatic patients, which would you explore a) zone II
2) posterior (given)

When would you explore the neck?

  • Any Region with SHOCK or STROKE ⇒ Surgery
  • Region 2 with SYMPTOMS ⇒ Surgery
    • All others cases ⇒ Angiography, CXR, +/- Esophagus Scope, +/- Tracheal Scope

Asymptomatic patients with penetrating injuries to the base of the neck (zone 1) require four-vessel arch angiography either to exclude major arterial injury or to plan the operative approach if an injury is present. The same applies to penetrating injuries above the angle of the mandible (zone 3), where both exploration and distal control are technically difficult; therefore, an endovascular solution, if feasible, may be the safest option.

Patients with asymptomatic midcervical injuries (zone 2) may undergo either neck exploration (a straightforward procedure associated with very low morbidity) or a combination of four-vessel angiography, esophagoscopy, and barium swallow to rule out significant arterial and esophageal injury. Both pathways are acceptable alternatives, and thus choice between operative exploration and a sequence of diagnostic procedures reflects individual preferences or institutional policies.
- Townsend: Sabiston Textbook of Surgery, 18th ed.

Zone I Injuries
- At risk: great vessels; esophagus; trachea
- Symptomatic: angio, then explore
- Asymptomatic: needs angio, laryngoscopy and esophagoscopy
- Mandatory esophageal evaluation is also recommended because a missed esophageal injury in Zone I may be clinically silent until mediastinitis and sepsis develop
- Contrast esophagram may also be useful (sensitivity and specificity as high as 90% to 100%)
- Flexible or rigid esophagoscopy has high specificity and sensitivity
- CT can show path of projectile
Zone II Injuries
- At risk: everything
- Symptomatic: explore
- Asymptomatic: either
- Directed evaluation with serial exam
- Mandatory neck exploration
Zone III Injuries
- At risk: great vessels and cranial nerves at skull base
- Symptomatic: angio, then explore and/or interventional radiology
- Asymptomatic: angio +/- exploration
- Bailey: Head and Neck Surgery – Otolaryngology

135
Q

A trauma patient sustains multiple facial fractures and abdominal injuries. He is noted to have CSF ottorhea. What is the BEST management option?
A. Delayed repair of facial fractures
B. Immediate repair of CSF leak
C. Repair facial fractures at time of laparotomy
D. There is no need to repair the facial fractures

A

Answer: Delayed repair of facial fractures

136
Q
A patient sustains a transverse fracture through the pterygoid plate, inferior to floor of maxillary sinus. What type of fracture does this represent?	
A. LeFort I
B. Lefort II
C. Lefort III
D. Panfacial fracture
A

Answer: Lefort I

Seminar Plastic Surgery

  1. Le Fort I (Gueurin fracture or Horizontal fracture)
    - Fracture line passes through
    • Base of pyriform aperture (above roots of maxillary teeth)
    • Base of maxillary sinus and nasal septum
    • Pyramidal process of palatine bone ; pterygoid plates
      - Separates maxillary alveolus from superior midfacial skeleton
      - 30% of maxillary fractures
  2. Le Fort II (Pyramidal fracture)
    - Fracture line passes through
    • Across nasal bones, nasofrontal suture and septum
    • Into medial wall of orbit (lacrimal bone)
    • Diagonally downward and outward through maxilla (involves inferior orbital rim and orbital floor)
    • Through pterygoid plates
      - Central pyramidally shaped nasomaxillary segment separated from the zygomatic and upper, lateral midfacial skeleton
      - Bony fragment contains
    • Lacrimal crests → possible lacrimal duct injury
    • Bulk of maxilla (displaces backward and up) → anterior open bite
    • Pyriform margin (incorporates nasal skeleton including: nasal bones, bony septum, perpendicular plate of ethmoid, lower half of vomer and infra-orbital foramen) → transient or permanent infra-orbital nerve injury
    • Alveolus
    • Palate
      - Above or below medial canthal ligament (attached to frontal process of maxilla); low and high varieties
    • Telecanthus may result if disrupted
      - 42% of maxillary fractures
  3. Le Fort III (Craniofacial dysjunction or transverse fracture)
    - Cranial base completely separated from the mid-facial skeleton (free floating maxilla)
    - Fracture line passes through
    • Nasofrontal (NF) suture
    • Across orbital floor to ZF suture
    • Zygomatic arch and pterygoid plates
      - Usually will develop anterior open bite (as per II)
      - Anesthesia of maxillary teeth will usually resolve
      - 28% of maxillary fractures
  4. Le Fort IV
    - Not described by Le Fort
    - Fractures consisting of frontal bone, sinus, superior orbital rims and anterior cranial base
137
Q

Patient has penetrating carotid artery in coma for 2 hrs. what should you do to improve his neurological status:

a. Revascularization
b. Anticoagulation
c. Carotid artery ligation
d. Close monitoring

A

Answer: depends on neurological status

  • awake = repair
  • coma = close monitoring

If awake should repair or ligate. Ligate if uncontrolled hemorrage or no antegrade flow.

Penetrating carotid artery injuries, regardless of the patient’s neurologic status, usually require repair, except in comatose patients. Inaccessible carotid artery injuries near the base of the skull can be treated by interventional radiologists with a stent if the anatomy of the injury is favorable. Otherwise the artery will need to be thrombosed or ligated. If ligation is necessary, the patient should be given anticoagulation therapy with heparin followed by warfarin sodium (Coumadin) for 3 months. This treatment may prevent a stroke by inhibiting the generation of thrombi from the surface of the clot at the circle of Willis while the endothelium heals. Without anticoagulation therapy the risk of stroke with ligation has been approximately 20 to 30 percent, and most strokes occur a few days after ligation.
- PRINCIPLES OF SURGERY, SEVENTH EDITION

138
Q

A patient suffers blunt trauma to their thorax. The most likely cause for acute cardiorespiratory collapse is:

a. Pulmonary contusion
b. Tension pneumothorax
c. MI
d. Pulmonary embolism

A

Answer: Tension pneumothorax

139
Q

The best way to diagnose tension pneumothorax:

a) CXR
b) needle into 2nd ICS mid-clavicular line
c) clinically

A

Answer: clinically

Tension pneumothorax is a clinical diagnosis and treatment should not be delayed by radiologic confirmation

Tension pneumothorax is characterized by:
o	chest pain
o	respiratory distress
o	tachycardia
o	hypotension
o	tracheal deviation
o	unilateral absence of breath sounds
o	neck vein distention
o	cyanosis (late)
Reference: ATLS manual
140
Q

A 24 year old man is an unrestrained driver in a motor vehicle crash. The patient remains hypoxic after administration of oxygen. He has bilateral chest tenderness. A chest X-ray shows bilateral infiltrates. Which of the following is the most likely diagnosis?

a. Bilateral pneumonia.
b. Acute Respiratory Distress Syndrome (ARDS)
c. Aspiration of gastric contents.
d. Atelectasis.
e. Pulmonary contusion.

A

Answer: Pulmonary contusion

141
Q

Trauma patient, during pericardiocentesis you inserted the needle in the subxiphoid space and aspirated some fluid which is not clotting on the green towel. Which of the following should you do next:

a. Remove the needle and do sternotomy.
b. Thoracotomy and pericardial window.
c. Insert the needle deeper
d. Insert chest tube.

A

Answer: Thoracotomy and pericardial window.

Fluid that is not clotting suggest pericardial fluid.
Treatment of traumatic pericardial temponade is immediate drainage.

emedicine
Pericardial window involves the excision of a portion of the pericardium, which allows the effusion to drain continuously into the peritoneum or chest.[2] The fluid can be drained in any of 3 ways: via a small subxiphoid incision, thoracoscopically,[6] or via a thoracotomy

indications for a pericardial window:
-Symptomatic pericardial effusions
-Asymptomatic pericardial effusions that warrant a pericardial window for diagnosis
-Hemodynamically stable patients with an undiagnosed pericardial effusion (a thoracoscopic approach is ideal)
-Coexisting pericardial, pleural, or pulmonary pathology that requires diagnosis or therapy (a thoracoscopic approach is ideal)
-Known benign effusions that reaccumulate after aspiration
-Drainage of a purulent pericardial effusion
-Early fungal or tuberculous pericarditis in which resection of the pericardium is required to prevent future pericardial constriction
etc

142
Q
A patient is involved in a deceleration MVC and sustains chest trauma, presenting tachycardic and hypotensive. CXR shows a widened mediastinum.  What is the MOST appropriate course of action?
A. Insert left sided chest tube
B. Obtain an aortogram
C. Obtain a CT scan with contrast
D. Perform cardiac catheterization
A

Answer: Obtain an aortogram

Aortic dissection
2016: CT-angiogram is more practical

143
Q

A patient arrives in the ER following a significant trauma. On exam, he is unconscious, his right pupil is fixed and dilated and he is tachypneic with a RR of 45. His trachea is deviated to the right. His blood pressure is 80/30 and his HR is 110 bpm. What is the most appropriate immediate action:

a) CXR
b) Intubation with propofol for induction
c) IV access
d) Chest tube
e) Immediate craniotomy

A

Answers:

  • Needle decompression in 2ed I.C.S
  • Intubation

ABCDE

Emergent needle decompression for tension pneumothorax is carried out on the affected side by placing an 18-gauge needle or angiocatheter into the hemithorax at the midclavicular line in the second anterior intercostal space. This emergency maneuver relieves the tension created within the thorax. It does not treat the pneumothorax; subsequent chest tube insertion is required. [no mention of intubation]
- Townsend: Sabiston Textbook of Surgery, 18th ed.

When the diagnosis of tension pneumothorax is suspected clinically, the pressure should be relieved immediately with needle thoracostomy, which is performed by inserting a large-bore (14-gauge or larger) catheter, at least 5 cm in length, through the second or third interspace anteriorly or the fourth or fifth interspace laterally on the involved side. This method can be easily performed in the field or ED, allowing vital signs to improve during transport or preparation for a tube thoracostomy. [no mention of intubation]
- Marx: Rosen’s Emergency Medicine, 7th ed.

144
Q

Patient suffers massive blunt chest trauma, most inkeeping with complete tracheobronchial disruption

a) subq emphysema
b) mediastinal emphysema
c) massive air leak and failed re-exapansion lung after chest tube

A

c?

a) subq emphysema (most common physican finding; high sensitive)
b) mediastinal emphysema
c) massive air leak and failed re-exapansion lung after chest tube
(highly suggestive of tracheobronchial injury; high specificity)

Diagnosis is difficult and often delayed [86]. Intrathoracic injury can be subtle and indolent, presenting with retained secretions, recurrent pneumothoraces, and obstruction. The sine qua non (“[a condition] without which it could not be”, or “but for…” or “without which [there is] nothing”.) of intrathoracic tracheobronchial injury is a significant air leak and pneumothorax or pneumomediastinum that reaccumulates despite tube thoracostomy. A cervical injury may present without a significant air leak, if the tear or rupture is contained by the adventitia. Signs of cervical tracheal injury include dyspnea, hoarseness, and subcutaneous emphysema.

145
Q

Guy gets stabbed in left 6th intercostals space, midaxillary line. No breath sounds on left and tender abdomen, most appropriate next step

a) thoracoabdominal incision
b) CT chest and abdo
c) Thoracotomy and explore diaphragm from chest
d) Tube thorocotomy (chest tube) and laparotomy

A

Answer: Tube thorocotomy and laparotomy

Airway, breathing and then circulation. With decreased breath sounds, this is a clinical diagnosis of a pneumothorax, although its only tension if he is hypotensive or hypoxic. Therefore he needs a chest tube(tube thoracostomy) and since he has a tender abdomen, he likely has a perforated bowel which necessitates a laparotomy.

146
Q

A 27 year old construction worker is crushed under falling building materials. Upon presentation to the ER 30 minutes later, he is dyspneic, had decreased air entry on his left and has signs of peritoneal irritation. An X-Ray reveals an indistinct density in the left lower lung field obscuring his diaphagm, as well as multiple rib fractures on the left. The least likely diagnosis is:

a) pneumothorax
b) hemothorax
c) ruptured diaphragm
d) pleural effusion
e) splenic rupture

A

Answers:
- pleural effusion (?)

Patients with diaphragmatic injury due to blunt trauma usually have associated injuried that demand first attention and prevent detailed initial evaluation. The first chest X-ray after rupture of either hemidiaphragm may show nothing more than a blurring of the diaphragm with or without evidence of a small hemothorax.
- Schwartz

Rupture of the diaphragm may occur after blunt trauma. The diaphragm most frequently is ruptured on the left side, perhaps because the liver may dissipate some of the force of an abdominal blow, lessening the likelihood of rupture of the right hemidiaphragm. The most common appearance is loops of bowel protruding into the lower chest cavity without the normal dome-shaped structure of the hemidiaphragm ( Fig. 3–91 ).
- Mettler: Essentials of Radiology, 2nd ed.

147
Q

All indicate positive DPL EXCEPT:

a. Aspirations of 10 cc blood.
b. 10,000 RBC
c. 1000 WBC.
d. Amylase more than 20 IU/dl

A

Answer: Amylase more than 20 IU/dl

Positive DPL criteria (8):

  1. > 10 mL of gross blood aspirated
  2. Bloody lavage effluent
  3. RBC count > 100 000/ml (> 5000/mL in penetrating trauma to the low chest)
  4. WBC count > 500/ml
  5. Amylase > 175 IU/dl
  6. Bile
  7. Bacteria
  8. Food
148
Q

A patient in a trauma has a liver laceration. Intraoperatively, he receives 2 units of PRBC’s and is stable. You also notice a retroperitoneal hematoma which is non-expanding (also seen question where it IS expanding). You should:

a. Explore it
b. Close and do a ct scan
c. Intraoperative IVP
d. Intraoperative arteriogram
e. On table cystoscopy with retrograde pyelography

A

EXPLORE: penetrating trauma, zone 1, or rapidly expanding/rupture/pulsatile?
DO NOT EXPLORE: blunt zone 2 and 3

The location of a retroperitoneal hematoma and mechanism of injury guide the decision to explore the hematoma. The retroperitoneum is divided into three anatomic zones: the midline retroperitoneum (Zone 1), the perinephric space (Zone 2) , and the pelvic retroperitoneum (Zone 3) .

Any hematoma in zone 1 mandates exploration for both penetrating and blunt injury because of the high likelihood and unforgiving nature of major vascular injury in this area.

A hematoma in zone 2 is the result of injury to the renal vessels or parenchyma and mandates exploration for penetrating trauma to assess the damage and repair the injuries.

A pelvic retroperitoneal hematoma (zone 3) secondary to penetrating trauma mandates exploration because of the likelihood of iliac vessel injury. However, zone 3 hematomas resulting from blunt trauma are usually associated with pelvic fractures and are not explored because the effective management of this type of bleeding is based not on operative control (which rarely proves effective) but on external fixation or angiographic embolization of the bleeding vessels. The only exception is a rapidly expanding hematoma in which the surgeon suspects a major iliac vascular injury that requires operative repair.
- Townsend: Sabiston Textbook of Surgery, 18th ed.

After blunt trauma, selected retroperitoneal hematomas in the lateral perirenal and pelvic areas do not require operation and should not be opened if discovered at operation. Midline, lateral paraduodenal, lateral pericolonic not associated with pelvic, and portal hematomas are opened after proximal vascular control has been obtained, if appropriate. Retrohepatic hematomas without obvious active hemorrhage are not opened. After penetrating trauma, most retroperitoneal hematomas are still opened. Exceptions include isolated lateral perirenal hematomas that have been carefully staged by CT and some lateral pericolonic hematomas. As with blunt trauma, retrohepatic hematomas without obvious active hemorrhage are not opened.
- Feliciano. Management of traumatic retroperitoneal hematoma. Ann Surg. 1990 February; 211(2): 109–123

149
Q

A 35 year old sustains a gunshot wound to the left buttock while running from the police. He is seen in the ER where he is speaking coherently and is complaining of diffuse abdominal pain, but no buttock pain. On examination, his vital signs are stable, and he has signs of peritonitis. Your next move is to:

a) observe
b) get a contrast enhanced CT scan
c) perform a diagnostic peritoneal lavage
d) perform a flexible sigmoidoscopy
e) perform a laparatomy

A

Answer: laparotomy

Given the kinetics of bullet wounds, this pt probably has a bullet that has entered him through the buttock and traversed through to the retroperitoneal and abdominal organs. The bullet also forms a cavitation, injuring tissues not in the actual path of the bullet. Exit wounds are usually larger than entrance wounds. This pt has probably sustained an intrabdominal injury in addition to possible msk injury.
The basic principles of patient care for abdominal gunshot wounds is:
A,B,Cs
NG tube
Foley
Chest and abdominal X-rays if pt stable
Antibiotics and Tetanus
Surgery
Advanced Surgical Recall, blackbourne et al P329-334

Uptodate:
Clinical indications for laporotomy
-Unexplained signs of blood loss or hypotension in a patient who cannot be stabilized and in whom intraabdominal injury is strongly suspected
-Clear and persistent signs of peritoneal irritation
-Radiologic evidence of pneumoperitoneum consistent with a viscus rupture
-Evidence of a diaphragmatic rupture
-Persistent, significant GI bleeding seen in nasogastric drainage or vomitus

150
Q

What is the most common solid organ injury in blunt trauma?

A

(there are multiple references for both liver and spleen but the consensus was spleen)

Injuries to the spleen are the most common injuries in pediatric abdominal trauma. The liver is the second most commonly injured solid organ in the pediatric patient with abdominal trauma. However, it is the most common cause of lethal hemorrhage, with a mortality of 10 to 20% in severe liver injury.
- Marx: Rosen’s Emergency Medicine, 7th ed.

The liver and spleen seem to be the most frequently injured organs, although reports vary. Small and large intestines are the next most injured organs, respectively. Recent studies show an increased number of hepatic injuries, perhaps reflecting increased use of CT scanning and concomitant identification of more injuries.
- http://emedicine.medscape.com/article/821995-overview

151
Q

Which of the following statements is TRUE regarding renal trauma?

a. CT scan inferior to IVP for evaluating renal function.
b. Any extravasation after blunt renal injury is an indication for surgical exploration.
c. On routine exploration of a patient with a splenic rupture after blunt trauma, if a non-expanding perinephric hematoma is encountered, it should not be opened.
d. Before opening a perirenal hematoma, vascular control of the renal artery and vein must be controlled.
e. None of the above

A

Answer: D is true. C may be true, but is based on expert opinion. Answer key says C

An important technical aspect to keep in mind is that in major perinephric hematomas, proximal control of the renal pedicle before opening Gerota’s fascia is advisable.
Renal contusions encompass the vast majority of minor renal trauma and can almost invariably be treated nonoperatively. It is our opinion that all perinephric hematomas caused by penetrating mechanisms that have not previously been evaluated by IVP should be explored. If preoperative IVP shows renal pedicle injury, extensive parenchymal laceration, or urinary extravasation, surgical exploration remains the best option.
- Townsend: Sabiston Textbook of Surgery, 18th ed.

D is true. C may be true, but is based on expert opinion. Answer key says C

152
Q

18 year old female involved in MVA. After stabilization of the patient she was taken to CT, Her findings include a non expanding haematoma at the left lower kidney pole. The best way of management is:

a. Exploration
b. Angiogram and emobolization of the bleeding vessels
c. Observation
d. None of the above
e. All of the above

A

Answer: Observation

Renal contusions encompass the vast majority of minor renal trauma and can almost invariably be treated nonoperatively.
- Townsend: Sabiston Textbook of Surgery, 18th ed.

153
Q

25 year old male patient involved in a MVA has microscopic hematuria other wise stable. The treatment of choice is:

a. Reassure with follow up
b. CT scan
c. IVP
d. Surgical exploration
e. Cystogram

A

Answers given:

  • Reassure
  • IVP (disagree)

If patient was hemodynamically unstable and microscopic hematuria, then requires CT scan with and without control, including delayed phases.

All blunt trauma patients with gross hematuria and those patients with microscopic hematuria and shock (systolic blood pressure

154
Q

A man is kicked in perineum, and suffers a hematocele and can’t visualize other testicle. The next best thing in management is:

a. observe
b. operate to explore
c. perc drain

A

Answer: observe (Disagree. Likely get an ultrasound at least.)

Need to get a Doppler us first
Most testicular injuries are manage conservatively
Hematocele is due to either testicular contusion or rupture.
If rupture is present, then surgery is indicated to drain and repair.

Early exploration and repair of testis injury is associated with increased testis salvage, reduced convalescence and disability, faster return to normal activities, and preservation of fertility and hormonal function ( Kukadia et al, 1996 ). Minor scrotal injuries without testis damage can be managed with ice, elevation, analgesics, and irrigation and closure in some circumstances.
Significant intratesticular hematomas should be explored and drained even in the absence of testis rupture to prevent progressive pressure necrosis and atrophy, delayed exploration (40%), and orchiectomy (15%) ( Cass and Luxenberg, 1988 ). Significant hematoceles should also be explored, regardless of imaging studies, since up to 80% are due to testis rupture.
- Wein: Campbell-Walsh Urology, 9th ed.

155
Q

A 32 years old male involved in MVC has bilateral superior rami fracture with blood at the meatus. Which test?

A young man involved in a motorcycle accident is found to have a pelvic fracture. On physical exam, he is hemodynamically stable and is found to have blood at the urethral meatus. The next best step is …

a) insert foley
b) retrograde urethrogram
c) observe
d) cystogram
e) CT abdomen and pelvis

A

Answer: Retrograde urethrogram

Blood at meatus = buzz word for retrograde urethrogram, don’t put in a foley yet

In the clinically stable patient, triphasic abdominal and pelvic CT (precontrast study, followed by a study immediately after injection of contrast agent, and then a 15- to 20-minute delayed study) is the most sensitive method for diagnosis and classification of genitourinary trauma”
- Cambell’s Urolog

In trauma patients with clinical evidence of urethral injury (scrotal hematoma, blood at the meatus, high-riding prostate), do not pass foley catheter since may increase damage
If urethral injury is suspected, before any attempts to insert a foley, do a retrograde urethrogram as the study of choice to investigate
A scout film is obtained with the patient at 25-30 degrees in oblique position. Then insert 14 or 16 Fr foley a few cm into urethra. Inflate balloon with 1-2ml and inject contrast through catheter and take X-ray. If no extravasation, deflate balloon, advance and place in bladder. A cystogram is then done to evaluate the bladder
- Schwartz

156
Q

Indication for laparotomy? all except:

  1. Bullet through abdomen
  2. diaphragmatic rupture
  3. fractured kidney
  4. intraperitoneal bladder rupture
A

Answer: fractured kidney
I don’t even think this is an accepted terminology.

The American Association for the Surgery of Trauma (AAST) classifies renal injuries into five grades (Figs. 1 and 2):
• Grade 1: Nonexpanding subcapsular hematoma/contusion with absence of parenchymal injury
• Grade 2: Less than 1cm laceration into the renal cortex, not extending into the collecting system, with a nonexpanding hematoma confined to the perirenal fascia
• Grade 3: Greater than 1cm laceration, extending through the renal cortex and medulla but not the collecting system
• Grade 4: Laceration extending to the collecting system with urinary extravasation or a segmental vascular injury with contained hematoma; renal artery thrombosis
• Grade 5: Shattered kidney or renal pedicle avulsion
Follow-up imaging is indicated for patients with grade 4 or 5 injuries with urinary extravasation to assess for worsening urinoma or hematoma that might require further intervention.
- Bope: Conn’s Current Therapy 2010, 1st ed.

In intraperitoneal bladder injuries, open surgical repair of the laceration is the recommended treatment modality. This allows the surgeon to reduce any herniating omentum or small bowel from the confines of the bladder and allows careful intravesical inspection of the bladder neck at the time of surgical exploration.
- Wein: Campbell-Walsh Urology, 9th ed.

Intraperitoneal bladder rupture accounts for 25% of all bladder injuries. The postulated mechanism of intraperitoneal bladder injury is thought to be rapid rise of intra-abdominal pressure during a blunt trauma.[6] In contrast to extraperitoneal bladder injuries, intraperitoneal bladder rupture requires operative repair with two-layer closure of the bladder injury and placement of a perivesical drain.
- Townsend: Sabiston Textbook of Surgery, 18th ed.

157
Q

The most common abnormality found in blunt abdominal trauma on IVP is …

a) focal or global filling defect
b) leakage into retroperitoneum
c) absence of kidney
d) leakage of contrast from the collecting system
e) hydronephrosis

A

Probably obsolete question given that no one does IVP’s anymore

Answers provided: (???)

  • focal or global filling defect
  • leakage into retroperitoneum

80% of bladder rupture is extraperitoneal
- secondary to bone puncture from pelvic fracture
- contrast most commonly extravasates into the retropubic space of Retzius
- usually managed conservatively with foley catheter decompression unless laparotomy required for other reasons, then can fix primarily
20% of bladder rupture is intraperitoneal
-secondary to blunt trauma to distended bladder with rupture at the dome
-contrast extravasates into the paracolic gutters and outlines loops of bowel
BUT, apparently the kidney is the most commonly injured structure in the urinary tract (see above question) in traumatic injury
ALSO, hydronephrosis takes time to occur (days to week) and less than 1% of people have a single kidney, therefore these answers are less likely correct
- Fundamentals of Diagnostic Radiology, 2nd Edition. Brant

Blunt renal injuries are divided into minor and major injuries. Minor injuries make up approximately 85% of cases. Renal contusions encompass the vast majority of minor renal trauma and can almost invariably be treated nonoperatively. Major renal trauma includes deep cortical medullary lacerations with extravasation, large perinephric hematomas, and vascular injuries of the renal pedicle. Injury to the ureter is uncommon and occurs mostly after penetrating trauma.
- Townsend: Sabiston Textbook of Surgery, 16th ed., Copyright © 2001

158
Q

Trauma patient with flank pain and gross hematuria. CT shows a non expanding subcapsular hematoma. What is the most appropriate course of action?

a. Bed rest and repeat U/S in 3 days
b. OR now
c. OR in 3 days

A

a. nonop

The American Association for the Surgery of Trauma (AAST) classifies renal injuries into five grades (Figs. 1 and 2):

  • Grade 1: Nonexpanding subcapsular hematoma/contusion with absence of parenchymal injury
  • Grade 2: Less than 1cm laceration into the renal cortex, not extending into the collecting system, with a nonexpanding hematoma confined to the perirenal fascia
  • Grade 3: Greater than 1cm laceration, extending through the renal cortex and medulla but not the collecting system
  • Grade 4: Laceration extending to the collecting system with urinary extravasation or a segmental vascular injury with contained hematoma; renal artery thrombosis
  • Grade 5: Shattered kidney or renal pedicle avulsion

Routine early follow-up imaging for grades 1 to 3 blunt renal injury is unnecessary. Grade 4 renovascular injuries can be followed up clinically. Follow-up imaging is indicated for patients with grade 4 or 5 injuries with urinary extravasation to assess for worsening urinoma or hematoma that might require further intervention.
- Bope: Conn’s Current Therapy 2010, 1st ed.

159
Q

Conversion of proto-oncogenes to oncogenes is associated with all EXCEPT:

a. loss of both alleles
b. chromosome translocation
c. amplification

A

Answer: loss of both alleles

The proto-oncogene can become an oncogene by a relatively small modification of its original function. There are three basic activation types:

1) A mutation within a proto-oncogene can cause a change in the protein structure, causing
- an increase in protein (enzyme) activity
- a loss of regulation

2) An increase in protein concentration, caused by
- an increase of protein expression (through misregulation)
- an increase of protein (mRNA) stability, prolonging its existence and thus its activity in the cell
- a gene duplication (one type of chromosome abnormality), resulting in an increased amount of protein in the cell

3) A chromosomal translocation (another type of chromosome abnormality), causing
-an increased gene expression in the wrong cell type or at wrong times
the expression of a constitutively active hybrid protein

160
Q

What is correct about human oncogenes:

a. Regulate cell growth
b. Much bigger than viral oncogene
c. ???
d. ?????

A

Oncogenes encode proteins that control cell proliferation, apoptosis, or both. The products of oncogenes can be classified into six broad groups: transcription factors, chromatin remodelers, growth factors, growth factor receptors, signal transducers, and apoptosis regulators

161
Q

Cancer is:

a. clonal selection of cells with accumulated multiple genetic mutations
b. translocation to multiple proto-oncogenes
c. imbalance between cell growth and cell death

A

Answer: imbalance between cell growth and cell death

[M]alignant transformation is the process by which a clonal population of cells acquires alterations that confer a growth advantage over normal cells.
- Townsend: Sabiston Textbook of Surgery, 18th ed.

162
Q

With regards to cancer etiology which of the following is true:

a. Genetic mutation of clonal cells leads to cancer formation
b. No genetic rule
c. One abnormal stem cells that lead to cancer
d. All of the above
e. None of the above

A

Answer: Genetic mutation of clonal cells leads to cancer formation

[M]alignant transformation is the process by which a clonal population of cells acquires alterations that confer a growth advantage over normal cells.
- Townsend: Sabiston Textbook of Surgery, 18th ed.

163
Q

Which type of cells exhibits anti-tumour activiy/
Which of the following is TRUE regarding the immune system’s response to tumour cells?
A. Humoral response decreases tumour growth
B. Cellular response decreases tumour growth
C. Cellular response increases tumour growth
D. Humoral response increases tumour growth

A

Answer: Cellular response (T-cells) decreases tumour growth

The immune system is mediated by helper T lymphocytes (CD4); cytotoxic T lymphocytes (CD8), which directly lyse neoplastic cells, and natural killer (NK) cells and macrophages, which destroy tumor cells through secretion of cytokines such as interleukin-2 (IL2) and interferon-γ (IFN-γ).
- Walsh: Palliative Medicine , 1st ed.

T cells or T lymphocytes belong to a group of white blood cells known as lymphocytes, and play a central role in cell-mediated immunity.
- http://en.wikipedia.org/wiki/T_cell

164
Q

Cancer is caused by all except:

a) HBV
b) HCV
c) T H1 cells
d) CMV

Which of the following viruses is not associated with malignancy?

a. Hepatitis B Virus
b. Hepatitis C Virus
c. EBV
d. CMV
e. HTLV-l

All these viruses cause cancer except

a. Hep B
b. Hep C
c. EBV
d. HTLV-1
e. CMV

A

Answer: CMV (controversial) - found others say that Hep C is the correct answer

HBV infection is also a risk factor for hepatocellular carcinoma.
Hepatocellular carcinoma may ensue in 1% to 4% of chronic active hepatitis C patients per year.
- Townsend: Sabiston Textbook of Surgery, 18th ed.
However, found source that Hep C predisposes to cirrhosis which can lead to HCC but it is the cirrhosis, not the Hep C that is correlated.

Although human cytomegalovirus (HCMV) is generally not regarded to be an oncogenic virus, HCMV infection has been implicated in malignant diseases from different cancer entities. Detection of viral DNA, mRNA, and/or antigens in tumor tissues as well as seroepidemiologic evidence suggested a role of HCMV infection in the etiology of several human malignancies.

Transforming infections: This group includes several viruses that have been implicated in the causation of human cancer: EBV, HPV, HBV, and HTLV-1
- Kumar: Robbins and Cotran Pathologic Basis of Disease, Professional Edition , 8th ed.

165
Q

The TNM staging has been validated as improving all of the following EXCEPT:
A. Communication between specialists
B. Reproduction of published findings
C. Prognostication with specific diagnosis
D. Determination of most appropriate management
E. Standardize the treatment
F. Facilitate discussion
G. widely used by professionals

A

??E
helps select treatment but even within same TNM group?

TNM on initial presentation

166
Q

1) What is the most prevalent malignancy in men?
a) Prostate adenocarcinoma
b) Prostate squamous cell carcinoma
c) Lung adenocarcinoma
d) Lung squamous cell carcinoma
e) Colon adenocarcinoma

A
Top 3 as per CDC.gov - Rate per 100000
Prostate cancer (105.3) - adenocarcinoma makes 90% of it
Lung cancer (71.6)
Colorectal cancer (44.8)

Men
Incident: Prostate, lung, colorectal
Mortality: lung, colorectal, prostate

167
Q

5) What is the most common cancer to cause death among women?
a) breast
b) lung
c) colon
d) ovarian
e) endometrial

A
  • most common (by mortality) - women
    o lung cancer (26%)
    o breast cancer (15%)
    o colorectal cancer (9%)

Women
Incident: breast, lung, colorectal
Mortality: lung, breast, colorectal

168
Q

In which of the following situations is it LEAST appropriate to surgically resect an identified pulmonary metastasis

a. Primary tumor not controlled
b. Multiple unilateral lung metastases
c. Bilateral lung metastases
d. Resectable metastasis in another area
e. Unfavorable cell type

A

Answer: Primary tumor not controlled

169
Q

Cancer cells compared to normal (benign) tumour growth

a. Growth uncontrolled exponential
b. Diff fraction growth and cell death

A

Tumor growth follows a gompertzian exponential model ( Fig. 217-5 ). There is exponential growth in vitro and in vivo; it is fast in the beginning, and becomes slower as the tumor attains a larger size, tending to form a plateau that never completely arrives and that is fully achieved only in normal tissues.
- Walsh: Palliative Medicine , 1st ed.

170
Q

Chronic leg osteomyelitis developed chronic ulcer and discharge. What is the most common cancer will develop in this ulcer:

a. BCC
b. SCC
c. Osteosarcoma

PICTURE: The picture was a chronic ulcer, question what might be in the mass coming from it?

A

Answer: SCC

Can get SCC after chronic wound/chronic draining sinus/ostemoyelitis sinus

Squamous cell carcinoma may arise in the chronically infected granulation tissue that is adjacent to a chronically infected bone. This rare complication can occur in areas of burn scars, chronic pressure ulcers, and ostomies, as well as at sites of chronic draining osteomyelitis. The patient usually has had stable chronic osteomyelitis for an average of approximately 20 years, although the latent period can be as little as 18 months. The tumor can be difficult to distinguish visually from the granulation tissue. The focus of carcinoma appears to be an exuberance of proliferative, lobulated, and friable “granulation tissue.” A recent and progressive worsening of the soft-tissue defect in an otherwise stable case of chronic osteomyelitis is a clue that carcinoma has developed. Liberal sampling of the wound can easily be done in a clinic setting with a disposable skin punch biopsy trephine that dermatologists use for skin biopsies. Histologic examination of the biopsy specimens enables the pathologist to distinguish readily between squamous cell carcinoma and chronic granulation.

Marjolin’s ulcer: aggressive ulcerating squamous cell carcinoma presenting in area of previously traumatised , chronically inflammed, or scarred skin. Commmonly present in the context of chronic wounds including burn injuries, venous ulcers, ulcers from osteomyelitis and post-radiotherapy scars

171
Q
Which of the following has the highest specificity for the diagnosis of 
related disease: 
a. PSA- prostate 
b. Beta-HCG-choriocarcinoma 
c. CA19-9 – pancreatic cancer 
d. Calcitonin- medullary carcinoma
A

Answer: Calcitonin- medullary carcinoma (???)

Poor specificity of PSA screening, a great number of men who do not have prostate cancer will be subjected to biopsies, thus causing morbidity and increasing the cost of health care without any benefit. In fact, nearly 75% of men with a PSA between 4 and 10 ng/mL do not have prostate cancer.

Measurement of the free β-subunit is useful for the detection of recurrence or metastasis for choriocarcinoma when the intact hCG may remain normal. Analysis of serum hCG subunits is especially useful for managing patients with germ cell tumors ( von Eyben, 2003 ). However, elevated hCG can be found in trophoblastic tumors, choriocarcinoma, and testicular tumors. More than 60% of patients with nonseminomatous germ cell tumors and 10–30% with seminomas have elevated free β-hCG.
- McPherson ; Pincus: Henry’s Clinical Diagnosis and Management by Laboratory Methods, 21st ed.

CA119-9 not sensitive or specific

calcitonin is sufficiently elevated to be diagnostic of the presence of the tumor. In the early stages of the disease, however, the basal concentrations of calcitonin cannot be readily distinguished from normal. In these circumstances, provocative testing of calcitonin secretion can reveal the presence of the abnormal C-cells.

172
Q

Which of the following scenarios constitutes secondary prevention?
A. Completely avoiding a carcinogen
B. Detecting a cancer early when it is more amenable to successful therapy
C. Treatment of complications arising from cancer
D. Encouraging patients to quit smoking

A

Answer: Detecting a cancer early when it is more amenable to successful therapy

Primary prevention avoids the development of a disease. Most population-based health promotion activities are primary preventive measures.

Secondary prevention activities are aimed at early disease detection, thereby increasing opportunities for interventions to prevent progression of the disease and emergence of symptoms. (ex: screening)

Tertiary prevention reduces the negative impact of an already established disease by restoring function and reducing disease-related complications.

173
Q

Cancer secondary prevention:

a. Elimination of cancer cause.
b. Early cancer detection and treatment.
c. Early detection and management of cancer complication.
d. Decrease exposure to cancer causative factors

A

b. Early cancer detection and treatment.

174
Q

Which of the following cancers MOST frequently travel to the bone and present as satellite lesions?

a. breast
b. lung
c. prostate
d. kidney

A
Incidence of Skeletal Metastases from Autopsy Studies
 	 				Incidence (%) 
Primary Tumor	No. of Studies	Median	
Breast			        5		73	
Prostate		                6		68	
Thyroid		                4		42	
Kidney			        3		35	
Bronchus		        4		36	
Esophagus		        3		6	
Gastrointestinal tract	4		5
Rectum		                3		11
Bitches Like Penis that Kill 
Breast 
Lung 
Prostate 
Throid 
Kidney
175
Q
What is the most common presentation of a metastatic spine lesion
a – unilateral pedical disapprearance
b – anterior body loss of height
c – increased transpedicular distance
d – degenerative disc disease
A

Metastatic destruction of bone reduces its load-bearing capabilities, resulting initially in trabecular disruption and microfractures and, subsequently, in total loss of bony integrity. Rib fractures and vertebral collapse are the most common occurrences, resulting in loss of height, kyphoscoliosis, and a degree of restrictive lung disease.
- Abeloff: Abeloff’s Clinical Oncology, 4th ed.

176
Q

What is the mortality in a patient with 3 system organ failure?

a. >10%
b. >25%
c. >50%
d. >75%
e. >90%

A

C or D?

1 organ 30%
2 organ 50-60% (40-45%)
3 organ 80-90% (55-60%)
4 organ 100% (75-85%)

Other studies show diversity of results with rates varying from
14% to 40% with 1 system failure;
20% to 76% with 2 system failures;
30% to 90% with 3 system failures; the majority of studies showed mortality of
100% for 4 or more system failures.

177
Q

What is the mortality in a patient with 3 system organ failure?

a. >10%
b. >25%
c. >50%
d. >75%
e. >90%

A

1 organ 30%
2 organ 50-60% (40-45%)
3 organ 80-90% (55-60%)
4 organ 100% (75-85%)

Other studies show diversity of results with rates varying from
14% to 40% with 1 system failure;
20% to 76% with 2 system failures;
30% to 90% with 3 system failures; the majority of studies showed mortality of
100% for 4 or more system failures.

178
Q

A patient with C8 injury. BP 80/50 and HR is 50. He remained hypotensive despite 2L of RL. What is the most common cause:

a. Neurogenic shock
b. Spinal shock
c. Cardiac tamponade
d. Tension pneumothorax

A

Answer: neurogenic shock

Traditionally bradycardia with hypotension but are not always bradycardic. For this exam probably brady = neurogenic, tachy = something else

Complete injury to the cervical or upper thoracic spinal cord may result in sympathetic denervation manifested as loss of vasomotor tone in the periphery. Neurogenic shock should be considered in any trauma patient who is hypotensive but not actively bleeding.
- Goldman: Cecil Medicine, 23rd ed.

179
Q

Patient involved in MVA. Had C5 fracture. He is hypotensive and

tachycardic. What is the most probable diagnosis:
a. Head Injury
b. Neurogenic shock
c. Unrecognized intraabdominal injury
d. Spinal shock

A

Answer: Unrecognized intraabdominal injury

Traumatic Spinal Cord Injury
Other immediate concerns are bleeding and circulation. Hypotension may be due to either neurogenic shock or hypovolemia. For neurogenic shock, vasopressive pharmacologic agents such as phenylephrine (beginning as a continuous intravenous infusion at 100 μg/min with titration to clinical effect) may be needed. If tachycardia is present, hypovolemia is more likely, so fluid resuscitation would be more appropriate.
If the lesion is at T3 or above, sympathetic tone to the heart is compromised. In this setting, hypotension is accompanied by bradycardia, thus producing the neurogenic shock triad of bradycardia, hypotension, and peripheral vasodilation.
- Goldman: Cecil Medicine, 23rd ed.

180
Q

Which is not a criteria for SIRS?

A

Obsolete question since the criteria changed in 2016 and I doubt they updated the information.

BP not in this definition
Criteria for SIRS were established in 1992 as part of the American College of Chest Physicians/Society of Critical Care Medicine Consensus Conference.[2] The conference concluded that the manifestations of SIRS include, but are not limited to:
- Body temperature less than 36°C or greater than 38°C
- Heart rate greater than 90 beats per minute
- Tachypnea (high respiratory rate), with greater than 20 breaths per minute; or, an arterial partial pressure of carbon dioxide less than 4.3 kPa (32 mmHg)
- White blood cell count less than 4000 cells/mm³ (4 x 109 cells/L) or greater than 12,000 cells/mm³ (12 x 109 cells/L); or the presence of greater than 10% immature neutrophils (band forms)

181
Q

Septic patient in shock, which is not true

a. Cardiac Index 3.5
b. CVP 18
c. SVR 300
d. Hypotension

A

Answer: CVP 18

CI (N=2.4-4.0)
PCWP (N=6-12)
SVR (N = 900–1200 dyn·s/cm5 [90–120 MPa·s/m3])
CVP (N=8-12) high CVP means volume overloaded

a. Cardiac Index 3.5 – can be high in early sepsis
b. CVP 18 - volume depleted and usually reduced venous return
c. SVR 300 – true
d. Hypotension –true

182
Q

Septic patient in shock, which is not true

a. Cardiac Index 3.5
b. CVP 18
c. SVR 300
d. Hypotension

A

Answer: CVP 18

CI (N=2.4-4.0)
PCWP (N=6-12)
SVR (N = 900–1200 dyn·s/cm5 [90–120 MPa·s/m3])
CVP (N=8-12) high CVP means volume overloaded

a. Cardiac Index 3.5 – can be high in early sepsis
b. CVP 18 - volume depleted and usually reduced venous return
c. SVR 300 – true
d. Hypotension –true

183
Q

Of the following readings obtained from a Swan-Ganz Catheter, which is most consistent with a patient in cardiogenic shock?

a. CVP 1, CO 2, SVR 40
b. 15, 1.5, 35
c. 2, 4, 18
d. 7, 3.5, 25
e. 4, 5.0, 35

A

Answer: 15, 1.5, 35

increased, decreased, decreased

Cardiogenic shock is a reduction in the cardiac output, causing elevated central venous pressure and a reflex increased systemic vascular resistance and narrow pulse pressure.
- Rakel: Textbook of Family Medicine, 7th ed.

184
Q

Septic shock. Which is true:

a. Increased cardiac index.
b. Mostly due to gram positive bacteria.
c. Mortality is 5%.
d. Increased Arterio-venous O2 difference.

A

A. increased cardiac index

a. Increased cardiac index. (can be increased in early sepsis)
b. Mostly due to gram positive bacteria. (can be due to gram positive and ned)
c. Mortality is 5%. (much higher)
d. Increased Arterio-venous O2 difference. (? Mean that SV02 decreases)

Findings in sepsis:
HR : tachycardic
MAP 4 (high) : later can be depressed. heart is compensating
Pulmonary artery wedge pressure : decreased
SVR : low, distributive shock
O2 delivery: decreased (this is another common question)
Mixed venous O2 sat

185
Q

Which of these parameters is typical for septic shock

a) Increased vascular resistance, decreased vascular capacitance
b) Increased vascular resistance, increased vascular capacitance
c) Decreased vascular resistance, decreased vascular capacitance
d) Decreased vascular resistance, increased vascular capacitance

A

D.

CLASSIFICATION OF SHOCK
Increased vascular capacitance (venodilation) = Sepsis
- Goldman: Cecil Medicine, 23rd ed.

Definition of capacitance: The measure of a BLOOD VESSEL’s ability to increase the volume of BLOOD it holds without a large increase in BLOOD PRESSURE.

186
Q

HYPOVOLEMIC/HEMORRHAGIC SHOCK , clinical features?

A

HYPOVOLEMIC/HEMORRHAGIC SHOCK
- patient feels cold, postural hypotension and tachycardia, cool, pale, moist skin, low JVP,
decreased CVP, increased peripheral vascular resistance (PVR), concentrated urine

187
Q

Septic shock: clinical features?

A

SEPTIC SHOCK
- clinical features: warm skin (fever), decreased JVP, wide pulse pressure, increased cardiac output (CO),decreased systemic vascular resistance, increased heart rate (HR)

188
Q

OBSTRUCTIVE shock: clinical features?

A

OBSTRUCTIVE

  • increased JVP, distended neck veins, increased systemic vascular resistance (SVR)
  • insufficient cardiac output (CO)
189
Q

cardiogenic shock: clinical features?

A

CARDIOGENIC

- increased JVP, distended neck veins, increased SVR, decreased CO

190
Q

anaphylactic shock: clinical features?

A

ANAPHYLACTIC

- angioedema and hives

191
Q

Which of the following is the most appropriate initial step in the management of a patient in septic shock

a. identification of the source of sepsis
b. treatment with broad spectrum antibiotics
c. fluid resuscitation
d. dopamine infusion

A

C. fluid resusc

The first priority in any patient with severe sepsis or septic shock is stabilization of their airway and breathing. Next, perfusion to the peripheral tissues should be restored and antibiotics administered

1: airway + breathing
2: assess perfusion (BP, end-organ function, lactate…)
3: vascular access/fluid resuscitation - This is the answer
3. 5: pressors are second line
4. antibiotics, broad-spectrum

uptodate

192
Q
Activated protein C has been shown to be useful in sepsis, but may come with an INCREASED risk of:
A. Hemorrhage
B. Thrombosis
C. Anaphylaxis
D. Renal Failure
A

Answer: Hemorrhage

193
Q

Patient post op. developed C.diff positive diarrhea, abdomen is soft. Started on TPN, developed high anion gap metabolic acidocis, the most likely cause is :

a. TPN
b. Intra-abdominal sepsis.
c. Diarrhea

A

Answer: Intra-abdominal sepsis

From lactate?

TPN and Diarrhea cause NAGMA.

194
Q

A 65 year old female with a history of poorly controlled type I DM underwent an uncomplicated repair of a 6.5 cm AAA 10 days ago. She was discharged home yesterday in good condition. You are called to ER because she has returned via an ambulance with a decreasing level of consciousness and asked by the ER resident to rule out a leaking graft.
When you arrive, the patient is hypotensive. BP 60/, a non palpable radial pulse and non-responsive. You should do which of the following first?
a) Endotracheal intubation
b) Fluid bolus
c) Angiogram
d) Complete and rapid focused physical exam
e) Book the OR

A

A. ETT

ABC

A lot of questions from POS are like this - airway airway airway

195
Q
A patient presents with brain death.  Which of the following is the LEAST likely?
A. Hypoglycemia
B. DIC
C. Hypertension followed by hypotension
D. Decreased T3/T4
A

Answer: Hypoglycemia

In studies on brain death, the hypothalamic hormones, such as growth hormone–releasing hormone, corticotropin-releasing hormone, thyroxine-releasing hormone, and luteinizing hormone–releasing hormone were found in trace to subnormal levels.

When intracranial pressure increases, systemic hypertension occurs as a result of the Cushing phenomenon. This is followed by a sudden decrease in blood pressure because of sudden interruption of vasomotor output from the brainstem and hypothalamus to the spinal cord.

After an initial hyperdynamic response consisting of hypertension with or without tachycardia, brain-dead patients brought to the operating room for organ recovery experience hypotension, reduced cardiac output, myocardial dysfunction,[14] and vasodilation (decreased systemic vascular resistance). In addition, diminished oxygenation as a result of neurogenic pulmonary edema and diabetes insipidus secondary to decreased circulating levels of antidiuretic hormone may be observed. The latter also may lead to hypernatremia and hypokalemia. Hyperglycemia, coagulopathy, and hypothermia may be encountered and must be corrected when severe.
- Miller: Miller’s Anesthesia, 7th ed.

196
Q

All are necessary for declaration of brain death except

a. Dolls eyes and vestibulooccular reflex
b. Temperature >32 C
c. SBP >90mmhg
d. Lack of any response to painful stimuli

A

D. Lack of any response to painful stimuli

Pain reflex arch (ex: look up Lazarus sign - pretty cool)

A. Prerequisites.

  1. Clinical or neuroimaging evidence of an acute central nervous system catastrophe that is compatible with the clinical diagnosis of brain death
  2. Exclusion of complicating medical conditions that may confound clinical assessment (no severe electrolyte, acid-base, or endocrine disturbance)
  3. No drug intoxication or poisoning
  4. Core temperature ≥ 32°C (90°F)

B. The three cardinal findings in brain death are coma or unresponsiveness, absence of brainstem reflexes, and apnea.

  1. Coma or unresponsiveness—no cerebral motor response to pain in all extremities (nail-bed pressure and supraorbital pressure)
  2. Absence of brainstem reflexes
    a. Pupils
    i. No response to bright light
    ii. Size: midposition (4 mm) to dilated (9 mm)
    b. Ocular movement
    i. No oculocephalic reflex (testing only when no fracture or instability of the cervical spine is apparent)
    ii. No deviation of eyes to irrigation in each ear with 50 mL of cold water (allow 1 minute after injection and at least 5 minutes between testing on each side)
    c. Facial sensation and facial motor response
    i. No corneal reflex to touch with a throat swab
    ii. No jaw reflex
    iii. No grimacing to deep pressure on nail bed, supraorbital ridge, or temporomandibular joint
    d. Pharyngeal and tracheal reflexes
    i. No response after stimulation of the posterior pharynx with tongue blade
    ii. No cough response to bronchial suctioning
  3. Apnea testing performed as follows:
    a. Prerequisites
    i. Core temperature ≥ 36.5°C or 97°F
    ii. Systolic blood pressure ≥ 90 mm Hg
    iii. Euvolemia. Option: positive fluid balance in the previous 6 hours
    iv. Normal Paco2. Option: Paco2≥ 40 mm Hg
    v. Normal Pao2. Option: preoxygenation to obtain arterial Pao2≥ 200 mm Hg
    b. Connect a pulse oximeter and disconnect the ventilator.
    c. Deliver 100% O2, 6 L/min, into the trachea. Option: place a cannula at the level of the carina.
    d. Look closely for respiratory movements (abdominal or chest excursions that produce adequate tidal volumes).
    e. Measure arterial Pao2, Pco2, and pH after approximately 8 minutes and reconnect the ventilator.
    f. If respiratory movements are absent and arterial Pco2 is ≥ 60 mm Hg (option: 20 mm Hg increase in Pco2 over a baseline normal Pco2), the apnea test result is positive (i.e., it supports the diagnosis of brain death).
    g. If respiratory movements are observed, the apnea test result is negative (i.e., it does not support the clinical diagnosis of brain death), the test should be repeated.
    h. Connect the ventilator if, during testing, the systolic blood pressure becomes ≤ 90 mm Hg or the pulse oximeter indicates significant oxygen desaturation and cardiac arrhythmias are present; immediately draw an arterial blood sample and analyze arterial blood gas. If Pco2 is ≥ 60 mm Hg or Pco2 increase is ≥ 20 mm Hg over baseline normal Pco2, the apnea test result is positive (it supports the clinical diagnosis of brain death); if Pco2 is
197
Q

A patient is post op 5 hours from AAA repair. The CVP is 5, CI is low, PAWP is normal, PA pressures are normal. What is the most appropriate management?

a) Start Inotropes
b) Return to the OR
c) Transfuse
d) Shock
e) Give fluid

A

Answer: Give fluid

198
Q
Post AAA reair patient becomes unstable in ICU on day 1.  90/40, HR 140, u/o 10cc/hr. Received 6 L Crystaloid and 4U PRBC intra op. Best initial management
 a – PRBC
 b – Crystaloid
 c – Vasopressin
 d – Norepinephrine
A

Previous answer said go back to OR. I disagree. Given the options this person may have cardiogenic shock: Tx of cardiogenic shock = norepinephrine (first line), dopamine (second line).

Also, this person sounds like they are not dry based on the fluids received, and they were ok until POD1.

So I say D: norepinephrine

Complications:
From PMHx: heart failure, myocardial ischemia, acute kidney injury, pulmonary insufficiency and pneumonia

From surgery (early):
lower extremity ischemia, bowel ischemia, pelvic ischemia, renal dysfunction

Late complications :incisional hernia, anastomotic aneurysm and graft infection/aortoenteric fistula

199
Q

Swan ganz measure which of the following parameter directly:

a. C.I
b. LA diastolic pressure
c. SVR
d. Mixed venous O2 sat.

A

Answer: Mixed venous O2 sat.

Swan Ganz (pulmonary artery catheter) is on the right side of the heart.. So cannot detect L-sided stuff or peripheral stuff (CI, LA diastolic pressure, SVR)

Direct measurements: Right Atrial/Ventricle pressures, Pulmonary artery pressure, PCWP, and mixed venous 02 sat (CvO2)
Indirect: CO/C.I. via Fick Principle (CO= V02/CaO2-CvO2) or indicator-dilution technique, PCWP = Left atrial pressure = (assumed) left ventricle end diastolic pressures = LVED volume, Stroke volume

200
Q

PA catheter, all are correct except:

a. Reduce mortality
b. Can guide resuscitation
c. Can measure LV pressure

A

Answer: Reduce mortality

No difference in mortality and morbidity, except higher incidence of PE in the PA catheter group.

A randomized, controlled trial of the use of pulmonary-artery catheters in high-risk surgical patients. Conclusion: no benefit

201
Q

If minute ventilation and carbon dioxide production are fixed, which of the following will decrease PaCO2?

a) add PEEP
b) decrease the respiratory rate
c) increasing the tidal volume
d) increasing the residual volume
e) decrease dead space

A

Answer: increasing the tidal volume

Given that the minute ventilation is constant, we want to increase the % of non-dead-space ventilation per breath. Since the dead-space ventilation is a constant volume, we need to increase the tidal volume to result in increased ventilation.

Minute Ventilation = amount of air person inspires or expires in a minute
MV = TV X RR
MV = volume/breath * breath/min
MV = Volume/min

202
Q

Patient intubated in the ICU and on mechanical ventilation. He is on FiO2 of 75% with the following blood gas: PaO2 50, PCO2 38, pH 7.25. What would be your next step?

  1. increase FiO2 to 90%
  2. administer IV bicarb
  3. hyperventilate
  4. increase respiratory rate.
A

Answer: increase FiO2 to 90%

PaO2: hypoxia
PCO2: slightly elevated
pH: acidotic

Needs more oxygen. O2 exchange is not really affected by resp rate so need to increase oxygen supply. (Or increase PEEP but that is not an option)

A-a gradient = PAO2 – PaO2
[FiO2(Patm-PH20) – PaCO2/RQ] – PaO2
[.75(760-47) – 38/0.8] – 50
437.25

If A-a gradient > 15 = decreased diffusion capacity, ILD, emphysema
Tx = give 100% O2

203
Q
The MOST accurate measurement of CVP for a patient receiving positive pressure ventilation is:
A. At end expiration
B. At end inspiration
C. At mid-expiration
D. At mid-inspiration
A

Answer: At end expiration

At the end of expiration, intrathoracic and juxtacardiac pressures approach atmospheric pressure regardless of ventilatory status, and the CVP values will coincide. Proper pressure values can also be determined by visual inspection of the CVP waveform on a calibrated monitor screen or paper recording. This facilitates comparison of CVP values (and other cardiac filling pressures) obtained from the same patient under varying patterns of ventilation, a common situation in anesthesia and critical care.
- Miller: Miller’s Anesthesia, 7th ed.

204
Q

What is the BEST predictor of requirement for post-operative ventilation?
A. FEV1

A

Answer: probably pre-op dyspnea at rest

Pre-op dyspnea at rest = CHF probably = bad (40 pack-years)
• ASA-PS scores higher than 2
• Age >70 years
• COPD
• Neck, thoracic, upper abdominal, aortic, or neurologic surgery
• Anticipated prolonged procedures (>2 hours)
• Planned general anesthesia (especially with endotracheal intubation)
• Albumin less than 3 g/dL
• Exercise capacity of less than two blocks or one flight of stairs
• BMI greater than 30
Surprisingly absent predictors in this list are asthma and results from arterial blood gas analysis or PFTs.
- Miller: Miller’s Anesthesia, 7th ed.

205
Q

1) what are the criteria for extubation:
a. – inspiration pressure 15cc/kg (yes)
c. TV >5 cc/kg (yes)
d. FEV > 1l

2) Criteria for extubation include all, EXCEPT:
a) Negative inspiratory pressure 15ml/kg (yes VC between 10-15)
c) TV > 5ml/kg (yes)
d) FEV-1 > 50% predicted

3) Extubation criteria include all of the following EXCEPT
a) mean inspiratory pressure greater than 5cc /kg/ min
b) tidal volume greater than 5cc /kg /min
c) respiratory rate greater than 5bpm and less than 35bpm
d) forced expiratory volume in one minute
e) vital capacity greater than 15cc /kg / min

4) All of the following parameter predict successful weaning EXCEPT
a. negative inspired pressure >-25 cm H2O
b. tidal volume > or = 7 mL/kg
c. respiratory rate or = 10 mL/kg
e. minute ventilation >= 12 L/min (a systematic review found that minute ventilation is a poor predictor of weaning outcome, uptodate)

A

Answers: anything with FEV1

Possible Criteria for Ventilator Weaning
TEST 						CRITERION
MECHANICAL FUNCTION	
Vital capacity (FVC)				>10-15 mL/kg
FEV1 						>10 mL/kg
Tidal volume (V T)				>5 mL/kg
Maximum inspiratory pressure (MIP)		2 times resting V E
Respiratory rate (V F)				>25/min
Respiratory minute volume (V E)		50% predicted

GAS EXCHANGE FUNCTION
A-a gradient (F IO2 = 1.0) 200
P100 7.30
- Miller: Anesthesia, 5th ed., Copyright © 2000

206
Q

All of the following are seen with PEEP >20 EXCEPT:

a. Decreased left atrial venous return.
b. Pneumothorax.
c. Decreased right atrial venous return.
d. Decreased central venous pressure.

A

Answer: decreased central venous pressure

The degree of this elevation depends on the compliance of the lungs and intravascular volume and will vary between patients. For this reason, CVP measurements are best made and compared at the same point in the ventilatory cycle⎯usually end-expiration. When positive end-expiratory pressure (PEEP) is applied, the positive pressure is transmitted through to the right atrium, causing a decrease in venous return and a rise in CVP.

207
Q

Someone with PEEP develops sudden hypotension. What is the most likely cause?

a) tension pneumothorax
b) pneumomediastinum
c) pneumoperitoneum

A

Answer: tension pneumothorax

Potential risk of PEEP:
o pneumothorax can develop at high pressures (>20cm H2O)
o tension pneumothorax will subsequently develop as air is forced into the lungs and out the rupture site into the thoracic cavity (one-way leak), thus lung collapse. Results in displacement of mediastinum to opposite side, therefore decreasing venous return causing hypotension
Most common cause of tension pneumothorax is mechanical ventilation with PEEP in the patient with visceral pleural injury
- Schwartz, ATLS manual

208
Q

A resp question with the following blood gas: paO2=60, paCO2=24, pH= 7.55, FiO2= 30%, Vt=10ml/hr, What is the abnormality and what would you change in the vent settings?

A

Respiratory Alkalosis - reduced respiratory rate

209
Q

A patient is in stage 2 ARDS when:

a. Decreasing PaO2 despite increasing FiO2
b. Bilateral CXR infiltrates
c. Decreased LOC
d. Falling PCO2

A

Answer: Decreased pO2 despite increasing FiO2

Stages of ARDS
1) Injury or Exudative stage (1-7 days)
Accumulation of fluid/protein lead to endothelial damage
Inflammatory cells in response to the initial injury
Proteinaceous fluid flooding in interstitium and alveoli is an early appearance

2) Reparative or Proliferative stage (1-2 weeks)
Continued inflammatory response
Complete when lung composed of dense, fibrous tissue

3) Fibrotic stage (2-3 weeks)
Lung completely remodeled by fibrous tissue
Also called chronic or late phase

210
Q

2) Factors associated with ARDS, all except:
a) CVP >18.
b) PCWP

A

A

Acute respiratory distress syndrome (ARDS), is characterized by hypoxia (despite oxygen therapy), decreased pulmonary compliance, diffuse or patchy infiltrates on chest x-ray, and noncardiac pulmonary edema.

ARDS criteria (uptodate 2016):
-Resp symptoms started/worsened 5cmH2O: 
PaO2/FiO2 >200mmHg = mild
PaO2/FiO2 100-200mmHg = moderate
PaO2/FiO2
211
Q

A 67-year-old ♀ is on a ventilator in the ICU. Her blood pressure is noted to be 100/72. The tidal volume is noted to be 15cc/kg. The PaO2:FiO2 ratio is

A

Answer: Decrease the tidal volume TV

212
Q

The mechanism for low PO2 in ARDS includes:

a. leaky pulmonary capillaries
b. increased functional dead space
c. something about heart shunting

A

Answer: Previous answer leaky pulmonary capillaries but uptodate (2016) states ventilation/perfusion mismatch and shunt physiology

Consequences — Lung injury has numerous consequences including impairment of gas exchange, decreased lung compliance, and increased pulmonary arterial pressure.

●Impaired gas exchange – Impaired gas exchange in ARDS is primarily due to ventilation-perfusion mismatching: physiologic shunting causes hypoxemia, while increased physiologic dead space impairs carbon dioxide elimination [28,29]. A high minute volume is generally needed to maintain a normal arterial carbon dioxide tension (PaCO2), although hypercapnia is uncommon. (See “Oxygenation and mechanisms of hypoxemia”.)

●Decreased lung compliance – Decreased pulmonary compliance is one of the hallmarks of ARDS [30]. It is a consequence of the stiffness of poorly or nonaerated lung, rather than the pressure-volume characteristics of residual functioning lung units [31]. Even small tidal volumes can exceed the lung’s inspiratory capacity and cause a dramatic rise in airway pressures [30].

●Pulmonary hypertension – Pulmonary hypertension (PH) occurs in up to 25 percent of patients with ARDS who undergo mechanical ventilation [32-34]. Causes include hypoxic vasoconstriction, vascular compression by positive airway pressure, parenchymal destruction, airway collapse, hypercarbia, and pulmonary vasoconstrictors [35]. The clinical importance of PH in most patients with ARDS is uncertain. PH severe enough to induce cor pulmonale is rare, but it is associated with an increased risk of death [36,37].

213
Q

What is the best method of preventing alveolar fluid accumulation in ARDS?

a) Increase intravascular protein
b) End expiratory pressure
c) Increase tidal volume

A

Answer: Positive end expiratory pressure

Positive end-expiratory pressure (PEEP) can improve oxygenation by recruiting collapsed alveoli and increasing functional residual capacity. Conventional ventilation generally calls for the minimal PEEP necessary to provide acceptable oxygenation. However, in the setting of ARDS there may be benefit in increasing PEEP to improve oxygenation, as well as protect the lung, by preventing the repetitive recruitment/derecruitment of alveoli and thereby reducing cyclic reopening and stretch during mechanical breaths.
- Townsend: Sabiston Textbook of Surgery, 18th ed.

214
Q

ARDS causes all of the following except :

a. Increase functional residual capacity
b. Bilateral chest infilterates
c. Severe hypoxemia
d. PCWP

A

Answer: Increase FRC

Patients with ARDS have noncardiogenic pulmonary edema, with a reduced functional residual capacity and a mortality rate of 30 to 60%.
- Goldman: Cecil Medicine, 23rd ed.

215
Q

Patient post MVA received massive transfusion. He developed hypoxia.
CVP was 12. CXR showed bilateral infiltrate. His PaO2/FiO

A

c. ARDS

Hypoxia + low PaO2/FiO2 + bilateral infiltrate + trauma mechanism (?pulm contusion) and massive transfusion (both trauma + massive transfusion can cause ARDS)

216
Q

What is the goal of CPR?

a. Maintenance of systemic blood flow
b. maintenance of cerebral perfusion
c. maintenance of coronary artery perfusion
d. return of spontaneous cardiac output
e. none of the above

A

Answer: maintenance of cerebral perfusion

217
Q
A patient who underwent an elective bowel resection presents with crushing retrosternal chest pain, diaphoresis and tachycardia. His BP is 80/50, HR 140. What is the OPTIMAL next step?
 A. Emergency angioplasty
 B. Systemic thrombolysis
 C. IV nitroglycerine
 D. IV Beta-blocker
 E. Add oxygen by facemask
A

Answer: Add oxygen by facemask

Once the diagnosis is confirmed, treatment is largely supportive. Satisfactory relief of pain and anxiety is important and cn be accomplished using morphine and sedation. Maintenance of optimal oxygenation is crucial, and effective pulmonary toilet o tclear bronchial secetions is combined with supplemental oxygen to maintain the arterial oxygen saturation at greater than 95%. All patients should be initially treated in an ICU or monitored bed. If evidence of hemodynamic instability or respiratory distress is present, intraarterial catheter monitoring should be utilized. Frequent determinations of fluid status and cardiac output to guide diuretic and vasopressor therapy may require pulmonary artery catheter monitoring.
- Schwartz

SHOCK trial demonstrated that percutaneous coronary intervention (PCI) or coronary artery bypass are the treatments of choice and that they have been shown to markedly decrease mortality rates at 1 year. PCI should be initiated within 90 minutes of presentation; however, it remains helpful, as an acute intervention, within 12 hours of presentation. If such a facility is not immediately available, thrombolytics should be considered. However, this treatment is second best. An increased mortality is seen in situations where thrombolytics are used instead of PCI.

218
Q

A 65 y.o. ♂ undergoes a CABG x 3. Two hours later he develops rapid atrial fibrillation with a pulse of 180 and a blood pressure of 70/40. Which of the following is the BEST course of action?
A. IV B-Blocker
B. Electrical cardioversion
C. IV Verapamil
D. IV Amiodarone bolus followed by infusion

A

B. Electrical cardioversion

ATLS - unstable a. fib

Test-taking skills: verapamil/amiodarone are both reasonable medications for rapid A.fib, so therefore they are both not the answer to this question

219
Q
A patient is found in VF postoperatively. CPR is initiated and you arrive on the scene 90 sec after the start of the code. The FIRST appropriate step in management is:
A. IV fluid bolus
B. Epinephrine, 1 mg IV
C. 200J countershock 
D. Pericardiocentesis
E. Order a 12 lead ECG
A

C. Shock

(check rhythm then shock, biphasic 200J, monophasic 360J)

ATLS

220
Q

Given an ECG showing ventricular fibrillation post-hysterectomy, best management:

a. Lidocaine
b. Epinephrine
c. Bretylium
d. Shock
e. Fluid blous

A

D. zap zap zap

ATLS

221
Q

A patient is 4 hrs post cardiac surgery when she develops V Tach. Her HR=150, BP=100/70, and CI=3.4. She is still intubated. The best course of action at this point is:

a. defibrillate with 100 J
b. defibrillate with 360 J
c. give lidocaine IV
d. give procainamide IV

A

Previous answer: lidocaine
As of 2010, ATLS no longer recommends lidocaine

Procainamide if stable is reasonable
I would say shock cause she’s already intubated - don’t need to sedate. Plus electricity is cheap and meds only have ~30% chance to convert and take too long

222
Q
A 70 y.o. ♂ immediately post op has a BP 70/50, HR = 105 and ST depression in the anteroseptal leads. The MOST appropriate management is: 
A. b-blocker
B. nitro s/l
C. nitro IV
D. epinephrine
E. dobutamine
A

Answer: dobutamine or dopamine (obsolete - now norepinephrine is the correct answer)

Cardiogenic shock management

2016 - uptodate: suggest norepinephrine as first line, dopamine as second line pressors for cardiogenic shock:

In patients with cardiogenic shock, norepinephrine is preferred over dopamine as the first-line vasopressor because a subgroup analysis from a randomized trial found that patients with cardiogenic shock who received dopamine had a higher mortality than those who received norepinephrin. In addition, dysrhythmias were more common in the dopamine group.

223
Q

What is the effect of using epinephrine in asystolic arrest?

a. Increase preload
b. Decrease preload
c. Increase inotropy
d. Increase afterload
e. Decrease afterload

A

Answer: increase inotropy

Catecholamines are used in cardiac arrest not only in an attempt to achieve better electrical stability (e.g., conversion from fine to coarse VF, or increasing the rate of spontaneous contraction during bradyarrhythmias) but also for their inotropic and peripheral vascular effects. Epinephrine is the first choice among the catecholamines for use in cardiac arrest because it increases myocardial contractility, ele-vates perfusion pressure, may convert electromechanical dissociation to electromechanical coupling, and improves chances for defibrillation.
- Libby: Braunwald’s Heart Disease: A Textbook of Cardiovascular Medicine, 8th ed.

224
Q

In the recovery room, an 80 year old man has the following ECG (shows normal rhythm followed by only P waves/without QRS complexes). First step of management is:

a. Put on defibrillator pads
b. Set up for transcutaneous pacing
c. Set up for transvenous pacing
d. IV epinephrine
e. (note: CPR was not an option here)

A

Answer: Set up for transcutaneous pacing

Bradycardia/Complete heart block

225
Q

A 56 year old male develops a post-operative narrow complex tachycardia that is regular. His BP is 120/80. What is the drug of first choice in treatment?

a. Metoprolol
b. Verapamil
c. Diltiazem
d. Adenosine
e. Digoxin

A

Answer: adenosine

Regular narrow complex tachycardia = SVT.

In hemodynamic stable patients, the drug of choice is adenosine. 2nd drug of choice is verapamil.

If patient is unstable and HR over 150/min : electric cardioversion is the Tx.

Adenosine slows conduction through the AV node, interrupts AV-nodal reentry pathways, and can restore normal sinus rhythm in patients with PSVT, including those ass with WPW.
- ACLS

226
Q
A post-op patient is in respiratory distress. Upon exam, he has a Grade IV view and bag-valve mask ineffective. What is the MOST appropriate next step?
A. Perform a tracheostomy
B. Perform a cricothyroidotomy
C. Insert an LMA
D. Attempt nasopharyngeal intubation
A

B. Insert LMA

The goal here is to turn this “can’t intubate, can’t oxygenate” emergency situation into a “can’t intubate, CAN oxygenate” which is less emergent. In any case, you should only try one time to do the LMA before movign on to a cric. Ideally you slam the LMA in while someone else is getting things ready to cricothyroitomize in case you fail. If you get your LMA in - great. If not, cric.

Uptodate2016

227
Q

Facial trauma and mandibular fractures. Best way to provide initial airway is:

a. oral endotracheal tube
b. nasal trumpet
c. tracheostomy
d. jaw thrust

A

Answers provided:

  • oral endotracheal tube
  • Cricothyrodiotomy

In practice you can still intubate these patients… So I vote ETT.

Although facial fractures in and of them selves are not an indication for tracheotomy, in cases of severe maxillo-facial trauma, tracheotomy is sometimes used to secure an airway where intubation would be difficult or damaging.
- Cummings: Otolaryngology: Head & Neck Surgery, 4th ed.

The primary indication for cricothyrotomy is the need for a definitive airway in the victim in whom orotracheal or blind nasotracheal intubation (BNTI) has failed, is contraindicated, or is extremely difficult. Clinical scenarios include the victim with severe facial trauma for whom conventional airway management is extremely complicated or unfeasible, the victim with severe laryngeal edema, or the victim with upper airway foreign-body obstruction that cannot be alleviated by the Heimlich maneuver or direct laryngoscopy.
- Auerbach: Wilderness Medicine, 5th ed.

Patients with an unstable cervical spine, massive facial injuries, or inability to undergo laryngoscopic intubation benefit from the ease of intubation with the help of the flexible bronchoscope. Bronchoscopic guidance is also valuable in the performance of percutaneous dilational tracheostomy.
- Mason: Murray & Nadel’s Textbook of Respiratory Medicine, 4th ed

228
Q

A patient involved in MVA sustained the following injuries:
bilateral mandibular fracture, facial fracture and basal skull fracture. His air way was clear then his level of consciousness starts to deteriorate the best way to protect air way is:
a. Oral intubation with cervical fixation
b. Tracheotomy
c. Nasotracheal intubation
d. Cricothyrodiotomy
e. None of the above

A

A. ETT

Basal skull fracture = DO NOT DO NASOTRACHEAL INTUBATION

Wheeless Textbook
Facial Fractures and Upper Airway Injuries:
-in pts with major frxs of the mandible and maxilla (Lefort III) in whom massive edema has yet to occur, oral intubation is preferred, and if required is usually easily accomplished;
- blind nasal intubation following major facial injury is discouraged because of the hazard of potential false passages into nasal sinuses and cranial vault;
-injuries of the Larynx may cause rapid respiratory obstruction and require immediate tracheostomy;
- in less urgen situation, a history of trauma to the head and neck, stridor, hoarseness, and crepitus in the neck are all suggestive or laryngeal injury;

229
Q

Inhaled helium and oxygen mixtures decreases symptoms of airway obstruction by:

a) Increased oxygenation
b) Decreased turbulence
c) Decreased edema of the vocal folds
d) Forgetting stress by making your voice sound funny

A

Answer: Decreased turbulence

Gas: Helium 79%: Oxygen 21%
used since 1990 to help with upper airway obstruction
He: decreased airway resistance therefore decrease mechanical energy to vent

230
Q

Patient presents with hoarseness post extubation. History is consistent with a traumatic intubation. What is the most likely cause?

  1. cricoarytenoid dislocation
  2. compression injury to the superior laryngeal nerve
  3. endotracheal tube was too large
A

Better answer: ETT was too large
Previous Answer: cricoarytenoid dislocation (?? rare)

Complications are common during mechanical ventilation. Adverse events that are directly attributable to the ETT include laryngeal injury (eg, laryngeal edema, vocal cord dysfunction), swallowing impairment, tracheal stenosis, tracheoesophageal fistula, cuff leaks, sinusitis, and biofilm formation

Risk factors for laryngeal injury that have been described include prolonged intubation (variably defined as ≥36 hours to ≥3 days), traumatic intubation, not using a myorelaxant drug during intubation, a large ETT (>8 mm in men, >7 mm in women), aspiration, and the presence of a nasogastric tube

231
Q

A very obese person is being intubated. The reason for the difficulty in visualizing the airway is:

a) Soft tissues compressing the trachea
b) Inability to extend the neck fully

A

b?

Check for optimal head positioning: neck slightly flexed and head extended on the neck (conditions permitting). May be facilitated by placing a towel under the patient’s occiput to raise it 10 cm. Obese patients usually require significantly more occiput elevation.
- Roberts: Clinical Procedures in Emergency Medicine, 5th ed.

232
Q

55 year old man presents with induration and erythema in the submandibular region, crossing midline. Patient complains of sore throat and increasing dysphagia but is otherwise sating well and not in any respiratory distress. Examination of oral cavity reveals induration and swelling at the floor of mouth, pushing the base of tongue posteriorly. What is the most appropriate next step.

  1. incision and drainage of submandibular region in the OR.
  2. transfer patient to ICU and perform rapid sequence intubation.
  3. transfer patient to OR for flexible bronchoscopic intubation and possible tracheostomy.
  4. send patient home.
A
  1. transfer patient to OR for flexible bronchoscopic intubation and possible tracheostomy.

Ludwigs angina

233
Q

Which one of the following does not activate platelet directly:

a. IL-6
b. Collagen
c. Fibrin
d. ADP

A

c. Fibrin

Stimulators of Platelets

  • ADP = most important
  • collagen = attracts platelets to the subendothelium
  • vWF
  • thrombin

Inhibitors

  • NO
  • PGI2 (“platelet gathering inhibitor”)
  • ADPase
234
Q
When do platelets return to normal function after stopping naproxen?
A. 2 days
B. 4 days
C. 6 days
D. 8 days
A

Answer: 1-3 days

The remaining COX-1 NSAIDs such as naproxyn, ketorolac, diclofenac, piroxicam, ibuprofen, and others also act as prostaglandin synthesis inhibitors. All of them cause reversible competitive platelet inhibition, and platelet function usually returns to normal within 1 to 3 days after stopping the drug

235
Q

which of the following dose not affect the function of platelet:

a. ASA
b. Papaverin
c. Dextran
d. Dipyrimadole
e. Phenylbutasone

A

Answer: b. Papaverin

ASA inhibits COX decreases Thromboxane A2 which stimulates platelet aggregation

Papaverin smooth muscle relaxant. “PVD, CVD”

Dipyrimadole (persantine)-impairs platelet function by inhibiting activity of ADP. Used in pulomonary HT.

Phenylbutasone-NSAID COX inhibition.

dextran 70 may interfere with normal blood clotting, causing a deficiency with crossmatching procedures and possibly a bleeding diathesis

236
Q

A patient with chronic renal failure on haemodialysis has a coagulopathy. What is the most appropriate explanation:

a. Thrombocytopenia
b. Qualitative platelet defect
c. Hypoprothrombinemia
d. Vitamin K deficiency
e. Anaemia of chronic disorders

A

Answer: b. qualitative platelet problem

An association between renal failure and bleeding tendency has long been recognized. A qualitative defect in platelet function produced by uremia itself seems to be central. The defect results from accumulation of the compound guanidinosuccinic acid in uremic blood, which inhibits adenosine diphosphate–induced platelet aggregation.
- Miller: Miller’s Anesthesia, 7th ed.

237
Q

Antithrombin III deficiency:

a. Sensitive to heparin
b. Warfarin contraindicated
c. Risk of recurrent arterial/venous thrombosis
d. No known factor to replace

A

Answer: C

Insensitive to heparin since heparin acts by potentiating antithrombin III

The most common thrombotic manifestations in patients with antithrombin deficiency (AT deficiency) include lower extremity VTE, with recurrent VTE being common. Other sites of thrombosis include the inferior vena cava, hepatic and portal veins, and renal, axillary, brachial, mesenteric, pelvic, cerebral, and retinal veins. Arterial thrombosis is strikingly less common.
In a patient with a known inherited antithrombin deficiency (AT deficiency), management of the acute thrombotic event depends on the type of antithrombin deficiency, because a variable response to large doses of heparin occurs in some of these patients. When a therapeutic response to intravenous heparin is not achievable, additional support with an antithrombin concentrate may be necessary.
Antithrombin concentrates are used to raise the plasma antithrombin level from a reduced value to approximately 120%.
- http://emedicine.medscape.com/article/198573-treatment

In antithrombin III deficiency, the activity of LMWH is not as reliable as in an otherwise healthy person. Careful monitoring of the anti-Xa activity in the patient should be performed. Consider alternative anticoagulation medications (eg, warfarin) because the effectiveness of LMWH is likely reduced.
- http://emedicine.medscape.com/article/954688-treatment

238
Q

What disease casues decreased platelet quality and quantity?

a. Benard Soulier syndrome
b. Unremia
c. Liver disease
d. vWD

A

C is true
A can also be true
I don’t know the answer.

von Willebrand disease is due to an abnormality, either quantitative or qualitative, of the von Willebrand factor, which is a large multimeric glycoprotein that functions as the carrier protein for factor VIII (FVIII). von Willebrand factor is also required for normal platelet adhesion. As such, von Willebrand factor functions in both primary (involving platelet adhesion) and secondary (involving FVIII) hemostasis.
- http://emedicine.medscape.com/article/959825-overview

Bernard-Soulier syndrome (BSS) was first described in 1948 as a congenital bleeding disorder characterized by thrombocytopenia and large platelets.
- http://emedicine.medscape.com/article/954877-overview

Bleeding time is generally very prolonged in patients with uremia, signifying a major defect in platelet function, which improves after dialysis
- http://emedicine.medscape.com/article/201722-overview

The life span of platelets in circulation is 7 to 10 days. Under normal conditions, the spleen stores about one third of circulating platelets. Circumstances that increase splenic volume, such as hepatic cirrhosis or portal hypertension, cause a reduction in the circulating platelet count by a sequestration within the splenic sinusoids.
- Firestein: Kelley’s Textbook of Rheumatology, 8th ed.

239
Q
  1. The mechanism of LMWH is:
    a. Binds to anti-thrombin 3
    b. Binds to platelets
A

a. binds to AT3
Like heparin, low-molecular-weight heparin exerts its anticoagulant activity by activating antithrombin
Consequently, low-molecular-weight heparin catalyzes factor Xa inhibition by antithrombin more than thrombin inhibition

240
Q

The principal difference between LMWHs and Heparin are that LMWHs have a lesser effect on:

a. Haugen factor
b. Antithrombin III
c. Thrombin
d. Factor Xa

A

Answer: Thrombin

Like heparin, low-molecular-weight heparin exerts its anticoagulant activity by activating antithrombin. With a mean molecular weight of 5000, which corresponds to approximately 17 saccharide units, at least half of the pentasaccharide-containing chains of low-molecular-weight heparin are too short to bridge thrombin to antithrombin (Fig. 137-4). However, these chains retain the capacity to accelerate factor Xa inhibition by antithrombin because this activity is largely the result of the conformational changes in antithrombin evoked by pentasaccharide binding. Consequently, low-molecular-weight heparin catalyzes factor Xa inhibition by antithrombin more than thrombin inhibition. Depending on their unique molecular weight distributions, low-molecular-weight heparin preparations have anti–factor Xa to anti–factor IIa ratios ranging from 2 : 1 to 4 : 1.
- Hoffman: Hematology: Basic Principles and Practice, 5th ed.

241
Q

In comparing low molecular weight heparin (LMWH) to standard unfractionated heparin (UFH), which of the following statements is MOST correct?

a) LMWH exerts its activity via binding with thrombin and antithrombin III
b) LMWH therapy requires hospital admission in order to determine the appropriate dose
c) There is an increased rate of bleeding complications associated with LMWH therapy
d) LMWH has a shorter half-life
e) There is a decreased incidence of heparin-induced thrombocytopenia and thrombisis associated with LMWH therapy

A

There is a decreased incidence of heparin-induced thrombocytopenia and thrombisis associated with LMWH therapy

A is partially true. LMWH does bind both but to a lesser extent than heparin does cause the chains are shorter (his arms are too short!!)
The binding of LMWH to antithrombin causes less inactivation of coagulation factor IIa (thrombin) when compared with that of unfractionated heparin.[9] This is because inactivation of factor IIa requires the formation of a ternary complex in which heparin binds to both antithrombin and to a binding site on factor IIa (Figure 2).[17] This ternary complex is formed on pentasaccharide containing chains of at least 18 saccharide units. However, unlike UFH, many of the LMWH do not contain sufficient saccharide units to form the ternary complex

Low-molecular-weight heparins (LMWHs) derived from UFH have more selective anti-Xa activity than UFH does. LMWH has been associated with less bleeding complications and has become the first-line therapy for prophylaxis and treatment of deep venous thrombosis (DVT) and acute coronary syndromes. LMWH can also cause HIT (

242
Q

Half life of SQ LMWH is:

a. 2-4 hr
b. 4-6 hr
c. 6-8 hr

A
Enoxaparin
   Half-life = 4.5 hours
   Duration = 12 hours
Dalteparin
   Half-life = 3-5 hours
- Medscape
243
Q

A 70 y old lady is set to undergo elective surgery for bilateral carpal tunnel release. She has rheumatoid arthritis and a history of mitral regurgitation, as well as intermittent atrial fibrillation. What would be your plan for coagulation management of this patient leading up to surgery: Pt is not high risk. A fib with CHADS2 – 0

a) Stop coumadin 5 days before the surgery
b) Stop the coumadin 5 days prior to surgery and administer LMWH 3 days prior to surgery
c) Stop the coumadin the day of the surgery and administer vitamin 5
d) Stop the coumadin the day of the surgery and administer fresh frozen plasma

A

Stop the coumadin 5 days prior to surgery and administer LMWH 3 days prior to surgery

244
Q

Patient is on coumadin for a heart valve, best pre-op management

a) 7 days pre-op stop coumadin, start on full dose LMWH holding 12h hrs before surgery
b) 4 days pre-op stop coumadin, allow INR to normalize, start prophylactic dose heparin
c) stop coumadin the day before surgery, give vitamin K
d) allow INR to drop to 1.3-1.5 and operate

A

7 days pre-op stop coumadin, start on full dose LMWH holding 12 hrs before surgery

245
Q

Patient on warfarin for mechanical valve has hip fracture. What is the best course of action:

a. Stop warfarin
b. Stop warfarin and start heparin infusion
c. Continue warfarin and start heparin S/C
d. Stop warfarin and start heparin S/C
e. Continue warfarin

A

Stop warfarin and start heparin infusion

  1. 2.5. Bridging Therapy in Patients With Mechanical Valves Who Require Interruption of Warfarin Therapy for Noncardiac Surgery, Invasive Procedures, or Dental Care Class I
  2. In patients at low risk of thrombosis, defined as those with a bileaflet mechanical AVR with no risk factors,* it is recommended that warfarin be stopped 48 to 72 h before the procedure (so the INR falls to less than 1.5) and restarted within 24 h after the procedure. Heparin is usually unnecessary. (Level of Evidence: B)
  3. In patients at high risk of thrombosis, defined as those with any mechanical MV replacement or a mechanical AVR with any risk factor, therapeutic doses of intravenous UFH should be started when the INR falls below 2.0 (typically 48 h before surgery), stopped 4 to 6 h before the procedure, restarted as early after surgery as bleeding stability allows, and continued until the INR is again therapeutic with warfarin therapy. (Level of Evidence: B)
    - ACC/AHA 2006 guidelines for the management of patients with valvular heart disease: A report of the American College of Cardiology/American Heart Association Task Force on Practice Guideline
246
Q
Coumadin affects all of the following coagulation factors EXCEPT:
A. II
B. VII
C. VIII
D. IX
A

C. VIII

247
Q
Heparin does all of the following EXCEPT:
A. Reduces available thrombin
B. Potentiates antithrombin III activity
C. Decreases thromboxane A2 production
D. Decreases platelet aggregation
A

Answer: Decreases thromboxane A2 production

Unfractionated heparin effects anticoagulation by enhancing antithrombin activity, increasing factor Xa inhibitor activity, and inhibiting platelet aggregation.
- Gabbe: Obstetrics: Normal and Problem Pregnancies, 5th ed.

248
Q

Oncotic pressures between intravascular and extravascular compartments is best determined by?

a) Intravascular Na
b) Extravascular Na
c) Intravascular protein
d) Extravascular protein

A

C. intravascular protein
When they say oncotic pressure, this mean colloid-osmotic pressure (which does not include electrolytes!!)

By far the most important for osmotic pressure is intravascular Na

249
Q
What creates the osmotic forces between the intravascular and extravascular compartments of the extracellular fluid volume?
A. Intravascular sodium content
B. Intravascular protein content
C. Extravascular sodium content
D. Extravascular protein content
A

A. Intravascular sodium content

Osmotic pressure determined by intravascular Na - don’t get mixed up with oncotic.

Oncotic refers to the colloid-portion of the osmotic pressure, which is less than the osmotic pressure from Na.

EXCEPTION:

At the level of capillaries, the main pressure is the oncotic pressure because the membrane is readily permeable to ions. Therefore the only except where the osmotic pressure is determined by protein (albumin mostly) is at the level of the capillaries.

250
Q

What is the BEST fluid to use to resuscitate a 6 week old infant with moderate dehydration from vomiting in the setting of pyeloric stenosis?

a. ARL
b. NS
c. ½ NS
d. D51/2ns+30 kcl

Also know the 4:2:1 rule for maintenance fluids - there will be a question with a kg and different rate options

A

In cases of clinical dehydration, children with pyloric stenosis require rehydration with IV fluid therapy before surgery. Administer D5W with 0.45% NaCl IV at 1.5 times the maintenance rate. Severely dehydrated children should receive initial deficit fluid therapy with 0.9% NaCl.When urine output is demonstrated, KCl 10-20 mEq/L can be added to the fluids.

251
Q

Pediatric patient with non bilious vomiting. An olive mass in abdominal exam what would be the electrolyte abnormality:

a. Low K, Low Cl, metabolic alkalosis
b. Low K, Low Cl, metabolic acidosis
c. Hi K, low Cl, metabolic acidosis
d. Low K, hi Cl metabolic alkalosis

A

Answer: Low K, Low Cl, metabolic alkalosis
Olive = Pyloric stenosis

Paradoxically urine is acid because K is so depleted that kidney shifts to H+ excretion.

Labs: hypochloremic, hypokalemic metabolic alkalosis. Persistent emesis causes progressive loss of fluids rich in hydrochloric acid, which causes the kidneys to retain H+ ions in favor of potassium. May also have elevated unconjugated bilirubin.
Paradoxic acid urea.

252
Q
A 6 week old male is seen in the ER with a history of projectile vomiting for the previous 48 hours.  He had weighed 4 kg last week when he was seen at his well baby check.  Now the child weighs, 3.6kg and has a flattened anterior fontanelle. On abdo exam, a palpable olive sized mass is detected in the right of midline in the RUQ.
A capillary gas and electrolytes are ordered the most likely pattern would be:
	Na	K	Cl	HCO3
a.	135	3.5	103	14
b	135 	3.0 	95	20
 c.	130	2.5	88	28
 d.	135	2.5  	112	 30
 e.	135	2.0	90	 20
A

Answer: c. 130 2.5 88 28

In gastric losses, loss of Na, Cl, K, but not bicarb

253
Q

A patient experiences vomiting, what would you use for resuscitation?

a. NS
b. dilute HCl
c. Dextran

A

a. NS

254
Q

When you administer 1L of normal saline you:

a) distribute equally the volume between all compartments
b) pull fluid from the extravascular space intravascularly
c) increase intravascular volume
d) increase intracellular volume

A

C. Increase intravascular volume

  • NS will stay extracellular, approx ¾ interstitial and ¼ intravascular.
  • NS does not go into the ICF
  • Dextrose is distributed evenly among all compartments
  • Plasma protein solutions stay intravascular

A would be a dextrose
B would be a colloid
D would be a hypotonic solution?

255
Q

1L of RL will increase intravascular volume by:

a. 1L
b. 500cc
c. 250cc
d. 100cc

A

25% of RL or NS will go intravascular

256
Q

What are the fluid requirements for a 15 kg child:

a) 900 cc/d
b) 1000 cc/d
c) 1200 cc/d
d) 1500 cc/d

A

1200cc/day
4:2:1
4x10, 2x5 = 50cc/h
50cc/h*24h=1200cc

257
Q

What rate should you bolus a dehydrated child:

a) 10 ml / h
b) 20 ml / h
c) 30 ml / h
d) 40 ml / h

A

b. 20cc/h

Restore cardiovascular stability with a rapid bolus of 20mL/kg over 10 to 30minutes
- Bope: Conn’s Current Therapy 2010, 1st ed.

The child is given a fluid bolus, usually 20 mL/kg of the isotonic fluid, over approximately 20 min.
- Kliegman: Nelson Textbook of Pediatrics, 18th ed.

258
Q

The most water content can be seen in:

a) 60 y old man
c) 60 y old woman

A

Answer: A.
Muscle has more water
Old people have less water

Seems like babies have the most water content

Water is the most abundant component in the body. Total body water (TBW) accounts for approximately 50% of the lean body weight in women and 60% in men. In children, the contribution of the TBW to weight varies with age and constitution: it ranges from 80% or greater in the premature infant, to 70% to 75% in the term infant, to 65% to 70% in toddlers.
- Miller: Miller’s Anesthesia, 7th ed.

259
Q
A 6 year old child presents with 25% dehydration. The FIRST clinical sign you would expect to see is:
A. Tachycardia
B. Hypotension
C. Decreased urinary output
D. Moist mucus membranes
A

C. Decreased UOP. 25% dehydrate is severe dehydration

Clinical Evaluation of Dehydration
Mild dehydration (1.5 sec); cool and pale
Severe dehydration (>10% in an infant; >6% in an older child or adult): rapid and weak or absent peripheral pulses; decreased blood pressure; no urine output; very sunken eyes and fontanel; no tears; parched mucous membranes; delayed elasticity (poor skin turgor); very delayed capillary refill (>3 sec); cold and mottled; limp, depressed consciousness
- Kliegman: Nelson Textbook of Pediatrics, 18th ed.

260
Q

What is the content of RL:

a. Na 154, K0, Cl 113, HCO3 0 Ca 2.7
b. Na 130, K4, CL 109, HCO3 28 Ca 2.7
c. Na 135, K0, CL 103, HCO3 26 Ca 2.7

A

b. Na 130, K4, CL 109, HCO3 28, Ca 2.7

Memorize these numbers. Often asked as a “all of these are ringers’ numbers except”

261
Q
Which of the following has the highest sodium content?
 A – ringers lactate 
 B - Extra cellular fluid
 C – D5W
 D – Ringers lactate
A

B. ECF

NS & D5NS > ECF > RL > D5½NS > 2/3 1/3

262
Q

Major intracellular cations

a. Na + Ca (extracellular cations)
b. K + Mg
c. PO and protein (intracellular anions)
d. Cl and HCO3 (extracellular cations)

A

B. K+Mg

Tip: Banana floating in a sea. Or Caleb’s “Potassium and magnesium are good for you, so you want them in the cell” !

a. Na + Ca (extracellular cations)
b. K + Mg
c. PO and protein (intracellular anions)
d. Cl and HCO3 (extracellular cations)

263
Q

what is the plasma volume of 70kg male patient:

a. 3L
b. 4L
c. 2L
d. 2.5L

A

60% of body weight is water in male = 42kg of water
2/3 of that is intracellular = 28kg
Therefore 14 L of extracellular
1/4 of that is intravascular= 3.5L

Tip: remember YOUR plasma volume and then just guess based off that on a whim

Total Body Water = 60% of Body Weight (For Male) Total Body Water = 50% of Body Weight (For Female)
Extracellular Fluid = Total Body Water / 3
Interstitial Fluid = Total Body Water * 0.25
Plasma = Extracellular Fluid - Interstitial Fluid

264
Q

Pregnant lady has hypotension in general anesthetic.
R lateral decubitus, Dopamine, IV fluids,

Which of the following maneuvers is used in management of the pregnant patient in the OR?

a) Change to right lateral decubitus
b) IV Fluids
c) Reverse Trendelenburg
d) Left lateral decubitus

A

D) left lateral decubitus

In the later stages of pregnancy, as already described, uterine compression of the vena cava may result in hypotension from diminished venous return, thus the pregnant trauma patient needs to be placed in a left lateral decubitus position
- Townsend: Sabiston Textbook of Surgery, 18th ed.

265
Q

Laryngeal mask Vs. endotracheal tube, which of the following is true:

a) the laryngeal mask has less incidence of refux/aspiration
b) the endotracheal tube is techinically easy to insert
c) the endotracheal tube, although technically demanding to insert, offers the advantage of delivering medications through he endotracheal tube if needed

A

Canadian paper.
Advantages over the TT included: increased speed and ease of placement by inexperienced personnel; increased speed ofplacement by anaesthetists; improved haemodynamie stability at induction and during emergence; minimal increase in intraocular pressure following insertion; reduced anaesthetic requirements for airway tolerance; lower frequency of coughing during emergence; improved oxygen saturation during emergence; and lower incidence of sore throat in adults. Advantages over the FM included: easier placement by inexperienced persornel; improved oxygen saturation; less hand fatigue; and improved operating conditions during minor paediatric otological surgery

Disadvantages over the TT were lower seal pressures and a higher frequency of gastric insufflation. The only disadvantage compared with the FM was that oesophageal reflux was more likely

No difference in the risk of aspiration b/c not enough studies, but with ETT increase complications of voice hoarness, cough, and laryngeal spasms (see article)

266
Q
What does propofol do?
 a-decrease cerebral blood flow
 b-Significant N/V
 c-increases blood pressure
 d-broncho-constriction
A

Answer: decreases cerebral perfusion pressure

Propofol decreases intracranial pressure (ICP) in patients with either normal or increased ICP.[79] The decrease in ICP (30% to 50%) is associated with significant decreases in cerebral perfusion pressure (CPP), however.
- Miller: Miller’s Anesthesia, 7th ed.

267
Q
A patient was placed in a left lateral decubitus position for total hip arthroplasty which lasts over 4 hours. Post operatively he wakes up complaining of arm/hand weakness. What is the MOST likely abnormality?
A. Weak finger flexion 
B. Weak wrist extension 
C. Weak finger adduction 
D. Weak finger abduction
A

? wrist extension (radial nerve)

268
Q

Bier Regional of the upper Limb block, all true EXCEPT:

a. Systemic toxicity happens 30 minutes after removal of tourniquet.
b. Cannot obtain reasonable muscle relaxation.
c. Pain at the site of tourniquet limits its use.
d. Loss of analgesic effect will happen immediately after removal of the tourniquet.

A

Answer: Systemic toxicity happens 30 minutes after removal of tourniquet.

Loss of anesthesia after cuff deflation may occur. If the surgeon wants to attain hemostasis and then close the wound, there is only 5 to 10 minutes of post-deflation analgesia, which may be inadequate in some procedures.

When the tourniquet is released, a large bolus of anesthetic enters the systemic circulation; the level peaks within 1 minute and falls rapidly thereafter. This brief elevation of the local anesthetic blood level may produce systemic toxic reactions, including convulsions and cardiac irregularities

269
Q

Halothane causes:

a. Hepatitis

A

Morrell
• MAC 0.75
• Almost completely replaced by newer agents.
• Powerful cardiac depressant (↓ contractility and SV, HR unchanged → overall ↓ CO)
•Inhibits baroreceptor-increased HR usually seen with lowered BP.
• Sensitizes myocardium to catecholamines.
•Associated with fulminant hepatitis.
•Suitable for mask induction.

270
Q

Which of the following is not correct regarding anesthesia in children:

a. does not require preoperative testing in the majority of cases
b. preoperative evaluation includes an examination of throat and ears
c. the presence of parents up until OR time increases separation anxiety
d. is often helped by premedication
e. requires discussion with the child

A

Answer: the presence of parents up until OR time increases separation anxiety

it decreases it

271
Q

A patient receives spinal anesthesia. The surgeon requests the patient be placed in the Trendelenburg position and the patient becomes hypotensive. The most likely cause is:

a) anaphylactic shock
b) sympathetic block
c) injection of anesthetic into a spinal artery
d) aorta/IVC injury
e) none of the above

A

Answer: sympathetic block

Complications of subarachnoid block include hypotension (sometimes refractory), postdural puncture headache, transient radicular neuropathy, backache, urinary retention, infection, epidural hematoma, and excessive cephalad spread resulting in cardiorespiratory compromise. Frank neurologic injury, although recently described with continuous techniques using small-bore catheters, is rare. Hypotension, occurring as a consequence of sympathectomy, usually responds readily to fluids and small doses of pressors, such as ephedrine.
- Townsend: Sabiston Textbook of Surgery, 16th ed., Copyright © 2001 W. B. Saunders Company

Pharmacologic sympathectomy with local anesthetics is a very effective way of inducing hypotension. Epidural or spinal anesthesia produces arteriolar and venous dilation and hypotension. These effects are enhanced by a pooling of blood in the venous system that decreases venous return and cardiac output. If the block is extended to the midthoracic region, sympathetic innervation of the heart (T1-T4) is also affected, thereby preventing compensatory tachycardia. [37] The unpredictable degree of hypotension and the necessity for large infusions of fluids are the principal drawbacks of this technique. It was recently demonstrated, however, that if hemodynamic stability is maintained by intravenous infusion of low-dose epinephrine (1-5 mug/min), this technique can be used safely. [25] [35] An epidural anesthetic technique is most commonly used to minimize blood loss during lower abdominal or pelvic surgery.
- Miller: Anesthesia, 5th ed., Copyright © 2000 Churchill Livingstone, Inc.

272
Q
Malignant hyperthermia is characterized by:
A. Early hyperthermia
B. Autosomal dominant transmission
C. Late increased end-tidal CO2
D. Hypokalemia
A

B. autosomal dominant

Half of cases are autosomal dominant; others are unknown.
Diagnosis rests on acute awareness of the signs and symptoms of this syndrome, of which hyperthermia is a late sign.
The increased permeability of skeletal muscle results in increased serum levels of potassium, ionized calcium, CK (although MH-related changes in serum CK do not differ overall from changes in CK observed during surgery[236]), myoglobin, and serum sodium.
- Miller: Miller’s Anesthesia, 7th ed.

Miller describes a variable pattern in C02 rise. Also early vs late hyperthermia … there is no hard and fast rule that says early C02 rise and late hyperthermia. But B is likely the POS answer.

Early clinical signs of MH include an increase in end-tidal carbon dioxide (even with increasing minute ventilation), tachycardia, muscle rigidity, tachypnea, and hyperkalemia. Later signs include fever, myoglobinuria, and multiorgan failure.

273
Q

3) All the following medications can trigger malignant hyperthermia EXCEPT:
a. Propofol.
b. Succinyl choline
c. Enflurane.
d. Sevoflurane

A

a. Propofol

Triggering Agents

* Depolarizing muscle relaxants (eg, succinylcholine)
* Volatile general anesthetic agents (eg, ether, cyclopropane, methoxyflurane, halothane, isoflurane, enflurane, sevoflurane, and desflurane)

Safe (nontriggering) Agents

* All nondepolarizing muscle relaxants (eg, vecuronium, rocuronium, d-tubocurarine, pancuronium, atracurium, cis-atracurium, and mivacurium)
* Nitrous oxide
* Intravenous anesthetics (eg, ketamine, propofol, etomidate, thiopental, opiates, and benzodiazepines)
* All ester and amide local anesthetics  - http://cme.medscape.com/viewarticle/491434

Saw in some older literature that local anesthetics were thought to be a cause, but this has been debunked (they do not cause it)

274
Q

The treatment of malignant hyperthermia involves all of the following except:

a) cooling blankets
b) Oxygen
c) Dantrolene
d) Diltiazem

A

d. diltiazem

Acute therapy for MH can be summarized as follows: (1) Discontinue all anesthetic agents and hyperventilate with 100% oxygen. Normal ventilation is that required to remove metabolic carbon dioxide. With increased aerobic metabolism, normal ventilation must increase. However, carbon dioxide production is also increased because of neutralization of fixed acid by bicarbonate; hyperventilation removes this additional CO2. (2) Administer dantrolene (2.5 mg/kg intravenously [IV] to a total dose of 10 mg/kg IV) every 5 to 10 minutes until symptoms subside. (3) Administer bicarbonate (2 to 4 mEq/kg IV) to correct the metabolic acidosis with frequent monitoring of blood gases and pH. (4) Control fever by administering iced fluids, cooling the body surface, cooling body cavities with sterile iced fluids, and if necessary, using a heat exchanger with a pump oxygenator. Cooling should be halted at 38°C to 39°C to prevent inadvertent hypothermia. (5) Monitor urinary output and establish diuresis to protect the kidney from probable myoglobinuria. (6) Further therapy is guided by blood gases, electrolytes, temperature, arrhythmia, muscle tone, and urinary output. Treatment of hyperkalemia with glucose and insulin should be slow. The most effective way to lower serum potassium is reversal of MH by effective doses of dantrolene. (7) Analyze coagulation studies.
- Miller: Miller’s Anesthesia, 7th ed.

275
Q
Which drug should not be used in people with altered metabolism of acetyl cholinesterase at the motor endplate
 a – Pancuraonium
 b – Rocuronium
 c – Succinylcholine
 d – Halothane
A

Answer: Succinylcholine

276
Q

Patient for resection of facial basal cell carcinoma, states she is allergic to local anesthetic, how should management proceed?

a) Do under general anesthetic
b) Use marcaine (bupivacaine) with epinephrine
c) Use preservative free marcaine with epinephrine
d) Use procaine

A

d. preservative free marcaine with epinephrine

More likely to be allergic to ester which produced a PABA metabolite (procaine), or preservative (MPB) so give neither of those.

This stem is probably incorrect. There can be preservatives in epinephrine. The LEAST allergenic is preservative-free amide (ex: plain lidocaine)

Amino ESTERS - cocaine, tetracaine, benzocaine, procaine (1 “i”)
Amino AMIDES - lidocaine, bupivacaine, prilocaine, mepivicaine (2 “i”s) bupivicane (bupivicaine is also trademarked marcaine, which only has 1 i so you need to know this exception)

More info:

True allergic reactions are rare, accounting for only 1% to 2% of all adverse reactions, but they are important to recognize because of their serious potential. Ester solutions (procaine, tetracaine), which produce the metabolite para-aminobenzoic acid (PABA), account for the great majority of these reactions. Amide solutions (lidocaine, bupivacaine) are rarely involved, and usually the preservative methylparaben (MPB), which is structurally similar to PABA, is responsible.
Generally speaking, the optimal approach to the patient with a presumed anesthetic allergy is to determine the specific anesthetic agent associated with a presumed allergic reaction and then use a preservative-free agent from the other class (see earlier discussion). If the agent is unknown, use an antihistamine or give 0.1 mL of preservative-free lidocaine as a subcutaneous test dose, proceeding with the full dose if no reaction occurs within 30 minutes. Given the studies mentioned earlier, the prudent choices would seem to be diphenhydramine (Benadryl) or benzyl alcohol. Epinephrine (1 : 100,000) can be added to both of these drugs to prolong the duration of action. Ketamine anesthesia is also an alternative.
- Roberts: Clinical Procedures in Emergency Medicine, 5th ed.

Allergy to lidocaine is very rare. Most patients who claim to be allergic have had a vasovagal response. Bacteriostatic saline is an alternative for patients who are allergic to lidocaine. Commercially available bacteriostatic saline contains benzyl alcohol, which acts as a painless anesthetic. The anesthetic effect dissipates rapidly when injected subcutaneously. The volume of saline required to achieve anesthesia is at least two to three times that required when using 1% lidocaine and is of brief duration.
- Habif: Clinical Dermatology, 5th ed.

If an allergic reaction cannot be ruled out and the drug previously used is known, use an agent from the other class (whether amide or ester).
- Roberts: Clinical Procedures in Emergency Medicine, 5th ed.

277
Q

2) A 4 year old presents to the ER with a scalp laceration. After examining and cleaning the area, you are going to close the laceration. Assuming he is 25 kg, you will use which of the following doses of local anesthetic:
a) 10 ml of 1% xylocaine
b) 10 ml of 2% xylocaine
c) 30cc of 1% xylocaine with epinephrine
d) 15 ml of 0.5% marcaine
e) 25 ml of 0.5% marcaine with epi

A

a) 10cc 1% xylocaine (lidocaine)

lidocaine with epi : 5mg/kg
lidocaine without epi: 7mg/kg
bupivicaine : 2-3 mg/kg

% means % weight so 1% xylocaine is 0.01kg/L = 10g/L =10mg/mL

1 mL = 10 mg

278
Q

CNS lidocain side effect all are correct except:

a. Seizure
b. Nystagmus
c. Confusion
d. Hemiparesis

A

d. hemiparesis

  • Low levels → depression, lightheadedness, dizziness, circumoral numbness, metallic taste, visual disturbance, tinnitus & slurred speech
  • High levels → excitation, anxiety, nystagmus, fine tremors: hand and face, seizures
  • Very high levels →depression, coma, respiratory depression and arrest
  • Cocaine – euphoria, sense of well being

Major Side Effects
Side effects are proportional to the plasma concentration of lidocaine. At low concentrations, light-headedness, numb tongue, metallic taste, increased blood pressure, and dizziness can occur. As the plasma concentration increases, visual and auditory disturbances, confusion, muscle twitching, and decreased blood pressure develop. At high plasma concentrations, patients are at risk for arrhythmias, convulsions, coma, and respiratory and cardiovascular system collapse. Hypersensitivity has also been reported.
- Walsh: Palliative Medicine , 1st ed.

279
Q

What system is most commonly affected by lidocaine toxicity?

a) Cardiovascular
b) Neurological
c) Respiratory
d) Musculoskeletal
e) Renal

A

Answer: Neurological

  • Initially CNS (excitation followed by depression) then CVS collapse at higher level
  • Earliest toxic effect circumoral paresthesia, numbness of the tounge/metallic taste, dizziness,hyperacusis and tinnitus
280
Q

Lidocaine needs to be adjusted in the case where:

a. Decreased with hepatic dysfunction
b. Decreased with renal dysfunction
c. Increased dose with CHF
d. Increase dose with rifampin
e. Increase dose with other local aesthetics

A

Answer: Decreased with hepatic dysfunction

Ninety percent of a dose of lidocaine is metabolized in the liver via N-dealkylation. Urinary excretion is 10%.
- McPherson & Pincus: Henry’s Clinical Diagnosis and Management by Laboratory Methods, 21st ed.

Lidocaine is chiefly metabolized in the liver via de-ethylation. The half-life is prolonged in patients with liver but not renal failure.
- Walsh: Palliative Medicine , 1st ed.

281
Q

Of the following local anesthetics, which IS NOT an ester?

a) procaine
b) lidocaine
c) tetracaine
d) cocaine
e) all of the above

A

Esters

  • Procaine
  • Chloroprocaine
  • Cocaine
  • Tetracaine

Amides (have an “i” early in the name)

  • Lidocaine
  • Mepivacaine
  • Prilocaine
  • bupivacaine (Marcaine)
  • etidocaine
282
Q

A child has a seizure after injection of 1% lidocaine during an inguinal hernia repair. The best initial treatment is:

a) diazepam
b) midazolam
c) magnesium
d) propofol
e) none of the above

A

E. None of the above.

Airway

Treatment:
• Airway maintenance
• Hyperventilation →CO2 reduces seizure threshold
• Diazepam or thiopental Na for seizures
• Fluid and vasopressors for cardiac depression

283
Q

Which drug has the longest action:

a) mepivacaine
b) bupivocaine
c) lidocaine
d) procaine

A

b. bupivocaine

mepivacaine (120m with epi)
bupivocaine (180-240 with epi)
lidocaine (120m with epi)
procaine (30-45m with epi)
 - Source: Miller: Miller's Anesthesia, 7th ed.
284
Q
The MOST appropriate adjuvant analgesic for treating somatic pain is:
 A. Amytriptyline (Elavil)
 B. Clonidine (Catapres)
 C. Ibuprofen (Motrin)
 D. Lorazepam (Ativan)
 E. Gabapentin (Neurontin)
A

Answer: Ibuprofen (Motrin)

285
Q

Control of chronic pain associated with cancer:

a. NSAID’s
b. Meperidine
c. Parenteral morphine
d. Talwin when used inceases the effect of narcotics

A

Answer: NSAID’s can help in controling the pain.

a. NSAID’s (Yes)
b. Meperidine (not recommended)
c. Parenteral morphine (oral is preferred)
d. Talwin when used inceases the effect of narcotics (not on the WHO ladder)

286
Q

The use of NSAID in the preoperative period is associated with:

a. Decreased gastric emptying.
b. Decreased post operative narcotic requirement.
c. Increased post operative blood transfusion.

A

Answer: Decreased post operative narcotic requirement.

Think of pre-op celebrex in our arthroplasty patients

287
Q

An 87 year old ♀ with advanced osteoporosis has chronic back and hip pain, poorly controlled on 2 Percocet (5 mg oxycodone and 325 mg acetaminophen each) six times per day. The single best reason not to increase the number of Percocet tablets is:
A. Non-steroidals (NSAID’s) are the best drugs for bone pain
B. Oxycodone is a weak opioid
C. Oxycodone is contraindicated in the elderly
D. The dose of acetaminophen would exceed the recommended level
E. The maximal dose of oral oxycodone is 40 mg per 24 hours

A

Answer: The dose of acetaminophen would exceed the recommended level

288
Q

A 60 year old man with carcinoma of the colon is started on morphine for abdominal pain. On review the next day, his family reports that he has been having hallucinations. You would:
A. Rotate to hydromorphone
B. Rotate to fentanyl as it has less metabolites
C. Use haloperidol
D. Use a benzodiazepine
E. Avoid opioids

A

Avoid opioids, and if you need them, hydromorphone is better in elderly (less delirium)

 A. Rotate to hydromorphone (maybe) 
 B. Rotate to fentanyl as it has less metabolites (no)
 C. Use haloperidol (no)
 D. Use a benzodiazepine (no)
 E. Avoid opioids (probably yes)
289
Q

Demerol use is limited by what?

Short half life and toxic metabolites

A

Answer: active metabolite toxicity

Does anyone even use this anymore? keep in mind these questions are old - meds seem to have aged the most so this whole section is probably not super high yield

290
Q

The BEST way to administer analgesia post operatively in a morbidly obese female post thoracotamy is

a. continuous IV narcotics
b. nurse gives IV bolus narcotics prn
c. nurse gives IM narcotics PRN
d. epidural analgesia
e. background PO narcotics with nurse giving IM narcotics for break through pain

A

Answer: epidural

A meta-analysis of respiratory complications after various types of surgery has shown that epidural techniques reduce the incidence of respiratory complications.[252] Lumbar epidural analgesia has gradually been replaced by thoracic epidural analgesia for thoracic surgery in the 1990s.
- Miller: Miller’s Anesthesia, 7th ed.

Two very efficacious methods of post-thoracotomy pain treatment have become widely used and are considered by many to be the methods of choice. They are cryoanalgesia and administration of epidural narcotics. In addition, interpleural regional analgesia has recently been introduced as an alternative method for treating pain after thoracotomy.
- Miller: Anesthesia, 5th ed., Copyright © 2000

291
Q

The MOST important supplemental therapy to consider when starting patients on opioids for pain is:
A. Amphetamines to increase alertness
B. Antidepressants to supplement pain relief
C. Antiepileptic medications to treat neuropathic pain
D. Antiemetics to counteract nausea
E. Non-steroidals (NSAID’s) to treat inflammation

A

D. Antiemetics to counteract nausea

Or laxatives (if it was an answer choice)

292
Q

With regards to PCA which of the following is true:

a. Causes respiratory depression
b. SC and IM are better than PCA
c. Causes more addiction
d. All of the above
e. None of the above

A

Answer: Causes respiratory depression

I think depending on the question they may be wanting to see if you know that PCA are less likely to cause respiratory depression because you stop pressing the button when you are comfortable/sleepy (unless someone else is pushing the button for you!)

Intravenous PCA is based on the premise that a negative-feedback loop exists; when pain is experienced, analgesic medication is self-administered, and when pain is reduced, there are no further demands. When the negative-feedback loop is violated, excessive sedation or respiratory depression may occur.
- Miller: Miller’s Anesthesia, 7th ed.

293
Q

What agent also has an analgesic effect?

a. Ketamine
b. Midazolam
c. Propofol
d. 2 more anesthetic agents (can’t remember names)

A

Ketamine has a wide range of effects in humans, including analgesia, anesthesia, hallucinations, elevated blood pressure, and bronchodilation. Ketamine is primarily used for the induction and maintenance of general anesthesia, usually in combination with some sedative drug. Other uses include sedation in intensive care, analgesia (particularly in emergency medicine), and treatment of bronchospasm.
- http://en.wikipedia.org/wiki/Ketamine

294
Q

Most appropriate pain control post-op appendectomy in a 4 year old 15kg boy

a. codeine 15mg q4h po
b. meperidine 10mg q3h IM
c. acetaminophen 160mg q4h po
d. morphine 1.5 mg q3h IV
e. PCA

A

Answers given:

  • morphine 1.5 mg IV q3h = Immediate
  • Codeine 15 mg po q4h prn = Slow release (who uses codeine anymore? doubt this will be in POS post-medieval ages)

Both of these drugs could be used as effective pain control in the peds pop.

There is no fixed age below which PCA is not offered; however, patients below the age of 7 years generally are poor candidates for PCA.

Acetaminophen 160mg po q4h: Insufficient dose – need 240mg - Infants and children

295
Q

Horner’s syndrome includes all except:

a. Anhydrosis
b. Increased lacrimation
c. Miosis
d. Ptosis

A

Answer: Increased lacrimation

The clinical features of Horner’s syndrome can be remembered using the mnemonic, “Horny PAMELa” for Ptosis, Anhydrosis, Miosis, Enophthalmos and Loss of ciliospinal reflex.

Ptosis is an abnormally low position (drooping) of the upper eyelid.
Anhydrosis means lack of sweating.
Miosis is constriction of the pupil of the eye.
Enophthalmos (Also Endopthalmos) is recession of the eyeball within the orbit.
The ciliospinal reflex (pupillary-skin reflex) consists of dilation of the ipsilateral pupil in response to pain applied to the neck, face, and upper trunk.

296
Q

Distraction injury of arm. Horners injury is most likely to be secondary to injury to

a. C5
b. C6
c. C7
d. C8
e. T1

A

Upper arm, lower arm, and hand are involved. Horner syndrome (ptosis, anhydrosis, and miosis) exists if T1 is involved.
- Johns Hopkins: The Harriet Lane Handbook, 18th ed.

297
Q

Autonomic dysreflexia is explained by:
a-A lesion BELOW T6
b-increased parasympathetic reflexes
c-increased spinal reflexes despite the injury

A

None of the above

Autonomic dysreflexia (AD), also known as autonomic hyperreflexia, is a potentially life threatening condition which can be considered a medical emergency requiring immediate attention. AD occurs most often in spinal cord-injured individuals with spinal lesions above the T6 spinal cord level. Acute AD is a reaction of the autonomic (involuntary) nervous system to overstimulation. It is characterised by severe paroxysmal hypertension (episodic high blood pressure) associated with throbbing headaches, profuse sweating, nasal stuffiness, flushing of the skin above the level of the lesion, bradycardia, apprehension and anxiety, which is sometimes accompanied by cognitive impairment.[1] The sympathetic discharge that occurs is usually in association with spinal cord injury (SCI) or disease (e.g. multiple sclerosis). AD is believed to be triggered by afferent stimuli (nerve signals that send messages back to the spinal cord and brain) which originate below the level of the spinal cord lesion. It is believed that these afferent stimuli trigger and maintain an increase in blood pressure via a sympathetically mediated vasoconstriction in muscle, skin and splanchnic (gut) vascular bed

298
Q
Which returns latest and LEAST completely after repair of a severed nerve?
A. Pin prick sensation
B. Proprioception
C. Pain
D. 2 point discrimination
A

1st change is decrease in threshold for vibration, thus hypersensitivity to tuning fork.
2 point discrimination is the last sensory modality to go.

299
Q

Which investigation should be the first one ordered on a patient 5 days post-operatively who now complains of their worst headache ever.

a. Lumbar puncture
b. MRI
c. CT
d. Skull x rays

A

CT

300
Q

All of the following findings are associated with a diabetic patient who is hypotensive, tachycardic in coma except:

a. Ketonemia
b. High glucose
c. Leukocytosis
d. Hyperkalemia
e. Uraemia

A

e. Uraemia

DKA.

a. Ketonemia -yes
b. High glucose -yes
c. Leukocytosis -yes Leukocytosis — The majority of patients with hyperglycemic emergencies present with leukocytosis, which is proportional to the degree of ketonemia [6,61]. Leukocytosis unrelated to infection may occur as a result of hypercortisolemia and increased catecholamine secretion
d. Hyperkalemia - ???
e. Uraemia - ???

301
Q

What is the most determinant factor of cerebral blood flow:

a. Arterial PO2
b. Arterial PCO2
c. Intracranial pressure
d. Heart rate
e. Respiratory rate

A

Answer: c. Intracranial pressure

CPP=MAP-ICP
Cerebral autoregulation maintains constant blood flow by compensating for changes in CPP, unless
-high ICP such that CPP 160 mmHg or

302
Q

What is not part of cerebral autoregulation

a. MAP
b. H+ in brain
c. pH of CSF What is the difference b/w b and c?
d. Smooth muscle cell wall tension across vessel wall

A

??
Autoregulation of cerebral blood flow (CBF) responds to changes in arterial pressure, arterial Pco2, or Po2, by dynamically altering arteriolar caliber so as to maintain a constantly matched cerebral perfusion sufficient to meet metabolic demands ( Fig. 94-3 ). pH, Paco2, potassium, and adenosine are among many purported metabolic mediators of flow.
- Miller: Miller’s Anesthesia, 7th ed.

303
Q

Lady with weakness of right fifth finger flexion and pain along the medial side of her forearm. Dx:

a. Thoracic outlet syndrome
b. Cubital tunnel syndrome
c. Carpal tunnel syndrome
d. Meralgia paresthetica
e. Pronator Syndrome

A

Cubital tunnel syndrome
PHYSICAL FINDINGS & CLINICAL PRESENTATION
- Paresthesias and numbness along distribution of ulnar nerve (ulnar one and one-half fingers)
- Positive Tinel’s sign at elbow
- Positive elbow flexion test (flexion of elbow with wrist extended for 30 sec may reproduce symptoms)
- May be diminished sensation to tip of small finger
- Ulnar nerve may be subluxable with elbow motion or by manipulation
- Cubitus valgus may be present if prior bony injury
- Interosseous weakness in longstanding cases with atrophy
Ferri: Ferri’s Clinical Advisor 2010, 1st ed.

Carpal Tunnel Syndrome
The earliest symptoms are pain and paresthesias, which are characteristically more obvious at night, after prolonged activity, and with positional postural changes at the wrist such as when driving, using a blower hairdryer, or reading a book. The patient may complain of clumsiness and a tendency to drop objects. The paresthesias characteristically follow the distribution of the median nerve, including the thumb and index and middle finger. Physical examination consists of compressing the carpal canal, percussing the median nerve, and hyperflexing the wrist to produce paresthesias (Durkin’s sign, Tinel’s sign, and Phalen’s test, respectively).
Pronator Syndrome
The symptoms produced are similar to those of carpal tunnel, although nocturnal symptoms are uncommon. The palm may also feel numb because the palmar cutaneous branch is involved, but is specifically spared in carpal tunnel syndrome because that nerve branch passes superficial to the flexor retinaculum and arises proximal to the retinaculum.
Cubital Tunnel Syndrome
Motor and sensory symptoms develop in the distribution of the ulnar nerve and are worsened by adopting a flexed position at the elbow. Examination reveals a positive Tinel’s sign over the tunnel. Paresthesias are described in the distribution of the ulnar nerve to the little and ring fingers and ulnar border of the hand.
Thoracic Outlet Syndrome
All elements of the brachial plexus, as well as the subclavian artery and vein, pass through this narrow space and can be potentially compressed at this site.
- Townsend: Sabiston Textbook of Surgery, 18th ed.

304
Q
Commonest cause of stroke post op:
a. hypotension
b. large vessel atherosclerosis
b-cardiac emboli
c- carotid ?
A

??
Postoperative hypertension in particular has been shown to be associated with increased postoperative stroke and death and with postoperative cardiac complications.
- Townsend: Sabiston Textbook of Surgery, 18th ed.

Most strokes develop within the first 2 days after surgery, and are up to two times more common in combined cardiac procedures or technically challenging operations. Multivariable analysis has identified 10 variables that were independent predictors of stroke: history of cerebrovascular disease, peripheral vascular disease, diabetes, hypertension, previous cardiac surgery, preoperative infection, urgent operation, CPB time more than 2 hours, need for intraoperative hemofiltration, and high transfusion requirement.
Patients with ascending aortic atherosclerosis, older age (70 years), preoperative unstable angina, chronic obstructive pulmonary disease, and carotid artery disease are at risk for late postoperative stroke (new strokes during 5-year follow-up) after CABG.
- Libby: Braunwald’s Heart Disease: A Textbook of Cardiovascular Medicine, 8th ed

305
Q

In Sunderland classification of nerve injury. What is correct regarding
grade II:
a. Recovery is expected in days to weeks
b. Complete recovery is expected
c. No wallerian degeneration
d. Regeneration rate is 1cm /day.

A

B. Complete recovery is expected

In a Sunderland type 2 injury, the endoneurium, perineurium, and epineurium are still intact, but the
axons are physiologically disrupted. Because the endoneurium is intact, the regenerating axons are directed along their original course, and complete functional recovery can be expected. The time for recovery depends on the level of injury, as the axon must regenerate distally to the end-organ. It can usually be measured in months, as opposed to weeks for a Sunderland type 1 injury. Injuries to subsequent connective tissue layers upgrade the Sunderland classification.
J Am Acad Orthop Surg 2000;8:243-252

Regeneration is 1mm/day (or 1inch/mo)
Recovery is expected in days to weeks for neuropraxia (Sunderland grade I)

First-degree injury or Neuropraxia: a localized conduction block is produced that results in at most a segment of demyelination. Since the axons are not injured, regeneration is not required and after remyelination, recovery is complete.

Second-degree injury or Axonotmesis: axonal injury occurs and the distal segment undergoes Wallerian degeneration. Proximal nerve fibres will regenerate at a rate of 1mm/day. By definition the connective tissue layers are uninjured. Recovery will be complete unless the distance of the injury from the motor end plate results in such prolonged denervation of the receptor that motor recovery is adversely affected.

Third-degree injury: Wallerian degeneration is combined with some fibrosis of the endoneurium. Recovery will be incomplete because scar within the endoneurium may block or cause mismatching of regenerating fibres with the appropriate end organs.

Fourth-degree injury: A complete scar block results from injury to the axon, endoneurium and perineurium. Regeneration will not occur unless the block is removed and the nerve is repaired or grafted.

Fifth-degree injury or Neurotmesis: the nerve is completely divided at every tissue layer and must be repaired before any regeneration can occur

Sixth-degree injury: this represents a combination of any of the previous five levels of injury. Because of the longitudinal nature of crushing injuries, different levels of nerve injury can be seen at various locations along the nerve.

306
Q

A patient suffers the loss of pain and temperature on their right side, with contralateral motor weakness. The diagnosis is:

a. Brown-Sequard
b. spinal shock
c. central cord syndrome
d. anterior cord syndrome
e. cerebral injury and incomplete cord injury

A

a. Brown-Sequard

307
Q

Patient sustained spinal injury with complete neurological loss below T1 level. After few hours, bulbocavernous reflex returned with no improvement in neurological exam. Likely diagnosis:

a. partial spinal cord injury
b. complete spinal cord injury
c. anterior column injury
d. posterior column injury

A

b. complete spinal cord injury

Return of bulbocavernous reflex = end of spinal shock. Cannot fully diagnose extent of spinal cord injury until spinal shock ends to see what function is left.

Complete Spinal Cord Lesion
• bilateral loss of motor/sensory and autonomic function at ≥4 segments below lesion/injury, with UMN signs
• about 3% of patients with complete injuries will develop some recovery within 24 hours, beyond 24 hours, no distal function will recover

Definition:

  • complete cord injury implies unequivocal absence of motor or sensory function distal to injury in absence of spinal shock;
  • w/ complete injuries, an improvement of one nerve root level can be expected in 80% of patients, and approximately 20% will recover 2 additional function levels;
  • there is also evidence that operative decompression can also allow recovery of an additional nerve root level;
  • incomplete lesion:
  • present when there is any distal sparing of motor or sensory function along with sparing of perirectal sensation;

Anterior Cord Syndrome

  • syndrome is manifested by complete motor paralysis (corticospinal function) & sensory anesthesia (spinothalamic function);
  • there is sparing of the dorsal column:
  • deep pressure and proprioception are only retained sesibility of the trunk and lower extremities;
  • patient demonstrates greater motor loss in the legs than arms;

The bulbocavernosus reflex is a polysynaptic reflex that is useful in testing for spinal shock and gaining information about the state of spinal cord injuries (SCI). The test involves monitoring anal sphincter contraction in response to squeezing the glans penis or tugging on an indwelling Foley catheter.[1] The reflex is spinal mediated and involves S2-S4. The absence of the reflex without sacral spinal cord trauma indicates spinal shock. Typically this is one of the first reflexes to return after spinal shock. Lack of motor and sensory function after the reflex has returned indicates complete SCI. Absence of this reflex in instances where spinal shock is not suspected could indicate a lesion or injury of the conus medullaris or sacral nerve roots.
- http://en.wikipedia.org/wiki/Bulbocavernosus_reflex

308
Q

Patient comes in with a T5-6 fracture dislocation with no motor or sensory function below T6. No perianal sensation, flacid rectal tone and no bulbocavernosus reflex
a – Good prognosis of neurologic recovery
b – no chance of nerologic improvement
c – Can not determine neurologic prognosis
d – patient will walk out of the hospital

A

Answer: Can not determine neurologic prognosis

Must wait for the reflex to return for prognosis.

309
Q

Other than a Randomized Controlled Trial, which study design introduces the least bias?

a) meta-analysis
b) cohort study
c) case-control study
d) longitudinal

A

Answer: RCT > Prospective cohort > Case-control

Case control is retrospective and suffers from recall bias.

Order from least to most bias:
Experimental Studies
1. Randomized Controlled Clinical Trial (RCT):
Observational Studies
1. Cohort (Incidence, Longitudinal Study)
2. Case-Control Study:
3. Ecologic (Aggregate) Study:
4. Cross-Sectional (Prevalence Study) Study:
5. Case Series:
6. Case Report
- http://www.vetmed.wsu.edu/courses-jmgay/GlossClinStudy.htm (great resource)

310
Q

A world health organization epidemiologist is studying esophageal cancer in females. In Canada, 5,000,000 females over the age of 25 years have been followed from January 1, 1980 to January 1, 2000. Within this population, a group of 1,000,000 women chronically exposed to sulphur dioxide fumes are found to have an increased incidence of developing esophageal cancer as compared to the 4,000,000 that were not chronically exposed. The data is shown below:
Group Esophageal Cancer Incidence (per 1,000,000)
Exposed 100
Not exposed 20

The above study is an example of what kind of study?

a) A prospective randomized control study
b) A prospective non-randomized control study
c) A prospective case-control study
d) A prospective cohort study
e) None of the above

A

d) A prospective cohort study

Answer: Cohort study (prospective, incidence, longitudinal): follows a group of persons with common characteristics forward over a period of time and determines outcome according to exposure to different factors
Provides estimates of incidence, relative risk, attributable risk
Cannot by itself, establish causation but can show an association between a factor and an outcome

311
Q

Researchers designed an RCT looking at people with ameurosis fugax. They grouped the patients by degree of carotis stenosis. They found a benefit for enarterectomy in the >80% stenosis group, no benefit in the 50-79% stenosis group. A separate RCT study was published on asymptomatic carotid stenosis showing a benefit in their 60-79% stenosis group. The authors of the first study then decided to do a sub-group analysis to determine if patients with 60-79% stenosis and ameurosis fugax showed benefit from intervention. Why will their sub group results not be valid?

a) Retrospective.
b) Not controlled.
c) Not blinded.
d) Not randomized.

A

’?? bias

312
Q

The best randomization for a study of laparoscopic appendectomy vs. open is:

a. Alternative months
b. Monday ,Wednesday ,Friday laparoscopic and the rest open
c. Coin toss
d. Male to female
e. Odd & even numbers

A

c. Coin toss

everything else could have confounding factors

313
Q

When a test tested what is it is supposed to test is called:

a. Sensitivity
b. Specificity
c. Power
d. Validity

A

d. Validity

314
Q

If you did small RCT and you found difference but is not statistically significant. What should you do:

a. Add more patients
b. Do another study with larger sample
c. Ignore the result
d. ????

A

a. Add more patients

need more POWER

315
Q

A group of surgeons meets to create treatment guidelines for the management of pulmonary embolism. This type of evidence is:

a) Class I
b) Class II
c) Class III
d) Class IV

A

d) Class IV

The following is a description of the rating scale used to evaluate how strongly supported the evidence supports the use of a PARTICULAR THERAPY.

Class I: Prospective, randomized, controlled clinical trial with masked outcome assessment, in a representative population.
The following are required:
a) primary outcome(s) is/are clearly defined
b) exclusion/inclusion criteria are clearly defined
c) adequate accounting for drop-outs and cross-overs with numbers sufficiently low to have minimal potential for bias
d) relevant baseline characteristics are presented and
substantially equivalent among treatment groups or
there is appropriate statistical adjustment for
differences.

Class II: Prospective matched group cohort study in a representative population with masked outcome assessment that meets a-d above OR a RCT in a representative population that lacks one criteria a-d.

Class III: All other controlled trials (including well-defined natural history controls or patients serving as own controls) in a representative population, where outcome assessment is independent of patient treatment.

Class IV: Evidence from uncontrolled studies, case series, case reports, or expert opinion.

316
Q

The meaning of a double blinded trial

A

In a single-blinded trial, the participants don’t know whether they’re receiving a treatment or placebo until the trial is over. In a double-blinded trial, neither the participants nor the researchers know who’s receiving treatment or placebo until the trial is over.Both of these designs are meant to eliminate expectations that could bias the trial results. Placebo-controlled, double-blinded trials are the gold standard of clinical trials.

317
Q

Which of the following findings after doing an interim analysis would warrant extending the study?

a. smaller difference than expected
b. bigger difference than expected
c. no difference than expected
d. increased mortality in the control group
e. can’t remember

A

a. smaller difference than expecte

So you can get more power/less chance of a type ii error